2ちゃんねる ■掲示板に戻る■ 全部 1- 最新50    

■ このスレッドは過去ログ倉庫に格納されています

大学学部レベル質問スレ 13単位目

1 :132人目の素数さん:2019/12/31(火) 14:49:13.58 ID:ASiBPYNx.net
大学で習う数学に関する質問を扱うスレ

・質問する前に教科書や参考書を読むなりググるなりして
・ただの計算は
http://wolframalpha.com
・数式の表記法は
http://mathmathmath.dote ra.net
・質問のマルチポストは非推奨
・煽り、荒らしはスルー

関連スレ
分からない問題はここに書いてね478
http://rio2016.5ch.net/test/read.cgi/math/1511604229/

※前スレ
大学学部レベル質問スレ 12単位目
http://rio2016.5ch.net/test/read.cgi/math/1531805974/

2 :132人目の素数さん:2020/01/01(水) 07:21:58 ID:1fllAx58.net
明けましておめでとうございます。

3 :132人目の素数さん:2020/01/04(土) 23:55:39.27 ID:DEb5TKUW.net
複素関数 1/sin²(z) を不定積分すると -cot(z)+C になると思いますが、この場合、始点をpi/2、終点を-pi/2にすると直線で結んだ場合は途中の経路に原点が含まれて正則じゃない点を通るのでこの経路では積分不可能で、そこを避けるように経路を取ることになり、直線経路にならないのですが、それでもそのことは気にせずに直線の時と同じような積分で良いのでしょうか。

4 :132人目の素数さん:2020/01/05(日) 13:10:31 ID:rWtHJVIR.net
そもそも複素積分で直線経路など要求してない

5 :132人目の素数さん:2020/01/06(月) 13:03:00 ID:uJ3CsbNT.net
何言ってんだコイツ

6 :132人目の素数さん:2020/01/06(月) 13:36:29.00 ID:8MtkoUqY.net
テイラー級数を見て思ったんですが、単項式って何かの空間の基底になってますか?

7 :132人目の素数さん:2020/01/06(月) 13:49:11 ID:Uj8H4yY8.net
単項式しだいだろ

8 :132人目の素数さん:2020/01/06(月) 16:53:40 ID:D0sxZOqE.net
Pr(θ)をポアソン核、ポアソン積分をPr*f(θ) := ∫_0^{2π} (Pr(θ-Φ))f(Φ)dΦ とします
ここからが分からないのですが、次の変形
( Pr*f(θ) ) - f(θ) = ∫_0^{2π} Pr(θ-Φ)(f(Φ)-f(θ))dθ
が成り立つのは何故なのでしょうか
特に、何故 f(θ) がPr(θ-Φ)で括られるのかが分かりません

9 :132人目の素数さん:2020/01/06(月) 17:09:35.65 ID:8MtkoUqY.net
>>7
単項式は1, x, x^2, x^3, ...でう

10 :132人目の素数さん:2020/01/06(月) 17:34:17.30 ID:g5QBq4Ak.net
>>8
∫Prdθ=1だからじゃね?

11 :132人目の素数さん:2020/01/06(月) 19:11:01.71 ID:M4pZMN5j.net
複素数の超越数ってあんの?

12 :132人目の素数さん:2020/01/06(月) 19:16:35.74 ID:uJ3CsbNT.net
>>6
つルジャンドル多項式

13 :132人目の素数さん:2020/01/06(月) 19:56:31.83 ID:ZXWjDbLg.net
>>9
それらで張られる空間はk[x]
無限和も許すような基底であればC^ωかな

14 :132人目の素数さん:2020/01/06(月) 20:04:54.44 ID:WL9tARX2.net
>>10
しばらく悩んで理解しました
言われてみれば単にこれだけですね
ありがとうございます

15 :132人目の素数さん:2020/01/06(月) 20:40:56.43 ID:TlFZt9uI.net
A, B が正定値エルミート行列で、AB=BAをみたすとき、
行列の平方根 (√A) と (√B) が (√A)(√B) = (√B)(√A) をみたすことを証明できないんですが、教えてください。

16 :132人目の素数さん:2020/01/07(火) 02:40:49.88 ID:M3hr/9Ji.net
対角化したら

17 :132人目の素数さん:2020/01/07(火) 03:50:23.45 ID:dGX/W9Ay.net
>>16
ありがとうございます。積年の疑問が雲散霧消しますた。

18 :132人目の素数さん:2020/01/07(火) 14:19:21.99 ID:ZH/6DRT2.net
次のフーリエ変換を求めよ
1. f(x) = 1 (if |x|≦1)
f(x) = 0 (if |x|≧0)

2.f(x) = sinx*cosx/x

どなたかよろしくお願いします

19 :132人目の素数さん:2020/01/07(火) 14:20:00.56 ID:M3hr/9Ji.net
おいおい同時対角化できるんかよ
まあ、できるんだが

20 :ちびでぶハゲニート:2020/01/07(火) 19:08:53.59 ID:DW9cBFgT.net
質問です
z=0の周りで次の周回積分を正の向きに計算したい

兎xp(z) sinz/(z^2) dz

abs(z)=r
なる円を考え、かつ、rが十分小さいとき、
sinz≒zより

兎xp(z) sinz/(z^2) dz
≒兎xp(z) /z dz
=2πi

とやっていいものでしょうか…

21 :132人目の素数さん:2020/01/08(水) 16:08:41.08 ID:PHmBGYAC.net
理由がダメ
(sin z - z)/z^2 が正則にしとけ

22 :132人目の素数さん:2020/01/08(水) 18:46:35.90 ID:fublbSte.net
>>13
テイラー展開の係数は点ごとに代わるから、C^ωというのは1点における茎みたいなやつになるんでしょうか?

23 :132人目の素数さん:2020/01/08(水) 19:26:48.34 ID:LpZINTuE.net
知恵袋
https://chiebukuro.yahoo.co.jp/my/myspace_quedetail.php?writer=1043512917
https://chiebukuro.yahoo.co.jp/my/myspace_quedetail.php?writer=1147736549
二つのハンドルで質問しまくったがバカにされ始めたことを気づいたのか
https://detail.chiebukuro.yahoo.co.jp/qa/question_detail/q13218505343
で ID を非公開にwwwwwwwwwwwww

ここでも FFT

教えてgoo venomctun、 captain06
https://oshiete.goo.ne.jp/qa/11433028.html
ttps://oshiete.goo.ne.jp/qa/11423826.html
ttps://oshiete.goo.ne.jp/qa/11426289.html
ttps://oshiete.goo.ne.jp/qa/11418282.html
ttps://oshiete.goo.ne.jp/qa/11418068.html
ttps://oshiete.goo.ne.jp/qa/11417112.html
ttps://oshiete.goo.ne.jp/qa/11417088.html
ttps://oshiete.goo.ne.jp/qa/11418410.html
ttps://oshiete.goo.ne.jp/qa/11420102.html

24 :132人目の素数さん:2020/01/08(水) 22:54:10.93 ID:VmCviJFS.net
質問です
z=0の周りで次の周回積分を正の向きに計算したい

兎xp(z) sinz/(z^3) dz

abs(z)=r
なる円を考え、かつ、rが十分小さいとき、
sinz≒zより

兎xp(z) sinz/(z^) dz
≒兎xp(z) /z^2 dz
=0

とやっていいものでしょうか…

25 :132人目の素数さん:2020/01/08(水) 22:57:00.52 ID:r7hTvrOd.net
>>24
>>21を使いましょう。

26 :132人目の素数さん:2020/01/08(水) 23:21:14.75 ID:HNLdfCNE.net
D={x=x(t):x∈C^2([a,b]),x(a)=x(b)=0}と定め、Dにおける対称な二階線形微分作用素Tを
Tx=px''+p'x'+rx (p=p(t))と定めるとき、
Tの固有値はp(t)>0なら上に有界、p(t)<0なら下に有界であることを示せ
お願いします

27 :132人目の素数さん:2020/01/09(木) 00:19:46.47 ID:UM7VaLzp.net
>>26
rは定数?

28 :132人目の素数さん:2020/01/09(木) 00:38:37.59 ID:Ds5ttxyN.net
>>27
すみません
特に指定が無かったのですが出題範囲的に恐らくr(t)の略かと

29 :132人目の素数さん:2020/01/09(木) 02:32:10.42 ID:YAHlkjQy.net
あれ?p=1,r=0で反例になるような?

30 :ちびでぶハゲニート:2020/01/09(木) 08:20:35.79 ID:KxyVSemY.net
>>21
>>23

ありがとうございます
理解できた気がします

https://i.imgur.com/Tz4KZkR.jpg

31 :132人目の素数さん:2020/01/09(木) 14:47:25.11 ID:PcAbLiHe.net
誰やねん

32 :132人目の素数さん:2020/01/09(木) 21:57:18.45 ID:9a3qIGvn.net
>>13
k[[x]]

33 :132人目の素数さん:2020/01/09(木) 21:58:03.41 ID:9a3qIGvn.net
>>22
1+2x+3x^2+4x^3+5x^4+…
がC^ωと?

34 :132人目の素数さん:2020/01/09(木) 22:13:52.21 ID:m7YrTlcc.net
<u,n,k,o|unko=1>
うんこ群

35 :132人目の素数さん:2020/01/11(土) 21:21:12.31 ID:jM1ohGTi.net
R:微分作用素のレゾルベント、{φ_k}:Rの固有関数列
このとき、任意の連続関数fに対してParsevalの等式
納k=0] |(f,φ_k)| = || f ||^2
が成立、つまり{φ_k}が完全であることを示して下さい

36 :132人目の素数さん:2020/01/11(土) 21:45:51.55 ID:kJ5ju2rK.net
>>35
門外漢でよくわからないんですが、このような設問の場合、関数空間は自動的に決まるんですか?
微分作用素Dと固有方程式Df=λfについて‥で自動的にヒルベルト空間は指定されるんですか?

37 :132人目の素数さん:2020/01/12(日) 08:17:35.39 ID:GJsd9kX9.net
質問
数学をするのにもし次のどちらかを使うとするならどっちを選ぶ?
液晶タブレット
DPT-RP1等のデジタルペーパー

ノートとしての使い勝手を考えたらデジタルペーパーの方がいいけど値段が高いし用途がかなり限定される
一方液晶タブレットは5万円台もあるし用途は数学以外もある

38 :132人目の素数さん:2020/01/12(日) 10:06:36 ID:r4lOvqY/.net
>>37
デジタルペーパーって店で触ったことしかないけどメリットあるのかな
「iPadよりしょぼいものがiPadより高く売られている?!なぜ?!」ってびっくりした

39 :132人目の素数さん:2020/01/12(日) 10:23:03 ID:r4lOvqY/.net
多変数の複素積分ってRからR^nに一般化するみたいに普通にやったらいいんですか?
ネット上にPDF落ちてませんか?

40 :132人目の素数さん:2020/01/12(日) 14:59:38.27 ID:UJLLSll+.net
>>36
決まらないから出題者を忖度するしか無いわな

41 :132人目の素数さん:2020/01/12(日) 15:18:01.87 ID:afNssQeY.net
リュウビル型微分方程式の固有関数の直交性問われてるのかな?

42 :132人目の素数さん:2020/01/12(日) 17:18:45.84 ID:7WosHAht.net
>>41
その話だとするとどんな空間になるんですか?

43 :132人目の素数さん:2020/01/12(日) 17:41:13.39 ID:GJsd9kX9.net
>>38
宣伝されてた情報によると、バッテリーが数週間持つ、書き味が紙に近い らしい

44 :132人目の素数さん:2020/01/12(日) 17:54:35 ID:VU1Ix1FE.net
髪に書く方が楽じゃあないかね?
探すときも早いし。

45 :132人目の素数さん:2020/01/12(日) 20:08:03.31 ID:ezgETAIB.net
写真3枚目のAフーリエ正弦級数を求めるところでどのようにこの値を出したのか、途中計算込みで教えていただけないでしょうか、、、
https://i.imgur.com/3YINhDc.jpg
https://i.imgur.com/G78rDwM.jpg
https://i.imgur.com/vdjnc63.jpg

46 :132人目の素数さん:2020/01/13(月) 18:22:31.46 ID:cMSUvxxO.net
英語で検索したらわかったんだけど多変数の複素積分はかなりレベルが高いらしい
まだやめとく

47 :132人目の素数さん:2020/01/13(月) 18:39:35.24 ID:47SmkU3j.net
賢明な判断です

48 :132人目の素数さん:2020/01/13(月) 20:38:33.20 ID:y+Nah/OD.net
来世で頑張れ

49 :132人目の素数さん:2020/01/14(火) 11:43:19.83 ID:xSNOoyFb.net
来週頑張れ

50 :132人目の素数さん:2020/01/19(日) 14:39:47 ID:5EDEK9Kw.net
多変数の積分の変数変換の公式(ヤコビ行列式のやつ)ってシンプルで短い証明ないんですか?

51 :132人目の素数さん:2020/01/19(日) 14:55:34 ID:03Tx9jz+.net
つ微分形式

52 :132人目の素数さん:2020/01/19(日) 15:30:40.69 ID:sYEpf36R.net
Xを(0,2)で有界連続関数全体とするとき、||f||=sup|f(x)|とノルムを定めてバナッハ空間とする
ここで次の線形作用素TがX上でも有界線形作用素となるか判定して下さい
以下、全てx∈(0,2)です

Tf(x)=f(x)/x, Tの定義域は{xg(x);g∈X}
Tf(x)=∫[0,x]f(t)dt, Tの定義域はX
Tf(x)=f'(x), Tの定義域は{f∈C^1(0,2);||f'||<∞}

全て成立してるような気がするのですが判定せよなので1つくらい不成立例が混ざっててもおかしくないんですよね
どなたかわかる方いたらお願いします

53 :132人目の素数さん:2020/01/19(日) 15:55:33.30 ID:umPR//wq.net
一番最初はムリやろ
f(x)=min{ax,1}
が任意のaで||f||≦1だけど||Tf||は非有界じゃね?

54 :132人目の素数さん:2020/01/19(日) 15:56:37.28 ID:umPR//wq.net
最後もムリ。
fx)=sin(ax)
でダメじゃね?

55 :132人目の素数さん:2020/01/19(日) 16:03:32.67 ID:ZPEc7LR+.net
命題1
https://imgur.com/a/RkqdHXj


https://imgur.com/a/UCtEdJr
https://imgur.com/a/6Bzz4dA

系の証明が端折ってて不明瞭なので質問
系の証明で命題1はどのように使われてますか?

56 :132人目の素数さん:2020/01/19(日) 16:50:02 ID:sYEpf36R.net
>>53
>>54
有界性ってこの場合fに依存しないように上から評価出来ないとでしたね
ありがとうございます解決しました

57 :132人目の素数さん:2020/01/19(日) 22:39:15.83 ID:6wuluGga.net
>>50
線形変換の体積比が行列式になることの証明て複雑か?

58 :132人目の素数さん:2020/01/19(日) 23:44:35.56 ID:flshczPE.net
>>57
線形性と交代性から

59 :132人目の素数さん:2020/01/20(月) 01:45:11 ID:EYxlQYt2.net
そもそも行列式が符号つき体積なので

60 :132人目の素数さん:2020/01/20(月) 11:32:18 ID:fhxaa2PP.net
非線形の話だっつうの
線形代数じゃねえよ

61 :132人目の素数さん:2020/01/20(月) 11:38:25.50 ID:JQTUimdS.net
>>60
線形変換だろ?

62 :132人目の素数さん:2020/01/20(月) 13:12:53 ID:fhxaa2PP.net
ふざけていやがる
証明しようとしてみれば分かるけど、変数変換の写像(一般に非線形)によって直方体が直方体に移らない上に
点によって拡大率(ヤコビ行列式)が違うから分割が分割に綺麗に対応しない
局所的な議論に比べてかなり複雑になる

63 :132人目の素数さん:2020/01/20(月) 13:22:36 ID:VbzcAbmd.net
そもそも微分てのは局所的に線形ってことだがなー

64 :132人目の素数さん:2020/01/20(月) 15:27:45.64 ID:JQTUimdS.net
>>62
>局所的な議論に比べて
フンパン
局所的な議論をするのだが

65 :132人目の素数さん:2020/01/22(水) 00:45:16.78 ID:GMkbI24W.net
次のRのイデアルIが極大イデアルか判定するのはどうすれば

R=Z[i]に対してI=(97)
R=Z[X](多項式環)に対してI=(7,X)

66 :132人目の素数さん:2020/01/22(水) 00:54:21.98 ID:GMkbI24W.net
すみません下のは解決しました

67 :132人目の素数さん:2020/01/22(水) 01:26:29.36 ID:GMkbI24W.net
皆さんありがとうございます
上も自決しました

68 :132人目の素数さん:2020/01/22(水) 01:34:00.73 ID:sBbV/WRd.net
p,q:Z[i]→Z/97
をp(i)=22、q(i)=-22で定めるともちろん(97)の原点はどちらも0イデアルに移されるのでIはkerp、kerqの両方に含まれる。
しかしp(22+i)=44≠0、w(22-i)=0によりkerp≠kerq。
もし(97)が極大イデアルなら(97)=kerp=kerqとなって矛盾。
∴(97)は極大イデアルではない。

69 :132人目の素数さん:2020/01/22(水) 01:34:07.94 ID:YydrU0ZT.net
97 = (4 + 9i)(4 ? 9i)で既約元じゃないから素元じゃない

70 :132人目の素数さん:2020/01/26(日) 19:26:50.68 ID:KPF5SGO/.net
次の普遍被覆空間を求めて下さい
簡単な説明もあると助かります

R^2\{(0,0)}
S^2
S^2\{(0,0,1)}

71 :132人目の素数さん:2020/01/26(日) 22:07:15.38 ID:Ro1H2zIO.net
>>70
π:R^2→R^2-O:(x,y)→e^x(cosy,siny)
S^2, S^2-Nは自明

72 :132人目の素数さん:2020/01/27(月) 20:48:25 ID:GZww43cK.net
任意に与えられた超越数から、新たな超越数を構成する手続きって存在しますか?

73 :132人目の素数さん:2020/01/27(月) 21:00:30 ID:GZww43cK.net
>>72
T:=超越数全体のなす集合
Map(T):={写像f:T→T全体}としたときの、Map(T)の性質やf∈Map(T)が連続である時のfの性質とか気になるんだが、なんか分かってる事ってありますか?

74 :132人目の素数さん:2020/01/28(火) 00:08:44.51 ID:2Xgr28xI.net
>>72
x:超越数→x+1:超越数

75 :132人目の素数さん:2020/01/28(火) 14:31:00.41 ID:eAc+RoFn.net
無限少数で表して各桁に0を挟む
これが超越数って証明はどうすればいいのだろう?

76 :132人目の素数さん:2020/01/28(火) 14:38:27.48 ID:KMW2IGzj.net
>>75
どゆこと?
f(x)が十進表示の各桁の間に0を挟む関数
f(123.4567‥)=10203.4050607‥)
としてxが超越数→f(x)も超越数を示すの?
そもそもそんなの成立するん?

77 :ちびでぶハゲニート:2020/01/29(水) 08:29:13.90 ID:n8a8X7Bw.net
成立するんじゃない?
循環小数に0をはさんでも有理数になることをしめせばいい

0.10402080507010…
は有理数か。

成り立ちそう

78 :132人目の素数さん:2020/01/29(水) 09:00:34.21 ID:ChU8VoG8.net
>>77
何で有理数?

79 :132人目の素数さん:2020/01/29(水) 09:48:01 ID:bSeLoPS+.net
しかも→逆だし。
有理数⇔有理数は自明だけど超越数、超越数はどっち向きも相当ムズイ。
無理数と超越数の違いもわからんカスの自作問題か?

80 :132人目の素数さん:2020/01/29(水) 17:27:03 ID:HCB0YwtK.net
具体的な数学の問題ではないのですが、他学部出身ですが数学に興味があって、高校数学の復習が終わって、微分積分、線形代数、集合・位相、微分方程式まで学習しました。
数学科卒の最低レベルまで数学の知識を広めたいのですが、数学科卒といえるには、あと《最低限》何を勉強すれば宜しいでしょうか?

近い将来、転職する際に、数学科卒業程度の数学を独学した旨のアピールをしたいです。

81 :ちびでぶハゲニート:2020/01/29(水) 17:36:53 ID:n8a8X7Bw.net
>>79
あー(´・ω・`)
たしかに無理数なのは言えそうだけど
超越数は難しそうですね

でも真偽がかんたんにはわからない問題ってそれだけで価値ありそうじゃないですか?

82 :132人目の素数さん:2020/01/29(水) 17:41:25 ID:h7J/ibca.net
群論 環論 体論 ガロア理論 数論 ホモロジー代数 代数幾何
ベクトル解析 複素解析 フーリエ解析 測度論 関数解析 偏微分方程式 確率論
多様体論 位相幾何 微分幾何 微分位相幾何 表現論

83 :132人目の素数さん:2020/01/29(水) 19:20:07 ID:ChU8VoG8.net
>>81
真偽が分からない問題なら腐るほどある
価値があるかどうかは問題に取り組んでみて
話が発展していきそうかそうでないかで考えるべき
難易度とはまた別のパースペクティヴで見るよ

84 :ちびでぶハゲニート:2020/01/29(水) 19:27:28.14 ID:p6ipcoSG.net
>>83
おっしゃる通りなんですが、
超越数かどうかの判定ってのがさほど話が広がるものじゃないイメージもあったりします

85 :132人目の素数さん:2020/01/30(Thu) 13:47:45 ID:EjxXc1gy.net
まだ未発達な分野だろうから開拓は大変だね

86 :ちびでぶハゲニート:2020/01/31(金) 15:10:59 ID:ZhnajJxp.net
∫dx 1/(x^2+1)=∫dx (1/(x-i)-1/(x+i))/2

みたいな感じで定積分求められますかね(´・ω・`)

87 :132人目の素数さん:2020/01/31(金) 15:26:41.23 ID:TUBecDNT.net
複素対数関数がわかってればね

88 :132人目の素数さん:2020/01/31(金) 15:30:21.96 ID:NEbeyvsi.net
>>86
>)/2
2?

89 :132人目の素数さん:2020/01/31(金) 15:51:07.81 ID:NEbeyvsi.net
>>87
logz=log|z|+iargz
から導出して

90 :ちびでぶハゲニート:2020/01/31(金) 16:20:02.87 ID:ZhnajJxp.net
>>86
求めたいのは不定積分でした

やってみます

91 :132人目の素数さん:2020/01/31(金) 21:46:12.16 ID:NEbeyvsi.net
>>90
0からxまでの定積分を求めるよ

92 :132人目の素数さん:2020/02/01(土) 20:27:43.57 ID:mOhn9+D8.net
漸化式が綺麗な形で表すことの出来る一般校を持つかどうかを判定するアルゴリズムなり定理ってありますか?

93 :132人目の素数さん:2020/02/02(日) 01:09:22.61 ID:YdOZPgST.net
一般校には無理

94 :132人目の素数さん:2020/02/02(日) 01:22:25.10 ID:cz3bS1PT.net
この世には何にも綺麗な形なんて、ない!

95 :132人目の素数さん:2020/02/02(日) 12:29:29.54 ID:YdOZPgST.net
審美眼ないのね

96 :132人目の素数さん:2020/02/03(月) 18:52:50 ID:WxYjwRAT.net
デデキント切断で実数を切断するとき下組に最大値か上組に最小値があることの証明ってありますよね?
これを証明なしで公理として扱ってるサイトが多いのはなんでなのですか?
上限下限の存在も公理としてる場合が多いですがこれも証明できますよね?

97 :132人目の素数さん:2020/02/03(月) 18:57:24 ID:fPctNAEs.net
同値となる公理が多いからどれを公理とするかで変わる

98 :132人目の素数さん:2020/02/03(月) 23:08:18 ID:qSjfeJ+N.net
実数の定義にその事実使ったからわからん

99 :132人目の素数さん:2020/02/03(月) 23:43:40 ID:eRVaXKct.net
証明面倒くさいから

100 :132人目の素数さん:2020/02/03(月) 23:46:09 ID:eRVaXKct.net
>>98
定義には使わないでしょ
実数の定義は有理数の切断からだから

101 :132人目の素数さん:2020/02/04(火) 15:33:16 ID:kJM/Arwr.net
自然数が上に有界であることの証明に上限を使って背理法で示していますがコレって必要なんですか?
自明に見えるんですけど?

102 :132人目の素数さん:2020/02/04(火) 15:37:11 ID:kJM/Arwr.net
間違えました。上に有界でないですね。背理法で有界と仮定してたから混乱してた

103 :132人目の素数さん:2020/02/04(火) 16:33:48 ID:7i12z1UP.net
自然数の定義はどうしている?

104 :132人目の素数さん:2020/02/04(火) 16:59:09.03 ID:kJM/Arwr.net
定義というか1から始まって無限に続くという理解でした
高校でも普通にn->∞としてましたから自明だと思っていました

105 :132人目の素数さん:2020/02/04(火) 23:05:39 ID:jW3Vrn+m.net
自然数とは何かがちゃんと定義できないと数学ではないですね

106 :132人目の素数さん:2020/02/05(水) 12:51:06.25 ID:uj8c/TnE.net
ここって大学レベルなんだよね

107 :132人目の素数さん:2020/02/05(水) 18:59:41.30 ID:xck4ijq4.net
そだよ

108 :132人目の素数さん:2020/02/05(水) 21:02:35 ID:298bnSpu.net
そだねー

109 :132人目の素数さん:2020/02/05(水) 23:13:05.24 ID:XHYFRp+r.net
Michael Spivak著『Calculus on Manifolds』を読んでいます。

この本を読んでいて、思いついた以下の問題の解答をお願いします:


A を R の部分集合とする。
f を R から R への関数とする。
f は A の各点で微分可能とする。

A を含む開集合 B で以下の性質をもつものが存在するか?

B から R への微分可能な関数 g で g(a) = f(a) for all a ∈ A を満たすものが存在する。

110 :132人目の素数さん:2020/02/06(木) 00:46:15.96 ID:Anrgdgxr.net
>>109
小平の解析入門では、その定理が成り立つことを持ってAで微分可能の定義

111 :132人目の素数さん:2020/02/08(土) 12:24:47 ID:UoObQkgU.net
Loring W. Tu著『トゥー多様体』を読んでいます。

実解析的な関数は C^∞ 級です。

C^∞ 級でないとテイラー展開できないからです。

ところが、Tuさんは


実解析的な関数は必ず C^∞ 級である。なぜなら、実解析で学んだように、収束べき級数は
収束範囲で項別微分できるからである。


などと理由を書いています。

これはナンセンスではないでしょうか?

112 :132人目の素数さん:2020/02/08(土) 13:31:50.57 ID:kYb/Jpp8.net
C^∞と解析的は違う。
何冊解析の教科書よめばわかるん?

113 :132人目の素数さん:2020/02/08(土) 14:29:56 ID:ozXSNVVR.net
知能の低いやつに言っても無駄
こいつはきちんと理解することはできない
数学の理解にある程度の知能は必要

114 :132人目の素数さん:2020/02/08(土) 15:59:27 ID:K7Uczxp/.net
マルチは相手すんな

115 :132人目の素数さん:2020/02/09(日) 15:02:54.81 ID:drHYoHKW.net
新入生に 「幼稚園に入園おめでとう!」 って、誰(数学者)の言葉だっけ?

116 :132人目の素数さん:2020/02/10(月) 22:27:23.27 ID:u6CabzO4.net
複素積分による求積法について教えてください。
∫[-∞,+∞] dx sin(x)/x = lim{ε→0} ∫[-∞,+∞] dz sin(z)/(z - iε)
= lim{ε→0} ∫[C+] dz exp(+iz)/2i(z - iε) - ∫[C-] dz exp(-iz)/2i(z - iε) = π - 0 = π

単にコーシーの積分公式使うだけの後半部分はどうでもいいです。
最初の等式の ε→0 で一致するという箇所が気になっています。
そりゃあ一致するだろ...と直感が囁くものの、どうしたら数式で示せるのか分かりません。

117 :132人目の素数さん:2020/02/10(月) 22:48:11.37 ID:HCRp4j1T.net
>>116
まず最初の積分もεついてる方も広義積分可能である事を示す。
コレできなきゃ話しにならん。
すると任意のe>0に対してR>0を

|∫[|x|>R]sin(x)/xdx|, |∫[|x|>R]sin(x)/(x-iε)dx| < e

ととれる。
sin(x)/(x-iε)→sin(x)/x
は一様可積分に収束するから

lim∫[|x|<R]sin(x)/(x-iε)dx=∫[|x|<R]sin(x)/xdx

よって

limsup∫[|x|<R]sin(x)/(x-iε)≦∫[|x|<R]sin(x)/xdx+e,
liminf∫[|x|<R]sin(x)/(x-iε)≧∫[|x|<R]sin(x)/xdx-e。

118 :132人目の素数さん:2020/02/10(月) 23:08:23 ID:u6CabzO4.net
>>117 ありがとうございます
(|x| > R) 部分の見積もり、これは複素積分の積分路変更でいけますね。
ですが...
(|x| ≦ R) sin(x)/(x-iε)→sin(x)/x これは一様収束ではありませんよね?
sin(0)/(0-iε) = 0 < 1 = sin(0)/0 (= lim{x→0} sin(x)/x の意味で)
このギャップをどう処理すべきでしょうか

119 :132人目の素数さん:2020/02/10(月) 23:18:14 ID:1+8rzOtr.net
>>118
一様可積分なのでlimは中に入ります。(Lebesgueの収束定理)

120 :132人目の素数さん:2020/02/10(月) 23:36:07 ID:u6CabzO4.net
ああ、すみません「一様収束」とは違うのですね。
「一様可積分」ちらっと調べて見ましたが難しいです...、もうちょっと解析初歩の知識で示せないのでしょうか?

121 :132人目の素数さん:2020/02/10(月) 23:45:19 ID:1+8rzOtr.net
|sin(x)/x|, |sin(x)/(x+iε)|≦1
より任意のe>0に対し十分小さいrをとれば
|∫[|x|<r]sin(x)/xdx|<e, |∫[|x|<r]sin(x)/(x+iε)dx|<r
を用いてさっきと同じように示せなくはないけど一様可積分くらい使いこなせないとどのみち話にならない。
この問題はなんとかなってもいつか破綻する。

122 :132人目の素数さん:2020/02/10(月) 23:46:05 ID:EaGEQ+Vz.net
https://agree.5ch.net/test/read.cgi/sec2chd/1580220578/354
どうやら板の管理人が見ていないようだ
板の管理人が見ている場所を知っている人がいたら教えて欲しい、もしくは代わりに取り次いで欲しい

123 :132人目の素数さん:2020/02/11(火) 00:14:52 ID:NslqUA1u.net
>>121
ありがとうございます。なんとか納得できました。

124 :132人目の素数さん:2020/02/11(火) 00:39:23 ID:k9bizDpK.net
一様可積分って記憶にないからググってしまった
確率変数族の話なのね

125 :132人目の素数さん:2020/02/11(火) 12:05:43 ID:NslqUA1u.net
>>121 氏の 「一様可積分」て、どうも確率論のそれとは違うようで
広義積分の一様収束 (広義一様収束) を指しているようですね
杉浦 解析入門I p.319 に定義があり、少し先には sin(x)/x の積分も出てきます。
ほぼ積読本でしたが、ちゃんと勉強しようと思いました。

126 :132人目の素数さん:2020/02/11(火) 16:07:57 ID:k9bizDpK.net
まあ確率変数族でも関数族でも同様か

127 :132人目の素数さん:2020/02/14(金) 13:48:25 ID:7RCb7w1U.net
質問があります。
統語論的完全性がいまいち掴みきれません。
演繹システムとそれに対応するモデルがあるとして、モデルにおいて論理式となるものaに対応するのが演繹システムの論理式#aとする。
統語論的完全性とはaが閉論理式のとき演繹システムで#aまたは¬#aのどちらか一方を演繹できる。
という理解でよい?

128 :132人目の素数さん:2020/02/14(金) 13:53:19 ID:ekmNRCqQ.net
完全である
⇔ 任意のモデルで真であるものは証明できる

の話ではなく?

129 :132人目の素数さん:2020/02/14(金) 15:40:03 ID:7RCb7w1U.net
>>128
うん。
あるサイトとかの説明をみるとざっくりこんな感じで説明してあって。

>閉論理式AについてAが証明できるか¬Aが証明できるかのどちらかである。

なんかわかるようでわからないというか。

仕事中だから短いレスすまん。
また夜に。

130 :132人目の素数さん:2020/02/14(金) 16:40:22 ID:ekmNRCqQ.net
>>129
それはあってるとも間違ってるとも。
数理論理学で完全性、不完全性といったらゲーデルの定義したそれだと思うけど、その場合

Lが完全⇔Lの任意のモデルで真であるものは証明可能。

Lが不完全⇔Lの命題でそれ自身もその否定も証明できないものが存在する。

の事を指す事が多いけど、その場合"不完全"が"完全でない"の事を意味してない。
例えば自然数論は上の意味で"完全"(1階述語論理は完全)かつ"不完全"(不完全性定理)。
しかしそのサイトは"不完全でない"事を"完全"と言ってるみたい。
間違いではないんだろうけど、どうなんだろ?

131 :132人目の素数さん:2020/02/14(金) 18:53:03.07 ID:7RCb7w1U.net
>>130
付き合ってくれてありがとう。
前提として、完全にも色々あるのはうっすらとは理解している。
あとそのサイトがあれなのではなくて理解できてないこちらが全面的に悪い。

>しかしそのサイトは"不完全でない"事を"完全"と言ってるみたい。
>間違いではないんだろうけど、どうなんだろ?

一応そのサイトの説明では。(省略した形で)

>意味論的完全性 モデルにおいて真である命題に対応する論理式は証明できる
>統語論的完全性 全ての閉論理式AについてAが証明できるか¬Aが証明できるかのどちらかである

>完全性定理の方は「1階述語論理では健全性および意味論的完全性が成り立つ」というもの。ただしこのとき「真」というのはその形式的体系に当てはまる全てのモデルについて「真」であるという意味。
>ゲーデルの第一不完全性定理は「自然数の公理系を含む無矛盾な公理系」についての統語論的完全性を否定したもの。

とある。
なおサイトではここから掘り下げていく形になってる。
けれどもそこはまだ読んでない。

で、自分が躓いたのは上記の統語論的完全性のとこ。
「全ての閉論理式」ってのがなんとなく掴めない。
モデルにおいて閉論理式となるA→演繹システムで#Aまたは#¬Aのどちらか片方を演繹できる。
という捉え方でよいのかな?と。

とんちんかんなことを言っていたらごめん。
こちらモデルとかの概念をいまいち把握できてないから。

132 :132人目の素数さん:2020/02/14(金) 19:46:30 ID:ekmNRCqQ.net
>>131
私の理解の上では"モデルにおいて閉論理式になる"という考え方は無いと思う。
ある論理式φが閉論理式⇔φに現れる変数か全て束縛記号∀か∃のどちらかで束縛されている。
したがって閉論理式はかくモデルごとに真であるか偽であるかが決まる。
統語論的完全性とは

どんな閉論理式φを持ってきてもφかnotφのいずれかが証明できる。

という意味だと思う。
記号や言葉の定義が人によって微妙に違ったりするみたいだからなんとも言えないけど。

133 :132人目の素数さん:2020/02/14(金) 20:32:36 ID:7RCb7w1U.net
>>132
本当にありがとう。

>私の理解の上では"モデルにおいて閉論理式になる"という考え方は無いと思う。

うん、無いのだと思う。
こちらが大きな勘違いをしてる。
勘違いをしているから理解できない。

助言を乞いたいところだけどとりあえず本にあたってみる。
感謝!

134 :ちびでぶハゲニート:2020/02/14(金) 22:58:54.59 ID:KdjyD+uu.net
基礎論はかっこいい

135 :132人目の素数さん:2020/02/14(金) 23:26:00.54 ID:TDJhfa4J.net
統語と意味との区別をつけるところからですかね

普通の日本語とかの言語では、文法的には正しいけど何言ってるかよくわからんみたいなことがありますよね

言語には、記号の並べ方が規則通りかという面と、その意味は何かという二つの側面があるわけです

形式論理も同じです

論理式を作る際にどのようなものが論理式になれて、また論理式をいじる操作としてはどのようなものがあるのか、という統語的な側面

できた(閉)論理式の真偽はどうなっているのか、という意味的な側面

ここで大事なのは、統語的な側面は誰にとっても不変的ですが、論理式の意味、すなわち論理式の真偽というのは人によって変わるということです

各論理式に真偽を当てはめる規則がモデルですね

車は左側通行である
日本では真の命題ですが、アメリカでは違いますね
モデルが違えば、真偽は変わるというわけですね

136 :132人目の素数さん:2020/02/15(土) 03:30:17.32 ID:hYzDSWfT.net
論理学入門できない(;´Д`

137 :127:2020/02/15(土) 13:25:11 ID:D1vDdDsh.net
本を読むとかいっておきながら舞い戻ってきた。
どうやら数学の本は自分にとっては睡眠導入薬であるようだ。
というわけで皆さんのレスを参考にして、これだ!というのに辿り着いた。

どなたかこの理解で正しいのか教授してほしい。

記号列    演繹システムの言語を並べたもの。
論理式    演繹システムの言語を「ある文法に則って並べた」もの。
閉論理式   自由変数を含まない論理式。
ただの記号列 論理式に該当しない記号列。
自由変数   ∃∀で束縛されていない変数。

演繹システムの言語を組み合わせて(推論規則は無視)作られる記号列にはその様式によって論理式や閉論理式に分類できる。
統語論的完全性とは「この閉論理式に該当する様式を持った記号列aについて、演繹システムはaまたは¬aのどちらかを証明できる」。
でよいのかな?
そしてモデルとやらはあくまでも論理式に真偽を当てはめる規則であり、この統語論的完全性とは無関係である、と。

…また夜に。

138 :127:2020/02/15(土) 13:29:29 ID:D1vDdDsh.net
>>135さんありがとうございます。

139 :132人目の素数さん:2020/02/15(土) 16:22:59 ID:hfGcPsMk.net
記号列    演繹システムの記号並べたもの。
論理式    演繹システムの記号を「ある文法に則って並べた」もの。
記号列は論理式か「論理式でない記号列」のどちらか
論理式は閉論理式か「自由変数を含む論理式」のどちらか
統語論的完全性が成り立てば意味論的完全性(任意のモデルで成り立つ)も成り立つ

140 :132人目の素数さん:2020/02/15(土) 17:22:11 ID:zMKPGFwr.net
>>139
>統語論的完全性が成り立てば意味論的完全性(任意のモデルで成り立つ)も成り立つ


意味論的完全性誤解してません?

証明可能なものが任意のモデルで成り立つというのは、健全性定理で、これが成り立つのはある意味当たり前なんですけど

非自明なのはその逆で、正しいものが全て証明可能かどうかです

141 :127:2020/02/15(土) 20:08:43 ID:D1vDdDsh.net
>>139
ということは>>137のノリで良いということだろうか。
致命的に間違ってたら誰か違うとツッコミいれてくれれば嬉しい。

>統語と意味との区別をつけるところからですかね
>論理式を作る際にどのようなものが論理式になれて、また論理式をいじる操作としてはどのようなものがあるのか、という統語的な側面
>できた(閉)論理式の真偽はどうなっているのか、という意味的な側面

自分はこの区別を誤ってたんだと思う。

>統語論的完全性が成り立てば意味論的完全性(任意のモデルで成り立つ)も成り立つ

この辺りのことはこれから読んでいこうかと。
話に参加したいけれどもモデルってのがなんかモヤモヤしてるという。

142 :132人目の素数さん:2020/02/15(土) 20:23:48 ID:zMKPGFwr.net
上で書いたように、簡単に言えばモデルは論理式に真偽を付与する規則なわけですね

日本とアメリカで車が左側通行して良いかが変わると


統語と意味が異なるということさえ頭に入れておけば、モデルの定義くらいなら難しくないはずですから勉強してみると良いでしょう

それが統語と意味の理解にも繋がるはずです

143 :127:2020/02/15(土) 22:13:54 ID:D1vDdDsh.net
レス本当にありがとう。

>>142
>上で書いたように、簡単に言えばモデルは論理式に真偽を付与する規則なわけですね

こう言い切ってくれると、なんだかもうわかった気がしてくる。
>>135を念頭において読み進めてみる。

144 :132人目の素数さん:2020/02/16(日) 11:45:55 ID:1G9nri7d.net
「証明」は
シンタックスの世界では特定の文字列変換で移りあうこと
セマンティクスの世界では真か偽かを公理から導けること

でいいの?

145 :132人目の素数さん:2020/02/16(日) 12:00:47 ID:cQrn41UX.net
>>144
多分違う。
セマンティックスとは意味論、実際に言葉に"集合"や"写像"を割り当てて主張が成立しているかどうか実験してみる事。
推論規則で公理から演繹できるかどうかではなく、実例で確かめてみる事が意味論。

146 :132人目の素数さん:2020/02/16(日) 12:02:03 ID:6bgZlRh0.net
だからセマンティックスの意味では証明などありません。
実例で成り立ってるからどうかダイレクトにみる。

147 :132人目の素数さん:2020/02/16(日) 15:45:34 ID:OgSubFk2.net
>>141
「論理システム」のモデルとは特殊例のこと
大抵は集合論を基にして特殊な集合だけを集めてモデルを作る
「論理システム」の公理系を満たすような集合だけでモデルを作るから
元の「論理システム」で証明できることは全部成り立つが
特殊な集合の選択により証明できない事も成り立つようにできる
「ある命題」が成り立つモデルも否定するモデルも作れたなら
その命題は元の公理系と独立なことが証明される

148 :132人目の素数さん:2020/02/16(日) 17:34:05 ID:vbyzF2m/.net
トポロジーっていったら一般的には代数的トポロジーのことをいうの?

149 :127:2020/02/16(日) 20:01:18.27 ID:4gzXG3T5.net
>>147
レスありがとう。

趣味の独学なので躓くとそこで止まってしまって。
本音ではモデルについて納得できるまで会話したいところだけれども。
知識が不足しているのでとりあえず地道にまずは自分で。

>「論理システム」のモデルとは特殊例のこと
>大抵は集合論を基にして特殊な集合だけを集めてモデルを作る

持ってる本が初心者向けのためかモデルについても説明があるんだけど集合とか使って説明してないという。
とりあえず集合論を絡ませたとこまで自分でまずはやってみることにする。

150 :132人目の素数さん:2020/02/17(月) 01:40:55.18 ID:vqCEZ7Nl.net
集合論を基にしてたのは昔の話だからなー
その後、圏論を基にするのが流行ったはずだけど、今はどうなんだろ?

151 :132人目の素数さん:2020/02/17(月) 11:32:28.99 ID:2rd9yjjt.net
>>150
結局集合論だよ

152 :132人目の素数さん:2020/02/17(月) 13:02:17.13 ID:OWHQ5GSG.net
数理論理学関連でちょっと気になった質問があるんだが、
命題結合子はシェーファーの縦棒 | だけで足りるという事実はよく知られてますが、
命題の公理(公理図式)は最小で何個必要なんですかね?
ヒルベルトの公理系では公理図式は3個なのだが、1or2個で済むような公理系ってあり得るんですかね?
当然だが、1階述語論理と同等な公理系という条件の下での最小の公理の数の話です。

153 :132人目の素数さん:2020/02/17(月) 13:06:49.80 ID:OWHQ5GSG.net
>>152
あ、このような質問の仕方だと、ゲンツェン流のシーケントを使った体系だと、(推論規則が沢山あるおかげで)公理図式がA⇒Aのたった1つですね。
じゃあ、質問を変えてみます。
公理図式と推論規則を合わせた総数が最小となる1階述語論理の体系ってどんなのがありますか?

154 :132人目の素数さん:2020/02/17(月) 13:21:13.07 ID:Ts4bAXWN.net
全然詳しくないですけどなんか最小論理とかいうのがあるみたいですよ

あなたのいう意味で最小かは知りませんけど

155 :132人目の素数さん:2020/02/17(月) 13:23:13.53 ID:2rd9yjjt.net
>>154
古典論理と同等になるための最小のという質問だろう
最小論理は排中律も爆発律もない
最小論理が最小ということもなく
もっと減らしたものを考えるのに妨げはないよ

156 :132人目の素数さん:2020/02/17(月) 14:08:22.31 ID:/HnwZz/g.net
>>153
一階述語なん?
∀とか∃とかの推論則も入れたら一個や二個じゃ済まない希ガス

157 :132人目の素数さん:2020/02/17(月) 15:29:47.82 ID:vqCEZ7Nl.net
最小論理から更に減らした原始論理(primitive logic)てのもあったな
書物では見た事ないが名古屋大学で名誉教授の小野勝次さんの講義で研究中と言ってた
否定がなくても結構いろいろできるとか

158 :132人目の素数さん:2020/02/17(月) 20:40:21.57 ID:n7wiPCWM.net
出鱈目逝ったもん勝ちのスレ

159 :132人目の素数さん:2020/02/17(月) 21:00:12.77 ID:2rd9yjjt.net
命題論理だけに限って考えたら
論理記号は1つで足りるから
それに関する導入と除去の公理を2つ用意すれば
あとは二重否定の除去とMPだけで何とかならんか?

160 :132人目の素数さん:2020/02/17(月) 22:19:21.08 ID:OWHQ5GSG.net
>>158
思考停止のゴミは要らないから引っ込んどけ

161 :132人目の素数さん:2020/02/17(月) 22:31:04.01 ID:Nzms6mON.net
wikiには公理3つ、推論則1つの命題論理が紹介されてるな。
https://ja.m.wikipedia.org/wiki/%E5%91%BD%E9%A1%8C%E8%AB%96%E7%90%86

162 :132人目の素数さん:2020/02/17(月) 22:34:08.36 ID:Nzms6mON.net
あら?演繹定理の項には公理2つ、推論則1つの体系が紹介されてる。
https://ja.m.wikipedia.org/wiki/%E6%BC%94%E7%B9%B9%E5%AE%9A%E7%90%86

163 :132人目の素数さん:2020/02/17(月) 23:20:34.98 ID:OWHQ5GSG.net
そういえば必要な公理の数というのは公理の独立性の話ですね、結局は。
ヒルベルトの公理系の3つの公理図式は確か独立だったから減らしようがない

164 :132人目の素数さん:2020/02/18(火) 00:48:18 ID:ZlcMzP2c.net
>>163
それはどうかな
爆発律+排中律=二重否定除去
だけど
爆発律と排中律は独立
爆発律と排中律を持っていても
独立だから外せないけど
両方外して二重否定除去を入れれば問題ない

165 :132人目の素数さん:2020/02/18(火) 00:51:26 ID:ZlcMzP2c.net
推論規則でMPあるいはそれに同等なものは流石に外せないと思う
他の推論規則からMP出せるような気がしないけど
論理包含の代わりになる論理演算で同等なモノはできるのかな?

166 :132人目の素数さん:2020/02/18(火) 01:02:44.83 ID:4LYhi4WJ.net
ID:OWHQ5GSG = 知ったかニワカ

167 :132人目の素数さん:2020/02/18(火) 01:29:37 ID:ZlcMzP2c.net
>>164
>爆発律+排中律=二重否定除去
ML上での話です
LK=ML+爆発律+排中律=ML+二重否定除去

168 :132人目の素数さん:2020/02/18(火) 17:15:15 ID:hD1N+fXK.net
線形の部分空間がよくわからないなあ
次元が一つ減って原点を通る斜めに傾いたものだと考えていいんでしょか?

169 :132人目の素数さん:2020/02/18(火) 18:05:50 ID:ZaQp2jmm.net
原点通るのはそうだと思いますけど、そこは別にって感じで次元が減るってことですよね、まあ

今までa1e1+a2e2+....+anenと書いてたものを制限してa1e1+...+amemだけを考えると(m<n)

170 :132人目の素数さん:2020/02/18(火) 18:55:16 ID:Jhu30Pl6.net
ここは「大学学部レベル質問スレ」です

171 :132人目の素数さん:2020/02/18(火) 20:38:01 ID:75GKvHxb.net
f : R^n → R^m が C^1 級であることの定義ですが、

f が微分可能で、

f' : R^n → L(R^n, R^m)

が連続である

という定義を採用している本があります。

その本では、

f : R^n → R^m が C^r 級であることの定義は、

f の成分函数が C^r 級であることと定義しています。

一貫性がないですよね。だったら最初から

f : R^n → R^m が C^1 級であることの定義を、

f の成分函数が C^1 級であることとすればいいのにと思います。

172 :132人目の素数さん:2020/02/18(火) 20:50:35 ID:75GKvHxb.net
>>171

に関連した話ですが、

f : R^n → R^m の高階の導関数は定義しないんですね。

173 :132人目の素数さん:2020/02/18(火) 22:43:16.00 ID:ZlcMzP2c.net
>>168
なんでR^3とかの見える範囲で考えてみらんの?
あるいはR[x]_nとかで

174 :132人目の素数さん:2020/02/19(水) 00:19:29 ID:VGIb2vdY.net
集合Xの元の有限列全体の集合ってどういう風に構成すればいいですか?
∪_{n∈N}X^n (Nは自然数全体のなす集合)
でいいんですか?
でも、この集合はつまり、χ:= { X^n | n∈N } という集合が構成できるからこそ、和集合公理によって
∪χ = ∪_{n∈N}X^n
が構成できるという理屈だと思います。
では、χが構成できる根拠は何ですか?

175 :132人目の素数さん:2020/02/19(水) 00:43:04.33 ID:v8JOxEBI.net
置換公理じゃないの?
Nからある一定の方法で構成する集合を要素とする集合の存在

176 :132人目の素数さん:2020/02/19(水) 00:53:13 ID:z1VUWsY5.net
>>174
まず命題φ(f)を
φ(f):=
fは関数で有限集合
∧∀i,j∈ω∀x<i,x>∈f ,j≦i⇒∃y<j,y>∈f
(iが定義域にはいってたらjも入っている)
で定めておく。
すると分出公理から
χ={f∈ω×X | φ(f)}
が存在するけどこれが定義域が有限順序数であるXへの関数全体のなす集合、すなわちXの有限列の集合を与える。

177 :132人目の素数さん:2020/02/19(水) 08:11:16 ID:VGIb2vdY.net
>>176
{ X^n | n∈N }の形を経由せず一挙に有限列全体の集合を作ってますね。ちょっと気に掛かる。

さっき検討しましたが
P(x,y) := x∈N∧y=X^x
と置けば、置換公理により、∃χ∀y(y∈χ⇔∃n∈N P(n,y)) つまり χ={ X^n | n∈N } ですね

178 :132人目の素数さん:2020/02/19(水) 08:16:43 ID:VGIb2vdY.net
>>176
dom(z) := { x | ∃y (x,y)∈z } とすれば
{ f⊆ω×X | fは関数∧dom(f)∈ω } がもっと直接的な解ですね

179 :132人目の素数さん:2020/02/19(水) 08:53:46.23 ID:58L8v3uU.net
>>178
それだとfの全体はωの部分集合からXへの関数全体の集合になるので
数列の全体よりでかい。

180 :132人目の素数さん:2020/02/19(水) 08:57:07.21 ID:byVJxm+Q.net
あ、⊂と∈を見間違えた。>>179は撤回します。

181 :132人目の素数さん:2020/02/19(水) 17:54:38 ID:aOs+m1eu.net
>>168
もっと次元が減っていいぞ

182 :132人目の素数さん:2020/02/19(水) 19:17:48 ID:x9ZYauf9.net
部分空間が分からないというやつのことが俺には分からないよ

183 :132人目の素数さん:2020/02/20(木) 13:17:59.09 ID:LrsCNk81.net
「わからない」と「よくわからない」は違うぞ

184 :132人目の素数さん:2020/02/24(月) 00:09:03 ID:xUkHxSli.net
n次元複素ベクトル空間Vについて
W={dim(Im(f)∩Ker(f)):fはV→Vとなる線形写像}
とWを定めるとき、Wの最大元を求めよ

185 :132人目の素数さん:2020/02/24(月) 00:10:56 ID:xUkHxSli.net
問題文のみで途中送信してしまいましたが
上記の問題の方針など教えて頂ければ幸いです

186 :132人目の素数さん:2020/02/24(月) 20:12:52 ID:xUkHxSli.net
自決しました

187 :132人目の素数さん:2020/02/24(月) 20:20:33 ID:6w80C2eM.net
切腹禁止!

188 :132人目の素数さん:2020/02/24(月) 21:37:52 ID:VDBIyIdy.net
>>184
>Wの最大元
dim(Im(f)∩Ker(f))の最大値のコトね
dimImf+dimKerf=dimV
dim(Im(f)∩Ker(f))≦min(dimImf, dimKerf)≦[dimV/2]
V=U+W dimU=[dimV/2]≦dimW
j:U→U'⊂W
f:V=U+W→W+0⊂V
Imf=U' Kerf=W Imf∩Kerf=U' dim(Imf∩Kerf)=dimU'=dimU=[dimV/2]

189 :132人目の素数さん:2020/02/24(月) 23:14:45.88 ID:WNCybtEs.net
V=U+Wの意味がもうわからんのは俺だけか

190 :132人目の素数さん:2020/02/25(火) 09:02:43.48 ID:1MDaeo1X.net
素朴な疑問

ガンマ関数Γってxが自然数の時、Γ(x+1)=x!なのだが、
なんでΓ(x)=x!となるように定義の調節しなかったんですか?

191 :132人目の素数さん:2020/02/25(火) 09:30:58.93 ID:WMW0bPzH.net
>>189
ならR^nでm=[n/2]として
f(a_1,,,,a_n)=(0,,,0,a_m,,,,a_1)


192 :132人目の素数さん:2020/02/25(火) 12:11:03.47 ID:xlZ4iTwN.net
>>190



>>190
>ガンマ関数Γってxが自然数の時、Γ(x+1)=x!なのだが、
>なんでΓ(x)=x!となるように定義の調節しなかったんですか?

定義の積分の式の綺麗さを優先したのでは?
https://ja.wikipedia.org/wiki/%E3%82%AC%E3%83%B3%E3%83%9E%E9%96%A2%E6%95%B0
ガンマ関数

ガンマ関数(ガンマかんすう、英: Gamma function)とは、階乗の概念を複素数全体に拡張した特殊関数である。互いに同値となるいくつかの定義が存在するが、1729年、数学者レオンハルト・オイラーが無限乗積の形で、最初に導入した[1]。

定義
実部が正となる複素数 z について、次の積分で定義される関数
Γ(z)=∫ 0〜∞ t^(z-1)e^(-t) dt (Re z>0)
をガンマ関数と呼ぶ[2]。

元は階乗の一般化としてオイラーが得たもので、Γ という記号は、1814年にルジャンドルが導入したものである[1]。それ以前は Π(x) などと表記していた(ただし Π(x) = Γ(x + 1))。

193 :132人目の素数さん:2020/02/25(火) 14:53:34.00 ID:1MDaeo1X.net
>>192
積分の定義式の綺麗さを優先するなら、なお一層
Γ(z)=∫ 0〜∞ t^(z-1)e^(-t) dt
ではなく
Γ(z)=∫ 0〜∞ t^z e^(-t) dt
と定義すべきですよね?
後者の場合、zが自然数ならΓ(z)=z!となる。

194 :132人目の素数さん:2020/02/25(火) 15:25:29.52 ID:WMW0bPzH.net
は?
極見たら一発で分かろう

195 :132人目の素数さん:2020/02/25(火) 16:00:29 ID:9/awyf7F.net
極を原点に合わせたかったんだろうなー

196 :132人目の素数さん:2020/02/25(火) 18:00:15.29 ID:1MDaeo1X.net
>>194
もう少し詳しく

197 :132人目の素数さん:2020/02/25(火) 18:02:00.60 ID:1MDaeo1X.net
>>194
というか一々国語力使ってこっちが補って理解するの面倒なんでやっぱ要らない

説明出来る国語力ある人に求む

198 :132人目の素数さん:2020/02/25(火) 19:04:21.55 ID:+J7bndvq.net
ガンマ関数は任意の複素数の階乗を表すねんで

199 :132人目の素数さん:2020/02/25(火) 22:05:52.24 ID:f/hJzzaW.net
>>193
Γ(z)=∫ 0〜∞ t^(z-1)e^(-t) dt は Γ(z)=∫ 0〜∞ t^z e^(-t) t^(-1)dt
と読むんだよ
e^(-t)は加法指標, t^zは乗法指標, t^(-1)dtが乗法不変測度
すなわち,実数体上のガウス和な

200 :132人目の素数さん:2020/02/26(水) 07:04:40 ID:f2cVsmO8.net
>>199
ガウス和はガンマ関数の有限体における類似物である。 (wiki)

ちょっと期待してた回答じゃないっぽいです

201 :132人目の素数さん:2020/02/26(水) 07:08:02 ID:f2cVsmO8.net
不偏分散s^2は
s^2=1/n * Σ[k=1,...,n](x_k - x^\bar)^2
ではなく
s^2=1/(n-1) * Σ[k=1,...,n](x_k - x^\bar)^2
であることは合理的理由がありますが、それと同じような感じで理由を期待してたんですがね。

202 :132人目の素数さん:2020/02/26(水) 16:52:29 ID:C8Rtk1MY.net
一般の 1/(n-k) になる場合を見た方がわかりやすいとか?

203 :132人目の素数さん:2020/03/07(土) 10:34:38 ID:unCXzlIs.net
sin2ydx+sin2xdy=0の積分因子ってどうやって見つければいいですかね

204 :132人目の素数さん:2020/03/07(土) 11:03:31 ID:JUAM4CMV.net
>>203
dx/sin2x+dy/sin2y=0
∫dx/sin2x+dy/sin2y=C
あとは頑張って積分するだけ

205 :132人目の素数さん:2020/03/07(土) 13:52:25 ID:HCTzE85M.net
dx/sin2x=dx/(2sin x cos x)=cos x dx/(2sin x cos^2x)
=d(sin x)/(2sin x(1-sin^2x))

206 :203:2020/03/07(土) 22:34:22 ID:unCXzlIs.net
質問の仕方が悪かったようなのでもういいです

207 :132人目の素数さん:2020/03/07(土) 23:36:06 ID:1Y35bn+f.net
うん、もう来なくて良いよ

208 :132人目の素数さん:2020/03/07(土) 23:45:00 ID:4DKkOPQK.net
この板来るの初めてだからスレ違い・既出だったらすまん
この証明ってあってる?

数学記号の集合を?とする。このとき任意の数式は?*に属する。
また、?は有限集合だから?*の濃度は可算濃度である。
ここで無理数の集合Iの濃度は連続体濃度であるから、?*からIへの全射は存在しない。
よってどんな数式でも表せない無理数が存在する。

「有限集合のクリーネ閉方の濃度は可算濃度」っていうのがちょっと自信ないんだけど

209 :132人目の素数さん:2020/03/07(土) 23:45:44 ID:4DKkOPQK.net
悪いなんかシグマが文字化けしとる

210 :132人目の素数さん:2020/03/07(土) 23:56:16 ID:4DKkOPQK.net
数学記号の集合をSとする。このとき任意の数式はS*に属する。
また、Sは有限集合だからS*の濃度は可算濃度である。
ここで無理数の集合Iの濃度は連続体濃度であるから、S*からIへの全射は存在しない。
よってどんな数式でも表せない無理数が存在する。

何度もすまん

211 :132人目の素数さん:2020/03/07(土) 23:57:37 ID:7VYTGsts.net
S*って何?

212 :132人目の素数さん:2020/03/08(日) 00:06:50 ID:Vqnp04jj.net
lim[->]ΠSi のことかな?

213 :132人目の素数さん:2020/03/08(日) 00:19:30 ID:QWDJsDXG.net
それをxとおく
はい表せた

214 :132人目の素数さん:2020/03/08(日) 00:43:47 ID:3oaG3upS.net
>>209
問題とは関係ないがUnicodeの?(U+2211)は機種依存文字で文字化けする
ギリシャ文字のΣを使えば文字化けしない

215 :132人目の素数さん:2020/03/08(日) 00:50:01 ID:4gmVBNdC.net
>>211
>>212
S*はSのクリーネ閉方。調べたら数学の用語じゃなかったわ。すまん。
>>214
サンクス

216 :132人目の素数さん:2020/03/08(日) 00:50:38 ID:4gmVBNdC.net
閉方じゃない、閉包だ
ずっと間違えてた

217 :132人目の素数さん:2020/03/08(日) 03:31:25 ID:OgoqoyA5.net
>>210
屁理屈じみた結論言ってるけど、
数式から全ての実数への対応付けを一挙に与える事ができないというだけであって、
数学における議論は、常に有限個の数式の使用で収まる(=人間は有限個の記号列しか追えない)訳だから、そのような対応付けは必要としない。
>>213の言うように、今その時点で言及したい無理数が現れる度にそれを(今までの議論(証明)に現れてこない)xで表せば、議論に何の障害も起きない。
当然、より大きい濃度を持つような任意の集合I’に対しても、全く問題が無い。

218 :132人目の素数さん:2020/03/08(日) 03:52:04 ID:qKAqNmL+.net
>>210は単純に

Lが実数論、Mをその標準モデル、TをLの項の全体とするとき、M(L)は常にRの真部分集合であるか?

でないの?
それなら正しいのではないかと。

219 :132人目の素数さん:2020/03/08(日) 08:41:54.73 ID:qWZyRIJf.net
>>217
ありがとう、確かに無意味っちゃ無意味な証明だったなw
ただなんか、「数直線上に確かにあるはずなのに言葉で表せない(?)数がある」っていうのがなんか気持ち悪くてな

220 :132人目の素数さん:2020/03/08(日) 08:43:23.23 ID:qWZyRIJf.net
>>218
聞いたことない言葉が多くてニュアンスしかわかんなかったけど、主張自体は正しかったみたいでよかったわ
レスありがとう

221 :132人目の素数さん:2020/03/08(日) 12:00:59 ID:glDw13Zp.net
>>219
全然気持ち悪くない
つーかZの部分集合の全体とか
Zの数列の全体とか存在感がないかな?
全部を個別に表せないのは当然だと思うけど

222 :132人目の素数さん:2020/03/08(日) 12:19:40 ID:glDw13Zp.net
可算無限だけを特別扱いするとかある場面では有限しか認めないとか
そういうのの方が違和感あるなあ

223 :132人目の素数さん:2020/03/08(日) 13:16:20.87 ID:qWZyRIJf.net
うーん、大学じゃあんまり数学やってないからな
こういうことへの直感って言うか、勘?みたいなのが普段から数学やってる人とは違うのかも

224 :132人目の素数さん:2020/03/08(日) 14:03:32 ID:2Zioev9i.net
定数記号を非可算個用意するって議論はよくありますけどね

225 :132人目の素数さん:2020/03/08(日) 14:29:16 ID:OgoqoyA5.net
>>219
>、「数直線上に確かにあるはずなのに言葉で表せない(?)数がある」
間違ってる。
どんな実数も議論に出てきた時に適切な数式を使って表すことが出来る。
記号列の集合から全ての実数を"一挙に"対応づけさせることができないだけ

226 :132人目の素数さん:2020/03/08(日) 15:48:22 ID:VoZDjnoi.net
>>225
あーなるほど…
そうか、全部に式を当てる必要はないのか…

227 :132人目の素数さん:2020/03/08(日) 15:48:40 ID:/kIkBOsc.net
でも議論に出てきうるのって、文字列使って議論してたら加算無限個じゃないの?

228 :132人目の素数さん:2020/03/08(日) 15:55:00 ID:glDw13Zp.net
x∈R
とするとする場合xは1個と考えるのかってコトよ

229 :132人目の素数さん:2020/03/08(日) 16:21:27 ID:OgoqoyA5.net
>>227
人間が作る・読む証明は常に有限の記号列
でも使える記号列が有限個しか無かった場合、その有限個より1個多く必要とする議論が出来なくなるから無限個の記号は使えなきゃいけない
よって可算無限個の記号列が必要十分な記号の個数

230 :132人目の素数さん:2020/03/08(日) 16:28:14 ID:rJfzPpUl.net
義務教育というテスターで検知された優秀児のうち、支配階層に都合の悪い子供達は、冤罪を着せてでも潰される。コロされる。日本の悲劇。

Y軸がX軸の微分値になっているグラフって、数学界では一般に何と呼ばれるのですか?両逆対数グラフみたいに。

231 :132人目の素数さん:2020/03/08(日) 16:33:49 ID:/kIkBOsc.net
>>228
それは>>225の言う「数式を使って表せる」とは違うんじゃないの?

>>229
その必要十分な記号で可能な議論に出てきうる実数は加算無限個じゃないのかってのが疑問なんだけど
議論に出てきうる実数が加算無限個なら、
>「数直線上に確かにあるはずなのに言葉で表せない(?)数がある」
っていうのが正しいように思うんだけど

232 :132人目の素数さん:2020/03/08(日) 16:48:52 ID:zms4ogdv.net
>>231
その認識であってるよ。
ただし数理論理学てきに少し曖昧なところはある。
まず公理的な実数論の項として出てくる"実数"とその理論のモデルとして出てくる"実数"は切り分けて考えないとダメだし、一つの理論に対してはモデルは一つとは限らないんだから、話しの最初として実数論Lと標準モデルMを持ってこないと。
その上でLの項のモデル上の元とモデルの中に出てくる実数の全体を比較することになる。
オレ専門家じゃないから詳しくは知らないけど標準モデルでないモデルならそのモデルの全部の実数がLの項で書けるモデルも存在するんじゃなかろか?
誰かの定理で必ず可算無限集合上のモデルが存在するってのがあって、その時の証明で項全体の集合からなんかモデルを作るんだったと思うけど、その構成で作ったモデルなら全ての現在がLの項になるモデルも作れる希ガス。
標準モデルでは濃度がちがうからもちろん一致しない。

233 :132人目の素数さん:2020/03/08(日) 16:55:08 ID:glDw13Zp.net
>>231
>それは>>225の言う「数式を使って表せる」とは違うんじゃないの?
違って問題はない
>>227
>でも議論に出てきうるのって、文字列使って議論してたら加算無限個じゃないの?
議論に出てきうるのが
x∈R
で1個と考えるかそうでないかってことよ

234 :132人目の素数さん:2020/03/08(日) 17:19:33.54 ID:/kIkBOsc.net
>>233
問題ないかどうかは人次第だけど
元々の疑問と一連の流れは数式を使って表せるかどうかの話だから問題あるよね

235 :132人目の素数さん:2020/03/08(日) 20:51:49 ID:glDw13Zp.net
>>234
>問題ないかどうかは人次第だけど
元々の問題設定と>>227
「議論に出てきうる」ということが
異なる意味を持っていることは問題ないということ

236 :132人目の素数さん:2020/03/08(日) 21:13:13 ID:/kIkBOsc.net
>>235
>>227
>どんな実数も議論に出てきた時に適切な数式を使って表すことが出来る。
を受けて書いたもので、「数式を使って表せる」というつもりで書いてるし、元々の問題設定と同じ意味のつもりで書いてるよ
誤解させてたらごめんね

237 :132人目の素数さん:2020/03/08(日) 21:26:09 ID:glDw13Zp.net
>>236
つまり
議論に出てきうる
という認識が限定過ぎるのでは?という指摘が>>228の意図
あるいは
x∈R
を1個と考えるというコトも有り得るので
x∈R
を1個と考えるか無数にあると考えるかの認識の違いがあるということの指摘
誤解させてるかも知れないが
x∈R
を1個と考えても別にいいよ
それとRには無数の元がありxとしては無数の者を考えているという認識とは
別だということ

238 :132人目の素数さん:2020/03/08(日) 21:46:58 ID:/kIkBOsc.net
>>237
別なら、元の質問の文脈に沿って
x∈R
を1個と考えるというコト
ということで考えてくれると嬉しい

239 :132人目の素数さん:2020/03/08(日) 22:18:30 ID:87CiEl0i.net
文房具についての問題だ ウキッ!
紙をカミたがる文房具って
なぁんだっ ウキキキッ!!

240 :132人目の素数さん:2020/03/08(日) 22:46:48 ID:glDw13Zp.net
>>238
その場合
P⊂A
も具体的にAに有限の長さの条件P(x)を考えて
P={x∈A|P(x)}
のみしか区別して考えないってコトよね
たぶん数学の一般的な認識より限定的で
それはそれとして有益なこともあるにはあるけど
一般の数学としては窮屈

241 :132人目の素数さん:2020/03/08(日) 23:15:03.47 ID:0dR/DHmx.net
det(a 1 1,1 a 1,1 1 a)=(a+2)*det(1 1 1,1 a 1,1 1 a)
どうしてこうなるのでしょうか

242 :132人目の素数さん:2020/03/08(日) 23:38:28.42 ID:zms4ogdv.net
>>241
2行目と3行目を1行目に足す。

243 :132人目の素数さん:2020/03/09(月) 00:06:31.89 ID:mFksAk09.net
ありがとうございます!

244 :132人目の素数さん:2020/03/12(Thu) 12:21:19 ID:9VNgWc+g.net
前に統語論の完全性について質問したものだけれども次は可証性論理式について質問がある。
なんだかわかったようでわからないことになっている。

このprfの中身をチラ見した感じだとaが公理と推論規則を使った証明という形式になってるかどうかチェックしてるようにみえる。

質問1
prf(a,b)が真 ⇔ 演繹システム├ゲーデル数bとなる論理式
これは成り立っていると考えてよいのだよね?

あと表現定理というのが。

Rが二変数の原始再帰的述語ならばどのようなm,nについても以下が成り立つ二変数の論理式rが存在する。
R(m,n)⇒r(数項m,数項n)の形式的証明は存在する。
¬R(m,n)⇒¬r(数項m,数項n)の形式的証明は存在する。

これは。

言語Lがあるとして、その言語Lを使う演繹システムがあるとして、言語Lで組み立てられた二変数の論理式Rがあるとして。

質問2
表現定理は。
R(m,n)はモデルで真 ⇒ 演繹システム├R(m,n)
¬R(m,n)はモデルで偽 ⇒ 演繹システム├¬R(m,n)

ってことだよね?

また夜に。

245 :132人目の素数さん:2020/03/12(Thu) 12:29:07 ID:9VNgWc+g.net
ミスってたので訂正を。

質問2
表現定理は。
R(m,n)はモデルで真 ⇒ 演繹システム├R(m,n)
R(m,n)はモデルで偽 ⇒ 演繹システム├¬R(m,n)

こう。

246 :132人目の素数さん:2020/03/12(Thu) 18:35:14 ID:TSe1jck0.net
fが線形変換でrank(f*g)=rank(g*f)が成立するとき
gが同型写像であることはどう示せば

247 :132人目の素数さん:2020/03/12(Thu) 18:51:47 ID:fHSLdc4D.net
f=0、g:任意
で成立

248 :132人目の素数さん:2020/03/12(Thu) 18:53:11 ID:oP6dIOya.net
お見事

249 :132人目の素数さん:2020/03/12(Thu) 18:55:32 ID:TSe1jck0.net
すみませんfが零写像でない仮定が抜けていました

250 :132人目の素数さん:2020/03/12(Thu) 19:00:16 ID:fHSLdc4D.net
[[1,0],[0,0]]と[[0,0],[0,1]]

251 :132人目の素数さん:2020/03/12(Thu) 20:49:16 ID:TSe1jck0.net
問題文おもいっきし間違えてましたわ
これの(b)⇒(a)が示せないんです
https://i.imgur.com/4KL7asp.png

252 :132人目の素数さん:2020/03/12(Thu) 21:04:24 ID:fHSLdc4D.net
>>251
fか同型なら
rank(gf)=rank(g)=rank(fg)
fが0でない同型でない写像とする。
C=cok(g)とp:V→Cを自然写像、v∈Vをw=f(v)が0でない元、Wをvの張るVの一次元部分空間、i:W→Vを自然写像、h:C→Wい全射線形写像、g=ihp:V→Vとする。
gf=0。
hpは全射だからv=hp(u)となるuがとれるが、このときfg(u)=w≠0。

253 :132人目の素数さん:2020/03/13(金) 00:59:06 ID:YDfgEvKr.net
IをRの区間とする。a∈Iとする。f:I→Rとする。
この時、∀(x_n)∈I^N [ lim x_n = a ⇒ lim f(x_n) = f(a) ] ならば lim[x→a] f(x) = f(a) が成り立つ。
この証明には選択公理が使われてることはよく知られているけど、選択公理を使わなければ証明出来ないって事は証明されてますか?

254 :132人目の素数さん:2020/03/13(金) 02:09:28.04 ID:IbYZYELm.net
>>253
ググったらこんなのあった。

https://www.researchgate.net/publication/220083198_Some_Weak_Forms_of_the_Axiom_of_Choice_Restricted_to_the_Real_Line

によると

It is shown that AC(ℝ), the axiom of choice for families of non-empty subsets of the real line ℝ, does not imply the statement PW(ℝ), the powerset of ℝ can be well ordered.


AC(ℝ)→ >>253の二つの命題の同値性

しかし

AC(ℝ)からはPW(ℝ) が整列集合である事が示せない、寄ってPW(ℝ)はACより真に弱い。

ので>>253の二つの命題の同値性はACより真に弱い事になる。

255 :132人目の素数さん:2020/03/13(金) 03:32:38 ID:t79DBI+e.net
>>252
何度もすみません
hって具体的にどう対応させてるんですか?

256 :132人目の素数さん:2020/03/13(金) 07:57:38.78 ID:jD3NWBx5.net
>>255
hは何でも良い全射。
Cの基底ciを選んで
h(ci)=v (∀i)
と定めれば良い。

257 :244:2020/03/13(金) 12:44:36 ID:BFcpZF6h.net
ミスがあったのと質問の仕方が大雑把すぎたのかもしれないので書き直しを。
あと質問は一つに絞ることに。
あとスレを荒らす目的はないので、1日たってレスがないようであればこの質問は取り消しで。
またこの手の質問も繰り返さないことに。

Q.表現定理についての質問。

  意味  述語 何かしらの関係を記述するもの。
      命題 真偽が定まるような述語。

  統語  論理式 言語Lを組み合わせて作るある文法に則った記号列。
      閉論理式 自由変数を含まない論理式。

  Rが二変数の原始再帰的述語ならばどのようなm,nについても以下が成り立つ二変数の論理式rが存在する。
  R(m,n)⇒r(S[m],S[n])の形式的証明は存在する。
 ¬R(m,n)⇒¬r(S[m],S[n])の形式的証明は存在する。

 これは、原始再帰的な二変数述語であれば、この述語の自由変数を数項に置き換えたものを使った何かしらの論理式があってそれが証明できる、ということだと自分は思った。
 けれども、書籍の方では更にr(S[m],S[n]は論理式のゲーデル数のことだ」と書かれてて、???、と混乱した。
 このゲーデル数だってのはどう捉えればよいのだろう?

258 :132人目の素数さん:2020/03/14(土) 00:25:39 ID:HHdbhcc5.net
コサインってサインの角度がずれただけなのに、何でサインと同等の立場にいるん?
どっからどう見てもコサインはサインよりも格下だろ。何でこんな格下の分際がサインと同格気取ってんだよって感じ

259 :132人目の素数さん:2020/03/14(土) 00:33:06 ID:HHdbhcc5.net
しかも微分してマイナスが出てくるって言う厄介さもあって、どう考えてもサインより厄介者

260 :132人目の素数さん:2020/03/14(土) 04:25:09 ID:g3DxF95s.net
むしろcosの方が格上なイメージあったわ

261 :132人目の素数さん:2020/03/14(土) 13:23:39.37 ID:54Dthakd.net
対称だしな

262 :132人目の素数さん:2020/03/14(土) 13:26:40 ID:XMvlzpX1.net
>>258
関数空間の基底になるにはsinの他にもcosも必要なんですよ

263 :132人目の素数さん:2020/03/14(土) 17:00:58 ID:TD8dXg+h.net
偶奇性

264 :132人目の素数さん:2020/03/14(土) 23:48:13 ID:Qtllr5m8.net
>>251
V=Imf+W
g:V=Imf+W->Imf->>Kerf⊂V
Imfg=0
Imgf=Kerf≠0

265 :132人目の素数さん:2020/03/14(土) 23:53:12 ID:Qtllr5m8.net
>>264
2rankf≧dimV
>g:V=Imf+W->Imf->>Kerf⊂V
2rankf<dimV
g:V=Imf+W->Imf>->Kerf⊂V
fg=0
rankgf=rankf≠0

266 :132人目の素数さん:2020/03/15(日) 14:14:13 ID:/b+jtV7s.net
>>258
コサインはサインの逆数じゃないのにコタンジェントはなんでタンジェントの逆数なんだ
とかね

267 :132人目の素数さん:2020/03/16(月) 13:42:07 ID:Hsg1lcg1.net
sin/cos だから sin と cos を交換したら逆数になるわな

268 :132人目の素数さん:2020/03/21(土) 21:35:17 ID:8dRr19w1.net
a=1,2,3,...
b=2,3,4,...
を用いてa^bで表される数を累乗数と呼ぶ。1以外の累乗数の集合をXとおく。
X={4,8,9,16,25,27,32,36,...}
(1)Σ[x∈x](1/(x-1))が1になることを示せ。
(2)Σ[x∈x](1/(x+1))を求めよ。

また、これを見て気になったのですが、

(3)Σ[x∈x](1/x)はきれいに求まるか?求まらないなら近似値はどのくらいなのか?

(1)はゴールドバッハ・オイラーの定理という有名な定理だとわかりましたが、wikiの説明は何をやりたいのか見てもわかりませんでした。(3)はWolframで求まらないかと思いましたが、累乗数をどう表現すればいいのか分からず断念。
どうか丁寧に教えて下さい。よろしくお願いします。

269 :132人目の素数さん:2020/03/22(日) 12:52:44 ID:LapwV+OE.net
「素数積で表して指数の最大公約数が2以上」でいいじゃん

270 :132人目の素数さん:2020/03/27(金) 04:41:45.08 ID:sNUQEYjM.net
sin cosin tan
sec cosec cot

正矢、余矢なんてのもあんのね、ご免なさい

271 :132人目の素数さん:2020/03/29(日) 14:12:52 ID:py51p9Qu.net
余矢(よし)よし

272 :132人目の素数さん:2020/03/30(月) 01:11:34.16 ID:1DzJPPin.net
(環上の)加群Mについて、任意の部分加群Nに対してM=N+M/N(=は同型、+は直和)が成り立つようなもののクラスに名前はついてますか?
もちろん、どんな環Rに対しても0加群はこの性質を自明にもちますし、有限次元ベクトル空間に対しても成り立ちます

モチベーションとしては準同型定理M/Ker(f)=Im(f)の両辺に「Ker(f)を掛けて(直積をとって)」M=Ker(f)+Im(f)と変形できるような加群を知りたいです
加群でなくとも、似たような性質をもつ代数系に名前がついているものがありましたら教えて欲しいです

273 :132人目の素数さん:2020/03/30(月) 03:13:41 ID:aUGjpcGt.net
>>272
Mがsemi simpleと同値だと思う。

N:simple
:⇔ ∀L≦N L=0 or N.
M:semi simple
:⇔ ∃Mi a family of simple sub mods of M s.t M = ⊕Mi

Thm TFAE
1) M is semi simple.
2) ∀N≦M ∃L≦M s.t. M=N⊕L.

正しいと思うけどめんどくさそう。

274 :132人目の素数さん:2020/03/30(月) 04:23:03.91 ID:DeMSP4dS.net
>>273の1)を仮定する。
Nを任意の部分加群とする。
M=⊕MiをMiがsimpleであるようにとる。
N⊕L≦MとなるLで極大となるものが取れる。
π:M→M/N⊕Lを自然な射影とする。
π(Mi)が0でないとするとπの制限M→π(Mi)は同型となるが
この核はMi∩(N⊕L)であり、これが0なのでN⊕L⊕MiがMの部分加群に自然に同型となる。
これはLの極大性に矛盾。
∴π(Mi)=0(∀i)。
∴ N⊕L=M。
>>273の2)を仮定する。
容易にこれは次の条件と同値とわかる。

3)∀f:N→M monic ∃g:M→N st. gf is the identity of N

Mの任意の部分加群も3)を満たすので2)を満たす。
NをMのsemi simple部分加群の中で極大とする。
仮定によりM=N⊕LとなるLが取れる。
Lが0でないとし、K≦LをL/Kがsimpleであるようにとる。
Lに2)を適用してJ≦LをL=K⊕Jとなるように取れるが、このときJはL/Kと同型であり、simpleである。
このときN⊕JはMの部分加群でsemi simpleだからNの極大性に反する。

275 :132人目の素数さん:2020/03/31(火) 00:03:05 ID:hEwygeWH.net
>>273-274
ありがとうございます

276 :粋蕎 :2020/03/31(火) 04:02:37.94 ID:EDLtMypi.net
あちこちに貼り回らさせて貰って恐縮だが、激しくガイシュツ問題の魚拓が見付かったんで此ちらにも挙げさせて頂く。
飽く迄も魚拓なんで別途正規に保管して頂きたし。

激しくガイシュツ問題
https://web.archive.org/web/20181107033930/http://www.geocities.co.jp/CollegeLife-Club/7442/math/index.html

277 :132人目の素数さん:2020/03/31(火) 07:21:31.17 ID:4yzaAJjk.net
>>276
ゆかりたんハァハァではないか!
ゆかりたんAAのまとめサイトも貼ってくれ!

278 :132人目の素数さん:2020/04/01(水) 21:35:12 ID:Ms0SdLAT.net
これは佐藤幹夫先生の直筆の字なのでしょうか
http://www.kurims.kyoto-u.ac.jp/~kyodo/kokyuroku/contents/pdf/0388-01.pdf

279 :132人目の素数さん:2020/04/02(木) 05:14:00.64 ID:0TlU4A6G.net
解析接続ってなんのために生まれたんですか??
例えば高校の時、虚数って、おおお√-1があると回転が加法定理になってこんな便利なことができるのか〜!
って感動しました
それで勝手に、これに必要だから虚数を作ったんだな!って思ったんですが
解析接続はなんのために作ったんですか?

自然数の和が-1/12でも、現実には絶対違うし何もできなくない感じがします
でもきっと何かすごいことに役に立ってルと思うんですが!

280 :132人目の素数さん:2020/04/02(Thu) 05:44:30 ID:GGq6XaiY.net
解析接続の学問的勃興の更に昔のギリシャ天下時代…違った、ローマ天下時代に
正式認定史上最古の解析接続の例が有ったんだよね。誰だっけ?歩き目出崇だったかな?

281 :132人目の素数さん:2020/04/02(木) 10:18:36.63 ID:ToV7MfDY.net
>>279
解析的に複素函数に拡張したいからよ

282 :132人目の素数さん:2020/04/02(Thu) 11:40:29 ID:6jUoFBrx.net
>>279
定義域が広がれば便利だから

283 :132人目の素数さん:2020/04/02(Thu) 21:30:26 ID:0TlU4A6G.net
>>281
>>282
ありがとうございます
よくわかんないけど・・・なんかの役に立ってるんでしょうね!

284 :132人目の素数さん:2020/04/02(Thu) 21:32:11 ID:ToV7MfDY.net
>>283
e^zでも見てみたら?

285 :132人目の素数さん:2020/04/02(木) 21:49:54.85 ID:+to/2HC2.net
>>283
数学なんて何の役にも立たないよ

286 :132人目の素数さん:2020/04/02(Thu) 23:43:00 ID:cJVUYR/U.net
>>278
これは面白い原稿だね。
神憑っている。

287 :132人目の素数さん:2020/04/03(金) 00:26:38 ID:KIcZzszY.net
使えない奴に役立つわけがない

288 :132人目の素数さん:2020/04/03(金) 02:58:13 ID:JfHrjAX3.net
フィールズ賞をとったオクンコフが、どっかの対談記事で「私は未だに神保三輪の境地に達していないが」などということを言っていたけど、278の原稿を見ると単なる謙遜でもないのかなと思った。
オクンコフのいう神保三輪の仕事は278時点のものが中心なのか量子群後のことを念頭に置いているのかはわからんけど。

289 :132人目の素数さん:2020/04/03(金) 03:16:15 ID:JfHrjAX3.net
>>278
これは面白い原稿だね。
神憑っている。

290 :132人目の素数さん:2020/04/05(日) 01:28:16 ID:JCHTmcuw.net
https://mathscinet.ams.org/mathscinet/index.html

ここって部外者は使えないんですか?

291 :132人目の素数さん:2020/04/05(日) 11:46:33 ID:Wk6Sgfev.net
>>268
(1)
2以上の自然数の全体を 累乗数と非累乗に分ける。
 X ={4,8,9,16,・・・・}
 M ={2,3,5,6,7,10,・・・・}

n≧2 に対して n=r^k となる最小のrを r(n)と定める。
nはk乗数で底は r である。

m∈M に対して
1/(m-1)= 1/m + 1/m^2 + 1/m^3 + ・・・・
  = Σ[r(n)=m]1/n
  = 1/m + Σ[r(x)=m,x∈X]1/x,
よって
Σ[x≦N,x∈X]1/(x-1)
 = Σ[n=1〜N-1]1/n - Σ[m=2〜N,m∈M]1/(m-1)
 = H(N-1)- Σ[m=2〜N,m∈M](1/m + 1/m^2 + 1/m^3 + ・・・・)
 = H(N-1)- Σ[m=2〜N,m∈M](1/m + Σ[r(x)=m,x∈X]1/x)
 = H(N-1)- Σ[m=2〜N,m∈M]1/m - Σ[2≦r(x)≦N,x∈X]1/x
 = H(N-1)- H(N)+ 1 - Σ[2≦r(x)≦N<x,x∈X]1/x
 = 1 - 1/N - Σ[2≦r(x)≦N<x,x∈X]1/x
 → 1,   (N→∞)

*) lim[N→∞]Σ[x≦N,x∈X]1/x = s から
0 < Σ[2≦r(x)≦N<x,x∈X]1/x
 ≦ Σ[x>N,x∈X]1/x
 = s - Σ[x≦N,x∈X]→ 0,

292 :132人目の素数さん:2020/04/05(日) 13:13:17 ID:Wk6Sgfev.net
>>268
(2) log(2π)- π/3 = 0.79068
(3) 7/8 = 0.875

293 :132人目の素数さん:2020/04/07(火) 03:06:52 ID:ZlV3F5Vq.net
M: semi simple
 Mの固有ヴェクトル系が空間Vの基底をなす。
 ⇔ Mは対角化可能

294 :132人目の素数さん:2020/04/11(土) 21:51:27 ID:det5eRYO.net
数直線の有界部分集合A, Bに対して
|sup A - sup B|≦ sup |A - B|
って成り立ちますか? 成り立つなら、その証明を教えて下さい。
|A - B|は、{|a - b|; a ∈A, b ∈B}の意味です。

295 :132人目の素数さん:2020/04/11(土) 22:09:45 ID:y0KMNXEN.net
a<=|a-b|+b<=sup|A-B|+supB

296 :132人目の素数さん:2020/04/13(月) 14:14:39 ID:gmrn0Cgm.net
なるほど

297 :132人目の素数さん:2020/04/14(火) 07:12:52.58 ID:VmAtvvOk.net
3次関数はいわゆる「畳8枚の性質」(ある予備校の講師がこう言ってた)を持ってるけど、
4次以上の関数って類似の性質ありますか?

298 :132人目の素数さん:2020/04/14(火) 08:15:11.73 ID:y0PmH4ZA.net
そんなローカルな名前を言われてもどういう性質なのか全くわからん

299 :132人目の素数さん:2020/04/14(火) 14:30:02 ID:Qyt7VTcl.net
3次関数の性質なんて点対称と共通接線がどうとかしか知らんな

300 :132人目の素数さん:2020/04/14(火) 16:04:35.84 ID:GTlL0LZl.net
>>297
4次は一般に線対称ですらないからそんな綺麗な性質は無いだろうな

301 :132人目の素数さん:2020/04/18(土) 14:25:22 ID:M465pxpe.net
回転行列からオイラー角は一意に求められないけど
クオータニオンからオイラー角は一意に導くことが可能ですか?

302 :132人目の素数さん:2020/04/18(土) 15:25:41 ID:ma1vnuaf.net
εN論法について。|a_n-α|<αεとなったらa_nはαに収束すると言えますか?

303 :132人目の素数さん:2020/04/18(土) 15:31:52 ID:dTRSAMFq.net
もちろんαは固定されてるんでしょ
εを改めてε/αと取り直せばよろし

304 :132人目の素数さん:2020/04/18(土) 15:35:15 ID:ma1vnuaf.net
>>303
ありがとうございます

305 :132人目の素数さん:2020/04/18(土) 15:49:37 ID:VNcmPxi8.net
εを動かしてどうすんだバーカ
田島一郎死ね

306 :132人目の素数さん:2020/04/18(土) 15:50:23 ID:VNcmPxi8.net
任意の元は任意だから任意に動かせるwwwwwwwwwwwwwwwwwwwwwwwwwwwwwwwwwwwwwwwwwwwwwwwwwwwwwwwwwwwwwwwwwwwwwwwwwwwwwwwwwwwwwwwwwwww

307 :132人目の素数さん:2020/04/18(土) 15:50:52 ID:VNcmPxi8.net
横田一郎も同じ

308 :132人目の素数さん:2020/04/18(土) 15:51:56 ID:ma1vnuaf.net
これあってますか?新入生で予習してるんですけど、誰にも聞けないので。lim[n→∞]a_n=0ならば、lim[n→∞]Σ[k=1〜n]a_k/n=0を証明せよ。
https://i.imgur.com/VLOGoEf.jpg

309 :132人目の素数さん:2020/04/18(土) 16:07:27 ID:N+UPuMwp.net
|Σ[k=1,...,n]a_k/n - α| <= (Σ[k=1,...,n]|a_k - α| )/n < (Σ[k=1,...,n]ε )/n = ε

310 :132人目の素数さん:2020/04/18(土) 16:43:48 ID:cXu1ZGgp.net
数列{a_n} の初めのn項の相加平均として定まる数列{b_n}をチェザロ列と呼ぶ。
チェザロ列{b_n}の総和は 正則、・{a_n}について線型 かつ {a_n}の総和と無矛盾である。

311 :132人目の素数さん:2020/04/18(土) 17:25:50 ID:cXu1ZGgp.net
>>299
3次関数の標準形 (?)
x^3 + ax^2 + bx + c = X^3 + BX + C,
 B = b - aa/3,
 C = c - ab/3 + (2/27)a^3,
 X = x + a/3,

312 :132人目の素数さん:2020/04/19(日) 20:40:05 ID:PWktvEbT.net
>>301
> 回転行列からオイラー角は一意に求められないけど
どゆこと?求め方ぐぐるとでてくるよ たとえば
https://www.learnopencv.com/rotation-matrix-to-euler-angles/

313 :132人目の素数さん:2020/04/20(月) 16:29:49 ID:YSMPdYrN.net
オイラー角が一意じゃない状況があるのさ

314 :132人目の素数さん:2020/04/20(月) 19:40:29 ID:ygaItMbC.net
>>313
一意に求まらないの意味がそういう意味なら
>>301 の2行目の質問、意味なくなるじゃん

315 :132人目の素数さん:2020/04/21(火) 12:23:49 ID:lEwX0jJX.net
>>206
質問しておいてその態度はないだろ

316 :132人目の素数さん:2020/04/21(火) 13:23:39 ID:7VCyjd01.net
東大卒がコロナの件でこんな数式?出してきたんだけど合ってるの?

https://twitter.com/ryuichiyoneyama/status/1252404518530117633?s=21
(deleted an unsolicited ad)

317 :132人目の素数さん:2020/04/21(火) 13:25:33 ID:r1p1DmHw.net
あれ?

https://twitter.com/ryuichiyoneyama/status/1252404518530117633?s=21
(deleted an unsolicited ad)

318 :132人目の素数さん:2020/04/21(火) 14:33:24 ID:lxkip3W5.net
>>314
「不可能」と言う答があるぞ

319 :粋蕎 ◆C2UdlLHDRI :2020/04/21(火) 19:16:47 ID:gIPp5okj.net
>>277
ゆかりたんハァハァが何か知らんで過ごして来た人生の半分は損しとる儂に作品名くらい教えんと
AA集を探すにしろどうにも成らんぞ
2Dアイドルも3Dアイドルも知らん儂に『どうしろという。(by KingOfUniverse)』じゃ。

320 :132人目の素数さん:2020/04/21(火) 19:42:24 ID:yc3F3j7i.net
>>319
昔の話で記憶が怪しいが、高校数学の複素数の質問を単発スレを立てて質問した奴の名前がゆかり。
その糞スレに誰かがAAを作って、そのAAをいろいろ改造して貼られて、後は忘れた

321 :132人目の素数さん:2020/04/21(火) 19:44:18 ID:yc3F3j7i.net
>>319
検索したら一つ見つかった
http://science2.5ch.net/test/read.cgi/math/1058774018/l50

1 :ゆかり:03/07/21 16:53
      ___ ___ 
    , ´::;;;::::::;;;:ヽ   口の聞き方(数式の書き方)に気をつけな!
    i!::::::::::::;ハ;::::::ヽ 
    |:::::::ivv' 'vvvリ       ・指数 例: 2^5=32 (2の5乗 イコール 32)
   |:::(i:| ┬ イ |::|   人   ・対数 例: log_[3](9)=2 (ログ3底9 イコール 2)
   .|::::l:|.  ヮ ノi:|   n て  ・積分 例: ∫[1≦x≦3] (e^x) dx = e^3 - e
   |:::::|:l〈\/i:::|:|,  /E)    ・分数 例: (a+b)/(c+d)  (分子a+b、分母c+d)
   !/^リ;;;;;;;个;;;;リ;;∨::/ 
複素数テンプレ集 http://www.geocities.co.jp/CollegeLife-Club/7442/math/yukari.html
詳しい記号の書き方 http://members.tripod.co.jp/mathmathmath/

322 :粋蕎 ◆C2UdlLHDRI :2020/04/21(火) 21:31:02 ID:gIPp5okj.net
事故解決か。壺創始から覗き始めて不思議でない歳の儂が壺覗き始めたのが04じゃけ、その前年か

323 :132人目の素数さん:2020/04/22(水) 00:26:45 ID:zkDD5tDY.net
>>313
z軸まわりのφ回転の場合、β=0, α+γ=φ
とか・・・・

岩澤分解したらどうなる?

324 :132人目の素数さん:2020/04/28(火) 05:19:37.81 ID:B5ahXJh2.net
そのまんま東って、本名は東だと思うんだが あのハゲは学歴詐称だったろ? 政治犯だと思います

wikiによると、早稲田の入試試験を受けて合格して再婚して、通学して卒業後、離婚してその次、
又 早稲田大学の他の学科受験して入学出来てストレートに卒業出来ただと! そんな人、居る!?
何で通学時にニュースにならなかったの? 萩本欽一だとニュースになっていたのに???
絶対に嘘だと思うよね?単位を落とさなかったんで?編入学じゃなく、再入試を受けたってさ?

ビートたけしのテレビタックルで政治にも悪影響が及んでるのかと思うとゾっとします

325 :132人目の素数さん:2020/04/28(火) 10:11:53 ID:BBvMe2cs.net
>>324
君、何歳?

326 :132人目の素数さん:2020/04/28(火) 11:27:58 ID:yBKE5vIS.net
古い話だなー、自分の歳を感じるよ

327 :132人目の素数さん:2020/04/28(火) 15:52:00 ID:Ncn08YEb.net
これといった日向の神でもなし

328 :132人目の素数さん:2020/04/28(火) 15:52:45 ID:Ncn08YEb.net
とらぬ狸のもとのムジカ

329 :132人目の素数さん:2020/04/28(火) 20:40:07 ID:6BadWRUI.net
     ___ ___ 
   , ´::;;;::::::;;;:ヽ 
  i!::::::::::::;ハ;::::::ヽ 
  |:::::::ivv' 'vvvリ    
  |:::(i:| ┬ イ |::|   人  荒らしは失せな! 
  .|::::l:|.  ヮ ノi:|   n て
  |:::::|:l〈\/i:::|:|,  /E)
  !/^リ;;;;;;;个;;;;リ;;∨::/ 

330 :粋蕎 ◆C2UdlLHDRI :2020/04/28(火) 22:34:04 ID:BBvMe2cs.net
此れがかの有名なゆかりの姐御か

331 :132人目の素数さん:2020/04/29(水) 12:45:42 ID:U1zbPkei.net
>>327-328
うーむ分からん

332 :132人目の素数さん:2020/04/29(水) 14:30:07.18 ID:tsdk3gHc.net
検定問題の解答ですが、
最後の行の2.038は2.216の間違いですよね。
------------------------------------
N=10
平均 x~=99.5
不偏分散 u&sup2;=0.509
(1)
母平均の信頼度 95 %の信頼区間は、
t(0.05)=2.2622なので、

x~-u*t(0.05)/√N < μ < x~+u*t(0.05)/√N

を計算して、
99.03 < μ < 100.05

(2)有意水準0.1での両側検定では、
t(0.1)= 1.8331
なので、
t=|x~-100|*√(N)/u=2.038>1.8331

333 :132人目の素数さん:2020/04/29(水) 14:37:43.21 ID:9wCaOkjG.net
すみません、下記、教えてください。

xyz空間内に、原点Oを中心とした半径1の球Sがある。
点A(1、0、0)、点B(0、1、0)、点C(0、0、1)
を通る、平面Lによって球Sを分割する。
小さいほうの立体をDとする。
(1)分割されてできる断面の中で、z座標が最小となる点を求めよ。
(2)立体Dをz軸に周りに回転させてできる立体Dの通過する範囲の体積を求めよ。

334 :132人目の素数さん:2020/04/29(水) 17:10:20 ID:lSjpaToA.net
https://ja.wikipedia.org/?curid=12761
このページで
従って有限回のステップでは有限個の n に対してしか P(n) を結論づける事ができず、「無限個ある自然数全てに対して P(n) が成り立つ」という数学的帰納法の結論について有限の長さの証明が与えられたとはいえない。
ってかいてあるけど無限個のnなんてあるの?
例えば
11111111111111.........永遠に続く
22222222222222.........永遠に続く
なんて自然数あるってこと?

335 :132人目の素数さん:2020/04/29(水) 18:37:37 ID:WR6LpORw.net
>>334
ある
ない

336 :132人目の素数さん:2020/04/30(木) 06:23:21.12 ID:gk1moOfa.net
自然数nは無限個ある。
111…や222…は111…111.111…や222…222.222…故に自然数ではなく無限小数である。
一方、…111や…222は実数の中の自然数ではなく無限大超実数の中の無限大超自然数である。詳しくは
 超整数 - Wikipedia
https://ja.wikipedia.org/wiki/%E8%B6%85%E6%95%B4%E6%95%B0
を読め。

無限大超実数∋無限大超無理数+無限大超有理数∋無限大超整数∋無限大超自然数
超実数-非実数超実数=実数
超有理数-非有理数超有理数=有理数
超整数-非整数超整数=整数
超自然数-非自然数超自然数=自然数

これは、にわか覚えすると弊害しか生じないので、普通は誰も教えず本人の成長に任せる。
つまり簡単に答えだけを教える行為は、むしろ虐待である。
故に本当は>>335の冷たい回答こそが優しく、俺の回答は甘やかし虐待である。苦しめ。

337 :132人目の素数さん:2020/04/30(Thu) 11:53:12 ID:hxeTxTeP.net
>>333
(1)
 S: xx+yy+zz ≦ 1,
 L: x+y+z = 1,
より
 0 ≦ 2(xx+yy)-(x+y)^2
 ≦ 2(1-zz)-(1-z)^2
 = 1 +2z -3zz
 =(1-z)(1+3z),

∴ -1/3 ≦ z ≦ 1,
 最小となる点は(2/3, 2/3, -1/3)
(2)
球Sのうち z≧-1/3 の部分の体積は(80/81)π

平面Lをz軸の周りに回転させると
 xx + yy ≧ (1/2)(1-z)^2  ・・・・ 円錐
底面を z=-1/3 とすると底半径は (2√2)/3、高さ 4/3
円錐の体積は(32/81)π
したがって Dが動く範囲の体積は
 V =(80/81)π -(32/81)π =(16/27)π.

338 :132人目の素数さん:2020/04/30(Thu) 11:59:29 ID:hxeTxTeP.net
>>328
とらぬ狸と同じ穴にいるムジナ
ぢゃね?正しくは、

339 :132人目の素数さん:2020/05/01(金) 12:01:54 ID:p/uMvsb3.net
理想的なビリヤードを考える。
地に摩擦はなく、弾同士の反発係数は1つまりエネルギーの損失がないとする。

この時、最初の弾を任意の方向に打った時、有限の時間で全ての弾はポケットに入るか?
また、Yesであるならば、そうなる平均時間はいくらか?

340 :132人目の素数さん:2020/05/01(金) 18:13:42 ID:L6BRmC25.net
当たらん軌道があるからダメだろ

341 :132人目の素数さん:2020/05/01(金) 20:08:08 ID:8ge44aj8.net
>>339

> 理想的なビリヤードを考える。
> 地に摩擦はなく、弾同士の反発係数は1つまりエネルギーの損失がないとする。
>
> この時、最初の弾を任意の方向に打った時、有限の時間で全ての弾はポケットに入るか?
> また、Yesであるならば、そうなる平均時間はいくらか?
答えてる人がいるけど、noでしょう。
普通の形のビリヤード台だったら。
ジェネリックな方向にはイエスかもしれないが。

そして形とかの情報無しに平均時間とか聞いている出題者のセンスが疑われますね。

342 :132人目の素数さん:2020/05/02(土) 04:17:32.26 ID:hIQBs+5G.net
「ビリヤード問題」とかいって球の軌道方向(壁面からの角度θのときのtanθ)が有理数なら軌道が周期的になるから
穴に落ちない場合がでてくるけど、無理数なら非周期的になるから軌道は盤上を埋め尽くし、いずれ穴に落ちる
で、有理数より無理数の濃度は大きいから無理数になると考えられほぼ常にいずれ穴に落ちる
みたいな話が物理であったと思う
有限時間で落ちるかどうかはちょっとわからんけど

343 :132人目の素数さん:2020/05/02(土) 05:32:04 ID:+036Icv/.net
それにしても平均時間出すためにはビリヤード台の縦横の長さとポケットの大きさがないと出ない気はするな。

344 :132人目の素数さん:2020/05/02(土) 06:35:30 ID:E062Nxsw.net
あとスタート位置

345 :132人目の素数さん:2020/05/02(土) 07:07:10 ID:hIQBs+5G.net
簡単のため、直径dの球一つを速さvで盤の一隅から打ち、
盤は一辺1の正方形として、直径hの穴一つを球と逆の一隅に、とする
それでも、幾らでも穴に落ちる時間がかかる経路がありそう

球は壁にあたって反射するのではなく、壁を対称軸とした鏡面の向こう側に行くとすると
二次元面上の原点(0, 0)から球を打って、(2n+1, 2m+1)にある穴をねらうことになる
ちょっと日本語おかしいけどごめん

(2n+1, 2m+1)にある穴までの距離は√((2n+1)^2+(2m+1)^2)だから、到達時間はこれをvで割ったもの
この時間をどういう重みで平均取ればいいかだが…力尽きた

346 :132人目の素数さん:2020/05/02(土) 09:23:06 ID:0NcbQVI7.net
ラプラス変換の厳密な取り扱いって何勉強すればわかりますか?

347 :132人目の素数さん:2020/05/02(土) 09:33:50.76 ID:WJJOVbEa.net
反射する問題を鏡面の向こう側に行くって言う捉え方は良い発想だな
この発想に基づけば、一次関数が格子点と持つっていう考えに行くな

348 :132人目の素数さん:2020/05/02(土) 10:46:24.58 ID:hIQBs+5G.net
格子点ということならいっそのこと、四隅に穴があるとしたほうが、(n, m)の格子点になって問題としては自然なのか
ちょっとおもしろい

349 :132人目の素数さん:2020/05/02(土) 11:01:55 ID:tzT0D4Fv.net
>>348
面白いと言っても、じゃあ解けるのかといわれたら無理。
自然だけど現代数学の到達してる範囲
では解けない問題なんかアホほどあるよ。

問題
r>0とする。
(1/2,1/2)を通る直線の空間をp1とし、各p1の元lに対しa∈lである格子点bでd(a,b)≦rを満たす者がとれる点のうち(1/2,1/2)に最も近い点をa(l)とし、f(l)=d((1/2,1/2),a(l))とおく。
∫[l∈p1] f(l) dl を求めよ。

こんなの解けないよ。

350 :132人目の素数さん:2020/05/02(土) 11:03:45 ID:VhQnw0Y8.net
>>349
> (1/2,1/2)を通る直線の空間をp1とし

脳味噌うんこかよ?

351 :132人目の素数さん:2020/05/02(土) 11:43:50 ID:5tBfgPGE.net
バカばっか

352 :132人目の素数さん:2020/05/02(土) 14:45:26 ID:/cZAw4zP.net
>>346
微積分

353 :132人目の素数さん:2020/05/04(月) 00:54:49.88 ID:GE/wlG7e.net
代わりはいません
まちがひありませぬ!!!
xxxxxhttps://i.imgur.com/rsd6pAd.gif/hahahahaha

354 :132人目の素数さん:2020/05/04(月) 02:37:30 ID:jDRWX2Ph.net
3月の宿題で(1)のみ正解の数弱@shukudai_sujaku

昨年度の大学への数学(大数)での勝率は、

学コンBコースが 1/1 = 100% ,

宿題が 3/10 = 30% でした!

宿題の勝率が低すぎると思うので、

これからは一層精進していきたいです!

https://twitter.com/shukudai_sujaku
(deleted an unsolicited ad)

355 :132人目の素数さん:2020/05/06(水) 16:01:44 ID:KaOskql5.net
宣伝て馬鹿にしか見えんな

356 :132人目の素数さん:2020/05/06(水) 16:04:19 ID:voTvvKrw.net
スレ番が飛んでいるが、何か宣伝しているのか?

357 :132人目の素数さん:2020/05/06(水) 20:45:23.69 ID:T9TuxvKX.net
本質的にK上の体L1,L2がある時L1⊗_K L2がいつ体になるかで決まるってやってた記憶があるな。
LiがK上分離的ならよかった気もする。
永田先生の可換体論の3章にあったな確か。

358 :132人目の素数さん:2020/05/07(Thu) 11:27:06 ID:92UtUlkK.net
日本語なんとかしろ

359 :132人目の素数さん:2020/05/09(土) 19:48:42 ID:xctXILow.net
線形代数の固有ベクトルを求めるのに高次でなく
2次、3次のものはヤコビ法は使えないんですか?

360 :132人目の素数さん:2020/05/09(土) 22:16:04 ID:qFonhWVT.net
>>359
↓コレの事?

https://ja.m.wikipedia.org/wiki/%E3%83%A4%E3%82%B3%E3%83%93%E6%B3%95_(%E5%9B%BA%E6%9C%89%E5%80%A4%E5%95%8F%E9%A1%8C)

使えるんでね?

361 :132人目の素数さん:2020/05/10(日) 00:33:55 ID:Cn8Fdpki.net
アプリのプログラミングをしていて
123456
の6個の数字の距離を評価したいのですが、
6と1が連続している条件にしたいです。
つまり
6と1、6と5の距離はともに1
6と2、6と4の距離はともに2
といった具合です。
こういった数学の分野って何かなかったでしたっけ?

362 :132人目の素数さん:2020/05/10(日) 05:08:19.19 ID:pMQSbfO8.net
グラフ理論
重み付きグラフってやつ

363 :132人目の素数さん:2020/05/10(日) 05:25:36.86 ID:pMQSbfO8.net
てかアプリのプログラミングしてるならアルゴリズムの本なんかに載ってるかも
最短経路問題とかそんな感じで

364 :132人目の素数さん:2020/05/17(日) 13:49:03 ID:C3088KVg.net
n次元球B(0,r)上のポアソン核
P(x,y)=(r^2-|x|^2)/(nβ(n)r|x-y|^n), x∈B(0,r),y∈∂B(0,r),β(n):n次元単位球の体積
について、
∫[∂B(0,r)]P(x,y)dSy=1
を示すにはどうすればいいですかね?上手いこと変形出来ると思うのですがサッパリです

365 :132人目の素数さん:2020/05/19(火) 14:01:13 ID:rGvSl6e8.net
発散級数を有限値にする等式が解析接続を用いて説明されたりしますが
収束半径の外でのテイラー展開の値が無意味ではないですか
素粒子云々じゃなくて何かよい直感的解釈は無いですか

366 :132人目の素数さん:2020/05/19(火) 14:09:35 ID:6J5B37r3.net
>>365
解析接続による説明の残念な点は
値が複数個(無限個を含む)になる
場合があることです
((1+z)^(1/n)とかln(1+z)とか)

https://ja.wikipedia.org/wiki/%E3%83%A2%E3%83%8E%E3%83%89%E3%83%AD%E3%83%9F%E3%83%BC

まあ、面白いといえば面白いですけどね

367 :132人目の素数さん:2020/05/19(火) 14:25:45 ID:HnyGTIPw.net
>>365
多分「くりこみ」の話だと思うけど、それだとよくわからないから数学的な解釈が欲しいってこと?
所謂ゼータ函数正規化ってやつがくりこみに対応するんじゃないかな

他の数学的な定式化なら、例えばアーベル総和法とか、チェザロ総和法とか、タウバー型定理とかは知られているね

368 :132人目の素数さん:2020/05/19(火) 15:06:44 ID:6J5B37r3.net
ま、ゼータ関数の場合は、>>366のような残念なことにはならないけどね

369 :132人目の素数さん:2020/05/20(水) 16:06:40 ID:624MI5KL.net
>>365
「テイラー展開の値が無意味」は「テイラー展開が無意味」を意味しない
「テイラー展開の表現」自体は解析関数の情報を全部含んでいるから
解析接続も意味がある

370 :132人目の素数さん:2020/05/21(Thu) 00:05:06 ID:sZ7Wz5TO.net
>>365
>収束半径の外でのテイラー展開の値が無意味ではないですか
収束半径内だけど??

371 :132人目の素数さん:2020/05/21(Thu) 09:33:55 ID:52hIZUpT.net
多項式の計算をC言語でやりたいのですが、
多項式を扱う標準的なライブラリはありませんでしょうか。
(pythonのnumpy.poly1d() のような)

372 :132人目の素数さん:2020/05/22(金) 00:15:06 ID:es5tEsC8.net
極大イデアルがただ一つの(可換)環は局所環としてよく知られてますが、素イデアルがただ一つである環に名前はついてますか?
このような環をprime-local ring(PLR)と呼ぶことにすれば、次のことはすぐにわかります
・任意の体はPLR
・任意のPLRは局所環
・体でないPLRは整域でもない
・Z/p^nZ(有限体じゃないよ)はPLR
・(R,p),(S,q)をPLR、f:R→Sを環準同型とすればp=f^{-1}(q)、つまりf(p)⊂q(であるから局所環の間の局所準同型に相当する「PL準同型」は単なる環準同型に他ならない)
・上において、特にSが体であればp=Ker(f)(PLRから体への準同型について核はすべて等しい)

正直こんなものは誰しも考えつくもので、それが本に書かれてないというのはあまり意味のない概念だからだと思いますが、
非整域の例がもっとあれば割と(非整域の環論において)意味を持ちそうだとも思えますし、上のZ/p^nZの例からは素数冪の性質として数論への応用も多少はありそうに思えます
まあ非整域がそもそも面白くないとか「素数冪なら有限体を考えればよくね?」とか言われればそれまでですが

373 :132人目の素数さん:2020/05/22(金) 23:02:53 ID:y+ggBWMl.net
>>350
「味噌」(みそ)

用例
・・・・ すなわち R_(2n-2)の符号は(-1)^(n-1)に等しい。
さて(15)においてnにn+1を代用すれば
 R_(2n-2)= ・・・・ + R_(2n),    (16)
上に述べたように、R_(2n-2)とR_(2n)とは反対の符号を有するから
 R_(2n-2)= ・・・・・θ,  0<θ<1    (17)
これを(15)へ代入すれば(9)を得る。
 (16)から(17)を導くところが味噌である。
a=b+c において、aとcとが反対の符号を有するならば、0<a/b<1.

高木貞治:「解析概論」改訂第三版,岩波書店 (1961)
 第5章 §69 Stirlingの公式 p.262

バンドルとかうんこみたいな概念
http://rio2016.5ch.net/test/read.cgi/math/1487257038/139

374 :132人目の素数さん:2020/05/22(金) 23:46:38 ID:es5tEsC8.net
>>372
自己解決、準素環と同値かこれ

375 :132人目の素数さん:2020/05/23(土) 08:37:42 ID:+EfVzueJ.net
有限群ってどれぐらい特定されてるんですか?

376 :132人目の素数さん:2020/05/23(土) 13:02:09 ID:ZlSliJ3p.net
ttps://ja.wikipedia.org/wiki/有限単純群の分類

377 :132人目の素数さん:2020/05/23(土) 14:38:46.51 ID:xEXUhoKI.net
モンスターが有名だよな

378 :132人目の素数さん:2020/05/23(土) 14:42:55 ID:nttT/xBg.net
同型による位数2000以下の群の分類のプロジェクトが行われて位数ごとの個数の表がどっかに転がっていた
ちな位数2000以下の群のうち99%以上が位数1024の群

379 :132人目の素数さん:2020/05/23(土) 15:00:16 ID:nttT/xBg.net
位数10の半群の個数は位数2000以下の群の総数よりも多い

380 :132人目の素数さん:2020/05/23(土) 15:12:59 ID:XjWfJOJm.net
位数が可算無限の群は同型をのぞいて、その個数は無限かどうか

381 :132人目の素数さん:2020/05/23(土) 15:25:38.04 ID:0hJyw13e.net
Z*(Z/pZ)

382 :132人目の素数さん:2020/05/23(土) 17:31:11 ID:UQuKBuXn.net
>>377
>モンスターが有名
有名な割に実体知ってる人がほとんど居ない

383 :132人目の素数さん:2020/05/23(土) 21:30:20 ID:+EfVzueJ.net
群の基数から構造ってどれぐらい特定出来るんかな?

384 :132人目の素数さん:2020/05/23(土) 21:45:07 ID:ZlSliJ3p.net
基数っていっても有限群なら上の分類定理の話になるけど、
可算無限とか連続無限とかだとほとんど情報ないってことになるかな

385 :132人目の素数さん:2020/05/23(土) 22:35:39 ID:+EfVzueJ.net
というか、任意の無限基数?_αに対して、群が存在して|G|=?_αって成り立つ?

386 :132人目の素数さん:2020/05/23(土) 22:45:53.89 ID:+EfVzueJ.net
文字化け
アレフ_α

387 :132人目の素数さん:2020/05/23(土) 23:22:41.28 ID:ZlSliJ3p.net
どうなんだろ、知識はないので以下はヨタだけど

自己同型群ってのがあるから幾らでも群の位数(基数)が大きいのは作れそうだけど
単位元は恒等写像だから常に存在するとしても
どこかで結合法則が成り立たなくなるとかあるのかな

388 :132人目の素数さん:2020/05/24(日) 00:09:06 ID:mnbV8PSW.net
>>385
成り立つよ
Xの濃度が無限の時、Xの有限部分集合全体Pfin(X)の濃度はXと一致
A,B∈Pfin(X)についてA△B=A∪B\A△Bと定義すると
これは群になってる
単位元は空集合、Aの逆元はA自身
結合法則示すのはちょっと面倒だができる

389 :132人目の素数さん:2020/05/24(日) 00:09:26 ID:1QTHtJ1C.net
空集合でない任意の濃度の集合に群構造が入れられることと選択公理が同値

390 :132人目の素数さん:2020/05/24(日) 00:10:25 ID:mnbV8PSW.net
>>388
A△B=A∪B\A∩Bの間違いです。。

391 :132人目の素数さん:2020/05/24(日) 00:18:46 ID:BRaju6wI.net
>>388
サンクス

392 :132人目の素数さん:2020/05/24(日) 01:25:07 ID:7yrUqYW3.net
>>389
>>388の|Pfin(X)|=|X|にCがいるってことだろ

393 :132人目の素数さん:2020/05/24(日) 01:41:34.59 ID:vNtUsEnf.net
yes

394 :132人目の素数さん:2020/05/24(日) 02:03:21.31 ID:7yrUqYW3.net
あと無限集合X上の自由群の濃度もXと同じじゃない?

395 :132人目の素数さん:2020/05/24(日) 09:31:18 ID:7yrUqYW3.net
そうかPfin(X)はX上にZ/2で生成された可換群か
似たものがいくらでも作れる中で最小か
対称群S(X)は大きくなるけど
有限部分集合以外ではIdになる置換全体
Sfin(X)=∪{S(F)|F∈Pfin(X)}もXと濃度同じだろ

396 :132人目の素数さん:2020/05/24(日) 14:21:07.86 ID:2WLuGGnn.net
>>382
俺も知らねえや

397 :132人目の素数さん:2020/05/25(月) 01:54:16 ID:QBOZj+Pp.net
どなたかお力添えお願いします

次のftで定義される関数をフーリエ級数展開せよ.f(t)=(A/T)t (0<t<T)
f(t+T)=f(t)

次のftで定義される関数のフーリエ係数を導出せよ.
f(t)=1+4t/T (-T/2<t<=0)
f(t)=1-4t/T(0<t<T/2)

398 :132人目の素数さん:2020/05/25(月) 12:38:50 ID:as7r/XH1.net
(上)
 {t/T}= t/T -[t/T]
  〜 1/2 - Σ[k=1,∞]sin(2kπt/T)/(kπ),

(下)
 1 - 4|t|/T 〜 8Σ[L=奇数]cos(2Lπt/T)/(Lπ)^2

 〜 8Σ[k=1,∞]cos(2(2k-1)πt/T)/((2k-1)π)^2,

            (-T/2 <t< T/2)

399 :132人目の素数さん:2020/05/26(火) 04:30:08.90 ID:dPUzFCUy.net
複素数の超越数ってありますか?

400 :132人目の素数さん:2020/05/26(火) 04:48:10.92 ID:assh5D2W.net
其りゃ当然在るじゃろ
π*√i

401 :132人目の素数さん:2020/05/26(火) 09:21:53 ID:dPUzFCUy.net
>>400
実部も虚分も0じゃない複素数ではありますかね?

402 :132人目の素数さん:2020/05/26(火) 11:18:14.41 ID:moFWvn2F.net
√i = (1+i)/√2,
実部も虚部も0ではない。

403 :132人目の素数さん:2020/05/26(火) 11:35:27 ID:i2RxsrIG.net
超越数のwikipediaに載ってる定理でも見てみたらいい
簡単に作れる

404 :132人目の素数さん:2020/05/26(火) 12:19:05.24 ID:gRPzOECd.net
実部も虚部も超越数ではないのに全体として超越数、ということを言いたいのか?

405 :132人目の素数さん:2020/05/26(火) 12:39:03 ID:assh5D2W.net
>>402
惜しい、其れは複素代数的無理数で複素超越数ではない。
だが其れを借りて回答を完成させて頂く。


>>401
π*√i
=π/√2+π*i/√2 直交座標表示
=π*cos(π/4)+π*i*sin(π/4) オイラーの公式準拠表示
=π*cis(π/4) cis(x)はオックスフォード流cos(x)+i*sin(x)の略記
=π*exp(π*i/4) 複素指数表示
=π∠π/4=π*arg(π/4) 極座標表示

406 :132人目の素数さん:2020/05/26(火) 15:33:30 ID:WIRA0b0n.net
家族全員数字に弱すぎて理解できないので
どなたか教えてください

3月にアマゾンで7千円分の買い物をしましたがキャンセルしました
4月10日の支払い分から7千円引かれましたが
5月10日分で7千円を含んだ金額を請求されていました

これで正しいのでしょうか?

407 :132人目の素数さん:2020/05/26(火) 15:57:19.95 ID:kzTX9DSZ.net
>>398
ありがとうございます

408 :132人目の素数さん:2020/05/26(火) 16:30:14.62 ID:gRPzOECd.net
>>406
アマゾンに問い合わせたら?

409 :132人目の素数さん:2020/05/26(火) 16:54:19 ID:a06+1VIl.net
L…1B1EE3(max)…5(count)
N…103E847F(max)…3(count)
A…767B27621(max)…6(count)
B…X(max)…Y(count)

Rules
L+N=A
B⊂L

Hints
There is a shortcut.
This hint is \0 today.
Today is November 18. 2019.

X or Y?

410 :402:2020/05/26(火) 18:19:59 ID:moFWvn2F.net
>>404
 いいえ。>>401 へのレス

>>405
 は >>401 と同じもの

411 :132人目の素数さん:2020/05/27(水) 11:43:21.77 ID:I+PlhbAl.net
無限変数多項式環を厳密に丁寧に定義しようとすると結構しんどいな

412 :132人目の素数さん:2020/05/27(水) 14:07:17.63 ID:t80rJokb.net
変数集合で可換群を生成して
それを基底とする線形空間か

413 :132人目の素数さん:2020/05/27(水) 17:12:54.65 ID:VeAW514w.net
1変数でも、そもそも不定元を具体的に定義(構成)するのが面倒だった記憶
藤崎のガロア理論に載ってたはず

414 :132人目の素数さん:2020/05/27(水) 17:35:35.35 ID:bvljpjW2.net
環の圏での普遍性による定義があったような気がする

415 :132人目の素数さん:2020/05/27(水) 18:08:46 ID:iohOKfKE.net
>>411
なんで?有限のcolimじゃん

416 :132人目の素数さん:2020/05/28(Thu) 22:00:07 ID:an7VtCf2.net
インスタライブってどうやったら見れるんですか?

417 :132人目の素数さん:2020/05/28(Thu) 22:00:34 ID:an7VtCf2.net
すいません誤爆した

418 :132人目の素数さん:2020/05/28(木) 23:07:55.34 ID:vbBmFCb2.net
>>415
何の有限colimit?

419 :132人目の素数さん:2020/05/29(金) 01:06:59 ID:hj7GKLxH.net
感覚的には明らかもしくは難なく当然だと理解出来るような主張だけれども、
いざ証明しようとなると極めてしんどくて厄介だったり、高度な定理を用いなきゃ証明出来ないような証明って何がありますか?

ジョルダンの閉曲線定理は知ってます。

420 :132人目の素数さん:2020/05/29(金) 01:29:52 ID:GjRiU3Gi.net
ホモトピー球面=球面
とか?

421 :132人目の素数さん:2020/05/29(金) 07:40:05.22 ID:+fnZACzA.net
射影空間のハウスドルフ性

422 :132人目の素数さん:2020/05/29(金) 14:20:46.54 ID:GjRiU3Gi.net
どの定義?

423 :132人目の素数さん:2020/05/29(金) 16:56:55.09 ID:lkwM1NzW.net
>>418
は?
有限生成のcolim

424 :132人目の素数さん:2020/05/29(金) 17:31:33.53 ID:lkwM1NzW.net
>>420
>ホモトピー球面=球面
=とは?
波平の頭は球面じゃないぞ

425 :132人目の素数さん:2020/05/29(金) 17:34:29.34 ID:lkwM1NzW.net
>>421
なんで面倒くさいの?斉次座標系で一発だろ
どれを使わねばならないかで場合分けが面倒??
大した面倒でもないだろ

426 :132人目の素数さん:2020/05/29(金) 18:20:49.31 ID:wIBEwxNo.net
最大値最小値の定理

427 :132人目の素数さん:2020/05/29(金) 19:00:15.66 ID:wHrSs1tJ.net
>>423
いやだから何の帰納系のcolimなんだよっていう

428 :132人目の素数さん:2020/05/29(金) 21:58:27.69 ID:lkwM1NzW.net
>>427
はぁ
変数の有限集合の包含に決まってオロ

429 :132人目の素数さん:2020/05/29(金) 22:14:30.73 ID:edcEsYVb.net
>>428
変数の有限集合に代数的構造定まるの?

430 :132人目の素数さん:2020/05/29(金) 22:22:22.67 ID:lkwM1NzW.net
>>429
君バカだねw

431 :132人目の素数さん:2020/05/29(金) 22:26:03.18 ID:lkwM1NzW.net
>>413
>1変数でも、そもそも不定元を具体的に定義(構成)するのが面倒だった記憶
そんな記憶は棄てるんだね

432 :132人目の素数さん:2020/05/29(金) 22:27:11.15 ID:Dngc9gZq.net
>>428
お前それを「有限のcolim」や「有限生成のcolim」と略したのかw
有限colimit(帰納系の添字集合が有限集合の場合のcolimt)のことかと勘違いしたわ

433 :132人目の素数さん:2020/05/29(金) 22:27:36.05 ID:Dngc9gZq.net
変数の有限集合たちが包含でなす帰納系は添字集合が無限集合なので有限変数多項式環の有限colimtではなく無限colimtを考えることになる

434 :132人目の素数さん:2020/05/29(金) 22:29:37 ID:lkwM1NzW.net
>>432
>帰納系の添字集合が有限集合の場合のcolimt
アホカね

435 :132人目の素数さん:2020/05/29(金) 22:30:21 ID:lkwM1NzW.net
>>433
で結局それできれいに定義できてお仕舞い
なんの苦労もなし

436 :132人目の素数さん:2020/05/29(金) 22:30:44 ID:4nduiMe9.net
>>430
A_1={x_1},A_2={x_1,x_2},…として
包含関係による埋め込みを定める前に、埋め込みが準同型となるような代数的構造がA_iに入ってないと帰納系が定まらなくないか?

437 :132人目の素数さん:2020/05/29(金) 22:32:05 ID:lkwM1NzW.net
>>411
>無限変数多項式環
これに対応して有限って言ってんだよ
あとはやっと分かったらしいID:Dngc9gZqの言うとおり

438 :132人目の素数さん:2020/05/29(金) 22:32:48 ID:lkwM1NzW.net
>>436
要らない
何アホなこと書いてんだコイツ

439 :132人目の素数さん:2020/05/29(金) 22:33:37 ID:4nduiMe9.net
>>438
要らないとは
帰納系の定義が代数的構造を要求してるように見えるのだが

440 :132人目の素数さん:2020/05/29(金) 22:33:50 ID:lkwM1NzW.net
>>436
多項式環考えてるんじゃないのかw

441 :132人目の素数さん:2020/05/29(金) 22:34:48 ID:lkwM1NzW.net
>>439
あのね
変数集合の包含にそもそも代数構造要らない
その上で考える多項式環の包含に有ればいいだけ
ってことも書かないと分からないのか

442 :132人目の素数さん:2020/05/29(金) 22:35:51 ID:lkwM1NzW.net
>>439
>帰納系の定義が代数的構造を要求してる
最終的に入らないけれど
今考えているのは環の圏なんだろ?

443 :132人目の素数さん:2020/05/29(金) 22:36:54 ID:4nduiMe9.net
>>441
包含そのものにはそらいらないと思うが、帰納系にいるんだが

> {Ai | i ∈ I} を I で添字付けられた対象の族、fij: Ai → Aj (i ≤ j) を準同型の族として
https://ja.m.wikipedia.org/wiki/帰納極限

準同型を定義できるような代数的構造がA_iに入ってる必要がある

444 :132人目の素数さん:2020/05/29(金) 22:38:13 ID:lkwM1NzW.net
>>443
>包含そのものにはそらいらないと思うが、帰納系にいるんだが
だからそれは当たり前だろ?
環の圏考えてるんじゃないのか
それを前提で>>436のようなアホなこと書くのかね

445 :132人目の素数さん:2020/05/29(金) 22:39:36 ID:lkwM1NzW.net
>>443
お前の書いてるA_iの定義は>>436じゃないのか
参ったネそりゃw

446 :132人目の素数さん:2020/05/29(金) 22:39:37 ID:4nduiMe9.net
>>444
で、A_2={x_1,x_2}に入ってる代数的構造って何だ?

447 :132人目の素数さん:2020/05/29(金) 22:40:16 ID:lkwM1NzW.net
>>446
お前ホントに圏論勉強してないのか
はぁ

448 :132人目の素数さん:2020/05/29(金) 22:40:17 ID:Dngc9gZq.net
>>434
有限のcolimtと言われたら普通そう理解してしまうだろ

>>439
代数構造と言っていいのか分からんが
無限個の有限集合が包含でなす圏から無限個の有限変数多項式環の圏への関手を考えていることになる

449 :132人目の素数さん:2020/05/29(金) 22:41:12 ID:lkwM1NzW.net
>>448
>有限のcolimtと言われたら普通そう理解してしまうだろ
勝手に理解せいや
>>437が答えだ

450 :132人目の素数さん:2020/05/29(金) 22:42:32 ID:4nduiMe9.net
>>447
勉強しててもしてなくても帰納系の定義が準同型(ひいては代数的構造)を要求してるんだが

451 :132人目の素数さん:2020/05/29(金) 22:42:56 ID:lkwM1NzW.net
>>448
>代数構造と言っていいのか分からんが
環の圏だから環構造前提だろ
帰納系を変数の有限集合の包含で定義するだけ

452 :132人目の素数さん:2020/05/29(金) 22:44:06 ID:lkwM1NzW.net
>>450
>勉強しててもしてなくても帰納系の定義が準同型(ひいては代数的構造)を要求してるんだが
だからそれは多項式環考えてるんじゃないのか?
それ前提の話なんだがw
その多項式環の包含は変数の有限集合の包含から定まる帰納系になるってだけ
ここまで書かないと分からないか

453 :132人目の素数さん:2020/05/29(金) 22:46:51 ID:4nduiMe9.net
>>452
例えばA_1={x_1}でさえ、多項式環の積を持ってくるとx_1 * x_1がはみ出すので環にならないんだが
どんな代数的構造が入ってるんだ?

454 :132人目の素数さん:2020/05/29(金) 22:50:06 ID:lkwM1NzW.net
>>453
もうバカに付ける薬はないな
「お前の書いたA_i」のcolimはただの変数の無限集合だよ
そもそもcolimに代数構造は要らないが
今の話はすべて環の圏での話だ

455 :132人目の素数さん:2020/05/29(金) 22:51:41 ID:4nduiMe9.net
>>454
いやだから代数的構造が入らないからcolimが定義できないから変数の無限集合になることさえないんだが

俺のA_iが異なるなら、具体的に変数の有限集合とは何だ?

456 :132人目の素数さん:2020/05/29(金) 22:53:20 ID:lkwM1NzW.net
>>455
まあいいから圏論勉強してね

457 :132人目の素数さん:2020/05/29(金) 22:55:17 ID:lkwM1NzW.net
たぶん ID:4nduiMe9 は加群のcolimしか知らない

458 :132人目の素数さん:2020/05/29(金) 23:00:27 ID:4nduiMe9.net
>>457
帰納系が定まるなら加群に限らない
定義に書いてある

帰納系 〈Ai, fij〉 の帰納極限 A の台集合は、Ai の直和集合の適当な同値関係 ∼ による商集合として与えられる。
https://ja.m.wikipedia.org/wiki/帰納極限

つまり帰納系が定まりさえすれば群でも環でも加群でも良い
ただ変数の有限集合とやらが群や環になるか分からないので、帰納系に必要な準同型がなく、帰納系が定まらず、そして帰納極限が求められない

459 :132人目の素数さん:2020/05/29(金) 23:06:56.05 ID:Dngc9gZq.net
多項式環の係数環をRとする
R係数の有限変数多項式環(これをR代数と見なす)たちが包含写像に関してなす帰納系の無限colimtがR係数の無限変数多項式になるとも言えるので
R代数の

460 :132人目の素数さん:2020/05/29(金) 23:08:40.03 ID:Dngc9gZq.net
>>459
途中で送信してしまった
最後「R代数(特にR加群)の圏におけるcolimtとも思える」

461 :132人目の素数さん:2020/05/29(金) 23:17:10.62 ID:4nduiMe9.net
>>459
これなら(合ってるかどうかは自分には分からないけど)分かる

462 :132人目の素数さん:2020/05/29(金) 23:26:53.81 ID:Dngc9gZq.net
>>461
よかった
物事を正確に書いてくれないと誤解が生じちゃうよね

463 :132人目の素数さん:2020/05/29(金) 23:29:51.57 ID:4nduiMe9.net
>>462
本当それ

464 :132人目の素数さん:2020/05/29(金) 23:39:34 ID:HuOKgGGR.net
ID:lkwM1NzWは(個人的に)どの圏Cで考えてるかによって自動的にCにおける射が決まるとでも思ってるのかな

>>431
そら普段はそんな構成とか気にする必要ないしそれを利用して何か新しいことが分かるわけでもないし
そんなこと覚えてても蘊蓄にしかならんよ

465 :132人目の素数さん:2020/05/29(金) 23:47:13.79 ID:R6EGMWTw.net
でも知らないと気になるわ

466 :132人目の素数さん:2020/05/30(土) 01:33:33.26 ID:LP50efes.net
Rings を可換環の圏論、Sets を集合の圏、U:Rings → Sets を忘却関手とするとき関手F:X→ ℤ[X]はUの左随伴関手

467 :132人目の素数さん:2020/05/30(土) 02:21:59.35 ID:MUuBmQ2f.net
>>458
>帰納系に必要な準同型がなく
射が有ればいいのよ

468 :132人目の素数さん:2020/05/30(土) 02:23:44.85 ID:MUuBmQ2f.net
>>464
>ID:lkwM1NzWは(個人的に)どの圏Cで考えてるかによって自動的にCにおける射が決まるとでも思ってるのかな
全然?
けれど変数の集合の包含から多項式環の包含をどう定めるかはほぼ自明

469 :132人目の素数さん:2020/05/30(土) 02:25:43.33 ID:MUuBmQ2f.net
しっかしアホだらけのスレだな

470 :132人目の素数さん:2020/05/30(土) 02:28:16.60 ID:MUuBmQ2f.net
>>466
>Rings を可換環の圏論
多項式環だからR-Modのがいいかな

471 :132人目の素数さん:2020/05/30(土) 02:30:04.27 ID:MUuBmQ2f.net
>>455
>代数的構造が入らないからcolimが定義できない
この誤解を解くためにも圏論勉強してな

472 :132人目の素数さん:2020/05/30(土) 02:33:20 ID:LlGyfuWv.net
ここ、変数集合の集合としての包含から多項式環の環としての包含が誘導されることすら
分からんような奴が混じって議論してるの?
ヤバない???

473 :132人目の素数さん:2020/05/30(土) 06:10:14 ID:y1oT4TaI.net
>>467
wikipediaには代数的構造が定まっていない場合については書いていないが、それを信用して
{x_1}→{x_1,x_2}→…
という"帰納系"について求めてみると、同型は自分自身だけで、恐らく直和は{x_1,x_2,…}だから帰納極限が求められることになるな
>>454の言うとおりではあったな、サンクス

>>428を見ると、上の通り「変数の有限集合の包含」を帰納系とするcolimは{x_1,x_2,…}ということになる

474 :132人目の素数さん:2020/05/30(土) 06:13:41 ID:y1oT4TaI.net
同型→同値

475 :132人目の素数さん:2020/05/30(土) 13:38:59 ID:ckOSIoWd.net
>>470
R-mod だと普通はR加群の圏になる。

476 :132人目の素数さん:2020/05/30(土) 13:47:46.46 ID:J/PX2uhp.net
>>466

> Rings を可換環の圏論、Sets を集合の圏、U:Rings → Sets を忘却関手とするとき関手F:X→ ℤ[X]はUの左随伴関手

unut ε_X : X → U(F(X))はε_X(x) = xで定められる写像。
counit δ_R : F(U(R))= ℤ[R]→Rはδ_R(r) = rで定められる準同型写像。

477 :132人目の素数さん:2020/05/30(土) 14:40:19.77 ID:MUuBmQ2f.net
>>475
じゃあR-Alg

478 :132人目の素数さん:2020/05/30(土) 14:41:08.16 ID:MUuBmQ2f.net
>>473
>wikipediaには代数的構造が定まっていない場合については書いていないが、それを信用して
信用するなよ
勉強せいや

479 :132人目の素数さん:2020/05/30(土) 15:08:36.11 ID:9WsomZcN.net
>>478
どっちにしても>>428は誤りか

480 :132人目の素数さん:2020/05/30(土) 15:12:41.77 ID:LlGyfuWv.net
つーか、ウィキペディアにも圏における直系の直極限とか一般の定義のとことかに
書いてある内容よめば圏の射という以上の意味で「準同型」に拘る意味ないのはわかるし
なんなら代数系の帰納極限てとこでも環や加群に限らない代数系って言ってるから
「演算が何も備わってない代数系とその準同型」=「ただの集合と写像」
の場合でもできるってのはちゃんとわかるんだよなあ

ウィキペディアは不親切な記述しかないのかもしれないが教科書でも受験参考書でもないし
「ふーん、だから何」以上の内容があると考えてはいけない
自分からいろいろ調べる前提で資料探しの入り口として使うもんだ

481 :132人目の素数さん:2020/05/30(土) 16:52:35.26 ID:B53VPd6a.net
無限変数て可算個?

482 :132人目の素数さん:2020/05/30(土) 17:14:46.38 ID:Il8onmDq.net
>>481
colimitの取り方に依る
特定の加算無限集合の有限部分集合たちが包含でなす圏から有限変数多項式環への関手による帰納系の無限colimitを取れば加算無限変数多項式環が得られるが
特定の非加算無限集合の有限部分集合たちが包含でなす圏から有限変数多項式環への関手による帰納系の無限colimitを取れば非加算無限変数多項式環が得られる

483 :132人目の素数さん:2020/05/30(土) 17:16:49.54 ID:Il8onmDq.net
>>482
×有限変数多項式環への関手
〇有限変数多項式環の圏への関手

484 :132人目の素数さん:2020/05/30(土) 17:29:09.51 ID:MUuBmQ2f.net
>>481
別に濃度関係ない

485 :132人目の素数さん:2020/05/30(土) 17:41:43.53 ID:9WsomZcN.net
>>480
それが分かるのは結論ありきじゃないか

>>428といい相手がエスパーじゃないと分からん

486 :132人目の素数さん:2020/05/30(土) 17:48:30.19 ID:Azf9OBK/.net
論理的に説明するとかいう学問的な話はできないけどマウント取りたいという猿しかこのスレにはいないぞ
そういう猿がそれっぽいことを言うのがこの板だ
普通の知能してるやつがこんな板にいるわけない

487 :132人目の素数さん:2020/05/30(土) 17:49:42.36 ID:LlGyfuWv.net
>>485
「それ」と「結論」を具体的にしてくれ、意味が分からん
「代数系とその準同型」に「ただの集合と写像」が「演算が何もない場合」として含まれるのは
ふつうに代数の本にも出てくるような一般論だから、そういう内容の話ではないよね?

488 :132人目の素数さん:2020/05/30(土) 18:12:38.12 ID:9WsomZcN.net
>>487
いやwikipediaの帰納極限のページを読めば分かるという話だったのに、急に普通の代数の本に書いてあるようなこととか言われてもな
質問への回答はマウント取りじゃなくて相手の実力を図ってかないと

489 :132人目の素数さん:2020/05/30(土) 20:33:52 ID:LlGyfuWv.net
読めばわかるという話だったの?
ウィキペディアに書いてないって言ってるけどウィキペディアに不備があるわけじゃない(実質的には書いてあるじゃん)って話だろ?

おれはウィキペディアは不親切な記述しかないって立場だし
一般論で当たり前とされることは当然当たり前として読んたうえで
この件でウィキペディアの落ち度とするのは筋違いで失礼だと言ってるだけ

490 :132人目の素数さん:2020/05/30(土) 20:46:04.44 ID:bW8oN+mP.net
>>489
>つーか、ウィキペディアにも圏における直系の直極限とか一般の定義のとことかに書いてある内容読めば
……わかるし
なんなら
……ちゃんとわかるんだよなあ


読めば分かると自分で言ってるのだが
代数的構造に関しては質問だったが、流石にスレ違いになってきたからこれでレスやめるけども

491 :132人目の素数さん:2020/05/30(土) 20:54:35.53 ID:LlGyfuWv.net
>>490
そのページの圏論のとこ読めば分かるように書いてあるからわかるよ
ずっとそのページの代数系のとこの話だったろ

「なんなら」以降のはは補足として
 一般論として自明な事実を知ってれば代数系のところでも「抜け落ちてないことがわかるよ」
って言ってんだよなあ

マスゴミの切り貼りレベルでたちが悪いなお前

492 :132人目の素数さん:2020/05/30(土) 21:50:57 ID:+E5IqOKq.net
この定理5.4の途中に出てくるMは何故必要なんですか?
個人的には必要ない気がするのですが、教授曰く場合分けを省略するためにMを持ち出しているそうです。
その場合どのような場合分けを省略しているのでしょうか?
どなたか理由を教えてもらえませんか?
https://imgur.com/0PcJfdz.jpg
https://imgur.com/E7fTeLP.jpg

493 :132人目の素数さん:2020/05/30(土) 22:17:25.63 ID:XMH3pFCF.net
え、大学でこんなのやってるの?
教育学部?

494 :132人目の素数さん:2020/05/30(土) 22:18:53.56 ID:w0g7AzS8.net
学部で初等幾何か

495 :132人目の素数さん:2020/05/30(土) 22:50:44.67 ID:qBctBD7N.net
まあええやんけ

>>492
点Eは弧BC上でも、点Mは線分BCではなく直線BC上に来るかもしれない。つまり円の外側に来る場合もある。
1) BとCの間にMがある場合、2) MとCの間にBがある場合、3) BとMの間にCがある場合。

で、質問の答えは背理法(帰謬法)だから
Dがγの外部にあるなら → 1) または 2) または 3) の場合 → 矛盾、ってしたいわけだけど
三つの場合をわざわざ考えなくても1)という特別な場合だけ考えても矛盾が導けるから、ってこと
これが矛盾を導くのでなければ、三つの場合すべて考えなけりゃならん

496 :132人目の素数さん:2020/05/30(土) 22:52:43 ID:+E5IqOKq.net
>>493
教育学部です
円周角の逆だってことは分かるのですが、何故Mは出てきたんですか?

497 :132人目の素数さん:2020/05/30(土) 22:55:10 ID:+E5IqOKq.net
>>495
なるほど
点Mの場所で3通りの場合が合ったんですね
確かによく考えればその通りでした。。。
ありがとうございます!

498 :132人目の素数さん:2020/05/30(土) 23:03:28 ID:qBctBD7N.net
>>497
たとえば二枚目jpegのように角BCDが直角または鈍角の場合、3)の場合は無かったりするから、
こういうのを考えたくなくて1)の場合だけ考えてるんでしょうね

499 :132人目の素数さん:2020/05/30(土) 23:14:53.12 ID:pcI5Gy7c.net
>>494
大学でアルファベットや九九を教える底辺大学もあるからな。
中学の算数なら、かなりマシな方だろう…。

500 :132人目の素数さん:2020/05/30(土) 23:19:14.96 ID:VcrwYQA4.net
教員養成系で中高数学固め直すのは当たり前だろうにくだらんマウント取りたがる猿

501 :132人目の素数さん:2020/05/30(土) 23:27:44.19 ID:9reRjSvp.net
初等幾何の初等を勘違いしているサル
サルは高等という名があれば満足なのだろう
さあ山に登れ
専門はなんだ?
サル

502 :132人目の素数さん:2020/05/30(土) 23:34:47.78 ID:k45KIWTa.net
学問やってるやつの大半は興味ではなくマウントのためにやってるからな
猿山の猿になりたがるヒトの域にすら達してない低知能よ
数学やってるから頭いいなんてのは大きな誤解

503 :132人目の素数さん:2020/05/30(土) 23:39:31.77 ID:r5+8j3An.net
殺される一歩寸前までいかなきゃ分からねぇ畏れ知らずばかりだな
一回、首から下を潮が引いたばかりの海浜に埋められてみるかよ?

504 :132人目の素数さん:2020/05/30(土) 23:43:46.65 ID:wt71bB8t.net
まぁ数学界隈に我々は頭がいいなんて勘違いした愚か者が多いのはその通りだな

505 :132人目の素数さん:2020/05/30(土) 23:50:21.98 ID:+E5IqOKq.net
>>495です
自分の質問のせいでスレ荒れ始めちゃってすいません
専門は生物なので簡単な数学の質問についてはどうかお許しを。。。

もう一つ質問があるのですが、次の絵の場合点Mなしで∠BAC>∠BDCになると思うのですが何故これではだめなのですか?
https://imgur.com/undefined.jpg

506 :132人目の素数さん:2020/05/30(土) 23:52:04.13 ID:+E5IqOKq.net
>>505
すいません 自分492でした

507 :132人目の素数さん:2020/05/30(土) 23:53:46.87 ID:qBctBD7N.net
>>505
絵が違ってますよ

508 :132人目の素数さん:2020/05/30(土) 23:56:55.20 ID:+E5IqOKq.net
>>507
ありがとうございます
>>505
URL間違ってました
https://imgur.com/YhNJ8li.jpg

509 :132人目の素数さん:2020/05/31(日) 00:24:59.17 ID:iflqRtgW.net
>>505
駄目なことはないと思うけど、直線BDが弧BACと交わると限らない、ということで場合分けが必要になるんだと思う
弦BC上の点Mなら、弧BACと交わることが保証できるんだと思うが、それは以前のページで証明されてるのかな?

510 :132人目の素数さん:2020/05/31(日) 00:28:28.58 ID:BJeR+3Kt.net
MがBCの外側に来ることなんかあるの?

511 :132人目の素数さん:2020/05/31(日) 00:52:11.17 ID:iflqRtgW.net
点Dから弧BACに交点Eができるように点Mを考えたんだけど要らん気がしてきた
では場合分けを省略するためにってのが分からなくなってくるが

512 :132人目の素数さん:2020/05/31(日) 01:13:24.64 ID:768BhTNc.net
マウント取るも取られるも別に普通のことだからいいじゃん
否定も非難もする必要も無い
分かってるか否かだけ

513 :132人目の素数さん:2020/05/31(日) 01:14:47.01 ID:768BhTNc.net
むしろマウント取られたくなくて
分かってないのに理解する努力もしないのが数学的には最低だろ

514 :132人目の素数さん:2020/05/31(日) 01:15:57.18 ID:Vxk9yqfT.net
そこが数学の良い所。
どう言い繕うが、結果は隠せない。

515 :132人目の素数さん:2020/05/31(日) 01:40:26.39 ID:flj1fCWN.net
結果が分かる迄は怪しいけどな
望月新一RIMS一党にしろショルツ一党にしろ、どう落とし前付ける気なんだか

516 :132人目の素数さん:2020/05/31(日) 02:59:32.30 ID:Ri2qEuIC.net
>>500
まあでも教育学の一環としてという観点だと
内容は知ってて解くのは何でもないるがそれをどう教えるか→そのためにさらに深い理解を
という流れでならいいが
そもそも解けなくて完全に中学の復習から解けるように何とか持って行くとこから
みたいなのをここで見せられると、講師の嘆き声が聞こえてきそうだろ?

517 :132人目の素数さん:2020/05/31(日) 09:18:13.28 ID:768BhTNc.net
>>508
Eが無い可能性があるから
CDと円弧の交点もないかも知れない
>>509
円の内点Mと外点Dを結ぶ線分DMは必ず円と1点で交わり
Mが線分BC上の点だから
その点はBCのD側の円弧上かつ△BCDの内点ということは自明としてるんじゃないかな
>>511
>>509
の書いた
>直線BDが弧BACと交わると限らない
ということで場合分けが必要になる
BDもCDも弧BACと交わらないときはAが△BCDの内点となることを使うかな
結局M使うのが場合分けなくて証明はシンプルだけど思いつくものかな

518 :132人目の素数さん:2020/05/31(日) 16:23:06.42 ID:dxqrssce.net
>>509>>517
BDとの交点だけでなくCDとの交点の場合があり、それを省略するためにMを用いたということでよろしいですか?
お答えいただきありがとうございます!

519 :132人目の素数さん:2020/05/31(日) 16:42:36.93 ID:iflqRtgW.net
>>518
重箱の隅をつつくような感じだけど、>>517の人がいってるように、仮に円のBでの接線と、Cでの接線が交わる位置にDがあったとしたら、
直線DAでも直線DCでも、弧BAC上には交点Eができなくなる
ネットで高校の証明いくつか見てみたけど、徹底的に場合分けをする証明や、>>508の図のような特別な場合だけ考えて
実は穴がある証明なんかがあった
BCの内点のMの場合はそういう特別なことを考えなくていいので実にシンプルでいいと思う

ついつい描いた図に引きずられて特別な場合だけ考えてしまいがちなので注意しないといけない

520 :132人目の素数さん:2020/05/31(日) 16:47:52.52 ID:iflqRtgW.net
>>519
ごめん訂正
> 直線DAでも
直線DBでも

521 :132人目の素数さん:2020/05/31(日) 23:14:43 ID:LICLE/8y.net
>>397
(下)
[例2]f(x)=|x| (偶函数)
高木:「解析概論」改訂第三版、岩波書店 (1961)
 第6章、§77. Fourier級数の例 p.282

上下逆さだけど・・・・

522 :132人目の素数さん:2020/05/31(日) 23:16:55 ID:mdSV5zyq.net
>>516
マウント猿の鳴き声が聞こえる

523 :132人目の素数さん:2020/06/01(月) 00:26:52.56 ID:2A9Cuc38.net
劣等感は下火なのか

524 :132人目の素数さん:2020/06/01(月) 09:19:57.71 ID:Vp+Yn4h+.net
△BCDの外接円を考えて
直線BAとのもう1つの交点Eを使って
∠BDC=∠BEC<∠BAC
でどうかな
Aが外接円の内点だってことは自明とは言えない?

525 :132人目の素数さん:2020/06/01(月) 09:45:35.70 ID:Vp+Yn4h+.net
>>524
>Aが外接円の内点だってことは自明とは言えない?
2円が2点で交わっているとき
4つの円弧と交点を結ぶ線分は
((|))みたいな位置関係だってことは自明として良いんじゃ無いかな
それならAは△BCDの外接円の内点としていいから
BA延ばした先にEがある

526 :132人目の素数さん:2020/06/01(月) 13:24:07.79 ID:Vp+Yn4h+.net
>>510
模範解答の点の取り方とは違うが
先に円弧上にEを取って考えたいのが>>495だろ
DE延ばした先で直線BCとの交点をMとするなら
Mは外に来ることがある

527 :132人目の素数さん:2020/06/02(火) 00:13:55 ID:EuotxuXg.net
複素関数でz=x+iyとして∂/∂z=(∂/∂x-i∂/∂y)/2で定義する、とした場合に、
例えばzが現れないz*のみの関数に∂/∂zを作用させると0になる、ということは
定義に従って実際にそうなることを示す以外に、何かイメージ的な理解はできますか?

528 :132人目の素数さん:2020/06/02(火) 01:02:56.50 ID:NhHP3q63.net
イメージっていうなら二次元の流体力学的なはなしとかどう?

二次元の定常非圧縮渦なしの完全流体が複素速度ポテンシャルで記述される
ttps://ja.wikipedia.org/wiki/複素速度ポテンシャル

複素速度ポテンシャルW (z)を正則関数として
∂W/∂z = u_x - i u_y
u_xが流体のx方向の速度、u_yがy方向の速度
こいつに∂/∂z*を作用させるとゼロになるけど、これの意味するところは
速度場のdivが実部、速度場のrotが虚部に対応している

符号とか間違ってるかもしれん、詳しくはググって

529 :132人目の素数さん:2020/06/02(火) 01:34:22.69 ID:TPydHgX/.net
「zが現れないz*のみの関数」
これの意味が問題だと思うけど・・・
z* が決まれば鏡像 z も決まってしまう。
おそらく z* の多項式、有理式、ローラン級数など
(つまりz*の正則函数)を考えているのかな?
鏡像関係を反故にしてzとz*を独立変数と見なすなら
∂/∂z は z*を止めておくから、当然0だけど。

530 :132人目の素数さん:2020/06/02(火) 13:04:28 ID:lu0YtqDw.net
イメージ的なんて有り得んだろ

531 :132人目の素数さん:2020/06/02(火) 15:00:38 ID:prq3z1Wv.net
複素微分をイメージw

532 :132人目の素数さん:2020/06/02(火) 19:19:19.05 ID:EuotxuXg.net
>>529
>zとz*を独立変数とみなす
ということと、それに対してz(またはz*)で偏微分する、ということを、どう理解すればいいのでしょうか

「g(x,y)をG(z,z*)と表し、zとz*を独立変数とみなしてG(z,z*)をzで偏微分」というのを、
表式上は∂/∂zと書いてるけど、実際は∂/∂z=(∂/∂x-i∂/∂y)/2で定義されている、という形ではなく、
実際に2変数関数G(a,b)を偏微分したものとして「zとz*を独立変数とみなしてzで偏微分」という言葉に近い形で理解することはできるのでしょうか?

この形で理解しようとすると、a≠b*でもG(a,b)が定まっている必要があると思いますが、
一般にg(x,y)が与えられても、G(a,b)は一般にはa=b*でしか定まらなさそうで、a≠b*でのG(a,b)の値をどう定めればいいのかが分かりません
(g(x,y)が多項式の場合には、xとyをzとz*で置き換えてzとz*を独立変数とみなす、という手続きでa≠b*でのG(a,b)の値を定めることができるのは分かりますが、
 g(x,y)が各点での値としてだけ与えられている場合にも、a≠b*でのG(a,b)の値を定めることができるのでしょうか?)

533 :132人目の素数さん:2020/06/02(火) 19:45:05.60 ID:kxqNN22N.net
「実際は」の中身があったうえで「形式的に」そう見えるから面白いアナロジー
っていうのを、だから形式のほうが本質でそこに元とは違う実質があるというのはこじつけでは?

置換積分で微分商の分母を払うみたいなことをやるから微分商は分数なんだ
といわれても、詳しく調べていくと齟齬が出てくるみたいなことになればやめるでしょ

534 :132人目の素数さん:2020/06/02(火) 20:22:54.70 ID:EuotxuXg.net
こじつけ、というか、そういう理解はできないのでしょうか、という質問です

また、「実際は」の中身からは、「形式的に」そう見えることを直感的に理解できればいいなと思ったもののできる気がしないので、
何か行間を埋めるような理解はできないのかな、というのが大元の疑問です

そして、∂/∂zを「zとz*を独立変数とみなしてzで偏微分」という言葉に近い形で理解できれば、
それを手掛かりに行間を埋められはしないかな、という期待の元で>>532の質問をしています

535 :132人目の素数さん:2020/06/02(火) 21:14:14.05 ID:TPydHgX/.net
>>532
g(x,y) だけでなく G(a,b) も与えられている(既知)とします。
一般に2変数の場合
 dG(a,b) = (∂G/∂a)da + (∂G/∂b)db
      ↑     ↑
     b=一定  a=一定

     = G_a(a,b) da + G_b(a,b) db
です。

a,bが鏡像の場合は
 dG(z,z*) = G_a(z,z*) dz + G_b(z,z*) dz*
なので
 dg(x,y) = (∂g/∂x) dx + (∂g/∂y) dy
      ↑     ↑
     y=一定  x=一定

    = (G_a+G_b) dx + i(G_a-G_b) dy,
したがって
 (1/2)(∂g/∂x - i・∂g/∂y) = G_a(z,z*)
となりますがこれを記号的に
 G_a = (∂G/∂z) と表わしたのでしょう。

536 :132人目の素数さん:2020/06/02(火) 21:27:25.43 ID:TPydHgX/.net
g(x,y) に対して
 G(a,b) = g((a+b)/2, (a-b)/2i)
とおくことはできますが・・・・

537 :132人目の素数さん:2020/06/02(火) 21:27:34.54 ID:iA0eGlWC.net
ウィルティンガーの微分ってやつ?
何に使うのこれ?

538 :132人目の素数さん:2020/06/02(火) 22:00:42 ID:kxqNN22N.net
>>534
その「質問」への解答が「ただのこじつけだろ」ってことだって言ったつもりだったんだが
質問への難癖と思われたのか

539 :132人目の素数さん:2020/06/02(火) 23:39:19.45 ID:TPydHgX/.net
Gが「zが現れないz*のみの関数」のときは
 G_a(0, z*) = 0,
 (1/2)(∂g/∂x - i・∂g/∂y) = G_a(0, z*) = 0,
となりますが、これを記号的に
 (∂G/∂z) = 0,
のように表わすこともあるんだろうな。

540 :132人目の素数さん:2020/06/02(火) 23:53:19.17 ID:EuotxuXg.net
>>535,
なるほど
とりあえずG(a,b)がうまく与えられているとすると良い感じにイメージできそうな気がしました

>>537
そういう名前がついているんですね
ググるのにも難儀していたので、とても助かります

>>538
すみません、私の心が汚れていて悪く捉えてしまいました

541 :132人目の素数さん:2020/06/03(水) 10:26:50.12 ID:ii0n5Inq.net
「 z が現れない z* のみの関数」というのがよくわからない
例えば、 f(z*) = (z*)* とすれば、 ∂f/∂z = 1 だが

542 :132人目の素数さん:2020/06/03(水) 14:23:40 ID:VkvJF3Uh.net
間違い
g(z) = f(z*) = (z*)* とすれば ∂g/∂z = 1 だが
f(z) = z* だから ∂f/∂z は 0

543 :132人目の素数さん:2020/06/03(水) 14:52:10 ID:ii0n5Inq.net
f(z*) = z に注意
これを z* の関数とみなすか、 z の関数とみなすかで微分の結果が変わるのはおかしい

544 :132人目の素数さん:2020/06/03(水) 15:48:59.51 ID:SoWvP8oK.net
二変数函数 F(x,y) が最初にあり、x=z, y=z* を代入した函数 F(z,z*) を考える
すると F の二つの偏微分 F_x(x,y), F_y(x,y) に x=z, y=z* を代入した F_x(z,z*), F_y(z,z*) を
つくることができるが、これを記号の濫用で ∂F/∂z, ∂F/∂z* と書く

多変数函数の第一引数での偏微分の各成分に第一引数と同じ変数をパラメータとする函数を代入するときと
多変数函数に第一変数をパラメータとする函数を各成分に代入した一変数函数をそのパラメータ変数で常微分する
という二つの操作が混同しかねない記法で書かれるという多変数偏微分でよくある面倒な事例

545 :132人目の素数さん:2020/06/03(水) 15:51:26.09 ID:/bcrEffA.net
>>527-543
https://imgur.com/g4z5e8P

546 :132人目の素数さん:2020/06/03(水) 16:01:11.11 ID:ii0n5Inq.net
>>544
それでも解決しない気がする
F(x, y) = (x*)* とすると、 F(z, z*) = z で、
F_x(x, y) = 1 より、 F_x(z, z*) = 1 だが、
G(x, y) = y* とすると、 G(z, z*) = z で、
G_x(x, y) = 0 より、 G_x(z, z*) = 0
しかし、 F(z, z*) = G(z, z*)
これはどう解釈すればいい?

547 :132人目の素数さん:2020/06/03(水) 16:04:14.92 ID:SoWvP8oK.net
>>546
x,yは実変数、z.z*を代入するのは形式的操作で記号の濫用
でいいとおもう

548 :132人目の素数さん:2020/06/03(水) 16:09:37.45 ID:ii0n5Inq.net
>>547
形式的操作だとどう違う?
x, y が実変数なら y* = y だけど、この y に z* を代入すると
z = z* にならない?

549 :132人目の素数さん:2020/06/03(水) 16:10:03.46 ID:/bcrEffA.net
d・d=0
の幾何学的イメージでここまで技術論に逃げ込めるのは或る意味凄い才能ですなあ
という印象

550 :132人目の素数さん:2020/06/03(水) 16:40:10.18 ID:SoWvP8oK.net
>>548
y*自体が出てこない
代入後にz*が出て来たら、それはy由来のものでx由来のz*が存在することはない、という意味<形式的操作

551 :132人目の素数さん:2020/06/03(水) 16:42:18.73 ID:SoWvP8oK.net
>>548
y*がないということは、z*はあってもz**は考えることはないといった方が適切だったかも

552 :132人目の素数さん:2020/06/03(水) 16:58:03.69 ID:ii0n5Inq.net
よくわからない
実変数に対して複素数特有の操作ができないってこと?
別の例で言えば、
H(x, y) = Re(y) + Im(y) とすると、 y が実変数なら H(x, y) = y だが
Re(y) + Im(y) → Re(z*) + Im(z*) ∊ R
y → z*
これもおかしい

553 :132人目の素数さん:2020/06/03(水) 17:25:07 ID:/bcrEffA.net
カッツの太鼓の問題とかリウヴィルの定理とかそっち方面の幾何学的含意物理学的言い換えが問われてるんじゃないの?。

554 :132人目の素数さん:2020/06/03(水) 17:26:57 ID:ii0n5Inq.net
関連してこんな例を思いついた
g(z) = Re(z*) とすると、
∂g/∂z = 1/2 ≠ 0

555 :132人目の素数さん:2020/06/03(水) 17:44:02.90 ID:ii0n5Inq.net
さらに>>544の方法で
F(x, y) = Re(x)
G(x, y) = Re(y)
とおくと、F_x(z, z*) = 1 ≠ 0 = G_x(z, z*) だが、
F(z, z*) = G(z, z*)

556 :132人目の素数さん:2020/06/03(水) 18:00:07.44 ID:0v5Wpgob.net
>>553
だとしたら物理板で聞くのでは?

557 :132人目の素数さん:2020/06/03(水) 18:16:46.85 ID:ii0n5Inq.net
>>554
>>555
Re だと Re(z) = Re(z*) だからあまり適切な例ではないか
でも g(z) = Im(z*) なら g(z*) = - g(z) で、∂g/∂z = i/2 ≠ 0

>>544の方法の例なら、例えば
F(x, y) = x
G(x, y) = y - 2i*Im(y)
ととれる

558 :132人目の素数さん:2020/06/03(水) 18:48:14.56 ID:SoWvP8oK.net
>>552
そう、全く完全に実数の操作しかない実函数に、複素変数が後から濫用で代入される

559 :132人目の素数さん:2020/06/03(水) 18:52:26.37 ID:SoWvP8oK.net
>>557
どうしてもReやImを使いたいなら実函数してidと0に退化させてからじゃないかな
形式代入でzとz*の函数に替えてもidと0のままでReもImも復元されない

560 :132人目の素数さん:2020/06/03(水) 19:18:11.85 ID:ii0n5Inq.net
>>558
>>559
では絶対値はどうか
すなわち、
F(x, y) = |x|
G(x, y) = |y|
とすれば、これは全く完全に実数の操作しかない実関数だが、
F(x, y) は x ≠ 0 で偏微分可能で、F_x(x, y) ≠ 0 = G_x(x, y) だが、
F(z, z*) = G(z, z*)

561 :132人目の素数さん:2020/06/03(水) 19:38:33.43 ID:SoWvP8oK.net
>>560
それは解決できないんじゃないかな

なんにせよ、複素函数がヴィルティンガー微分を介して
実二変数函数の微分法っぽく振る舞うって言ったところで
俺の立場は>>533なので、理論のサブセットにしか議論が
適用できなくても特に不満はないのだけど

562 :132人目の素数さん:2020/06/04(木) 15:14:07.37 ID:hYLmjYDz.net
>>543
それが偏微分の特徴
偏微分では何を独立変数にしてるか常に意識してないとダメ
これを「おかしい」と言ってたら数学はできない

563 :132人目の素数さん:2020/06/04(木) 15:25:21.55 ID:eoDnCkjr.net
結局、 z と z* は独立でないから
>>527
>zが現れないz*のみの関数に∂/∂zを作用させると0になる

は一般には成り立たないってことなんだよね
複素共役をとる関数を f(z) = z* とすると、 z と z* の「2変数関数」 G(z, z*) は
G(z, z*) = G(z, f(z)) = G(f(z*), z*)
と書けてしまうから、 z と z* の多項式とか、そういう特別な場合しか成り立たない

564 :132人目の素数さん:2020/06/04(木) 18:51:22.16 ID:KwC6Ygxo.net
>>563
> と書けてしまうから、 z と z* の多項式とか、そういう特別な場合しか成り立たない
これはちょっと。

複素平面上で定義された複素数値関数f(x+iy)=u(x,y)+i v(x,y)を考える。・
そこで、偏微分として、∂_xと∂_yを考えましょうというのが実関数の延長線上での話。

その代わりに、∂_z=(∂_x-i ∂_y)/2, ∂_z*=(∂_x+i ∂_y)/2 を考えようってだけ。
そうすると、実関数の偏微分の時と同じような(全く同じとは言わない)種々の公式が成り立ち、
それに基づいて議論できるということ。

f(z, z*)と書くことはあるけれど、これは、あくまでz=x+iyから決まる関数f(z)のこと。
あと、f(z)に∂_z*を作用させて0になるというのが、fが正則であることに対応する。

chain ruleとかも成り立つし、∂_z (f*)=(∂_z* f)* とかの公式もあるから、多項式とかよりも一般に使えるもの。
正則関数しか出てこないようなシチュエーションでは、単に∂_zが微分だから、自然な一般化でもある。

565 :132人目の素数さん:2020/06/04(木) 18:59:02.18 ID:eoDnCkjr.net
>>564
特別な場合しか成り立たないことは確か
多項式はただの例
反例は>>557にある

566 :132人目の素数さん:2020/06/04(木) 19:56:15.97 ID:OqTWAEoM.net
離散力学系について質問です。
T^px=T'(T^(p-1)x)T'(T^(p-2)x)T'(x)を示せ

567 :132人目の素数さん:2020/06/04(木) 20:06:46.78 ID:KwC6Ygxo.net
>>565
> >>564
> 特別な場合しか成り立たないことは確か
> 多項式はただの例
> 反例は>>557にある
反例って何の反例?

z*のみの関数っていうものの意味を明確にしていないからおかしな勘違いが生まれていると思うのだが。
あと、そのあたりがあいまいなところで議論してもしょうがないので、
z*のみの関数に∂_zを作用させたら0になるという主張はしていない。
ウィルティンガーの微分は、多項式とかに限らずに使われるよってだけ。

z*のみの関数というのにこだわるなら、私の主張は、zの正則関数f(z)にz*を代入したf(z*)について、
∂_z f(z*) =0 となる。ということ。

あと、∂_z Im z*=∂_z (i z - i z*) /2 = i/2 で、557の計算結果がおかしいとは思わない。

568 :132人目の素数さん:2020/06/04(Thu) 20:29:27 ID:eoDnCkjr.net
>>567
>>527の人?
反例は、>>527
>zが現れないz*のみの関数に∂/∂zを作用させると0になる

の反例
ただ、「zが現れないz*のみの関数」というものの定義が明確でないことはおっしゃる通り
反例の意味は、もし Im(z*) を「zが現れないz*のみの関数」とするなら、これに ∂/∂z を作用させても 0 にならないということ

>ウィルティンガーの微分は、多項式とかに限らずに使われるよってだけ。

それはもちろんその通り

>z*のみの関数というのにこだわるなら、私の主張は、zの正則関数f(z)にz*を代入したf(z*)について、
>∂_z f(z*) =0 となる。ということ。

そうなの?
ウィルティンガーの微分に詳しくないから正しいかどうかわからない

569 :132人目の素数さん:2020/06/04(Thu) 20:39:07 ID:KwC6Ygxo.net
>>568
> >>567
> >>527の人?
564, 567は私だけど、527は別人。
そういえば昔、関数論で習ったなぁと懐かしくなって出てきただけ。

527に対するイメージ的な答えは持ってない。
何で正則ならコーシー・リーマンの式を満たすのか?とかと似たような問いかけだと思うが。

570 :132人目の素数さん:2020/06/04(木) 21:13:15.08 ID:SGJYxQyS.net
>>566
文字バグってますが、分かりやすいサイトでもいいのでお願いします

571 :132人目の素数さん:2020/06/05(金) 09:24:58.30 ID:1j2eCmzC.net
研究者の方に聞きたいんだけど、数学の研究を
大規模コンピューターシステムの開発みたいに、各人が各部位を担当して、その結果を統合することによって大きな結果を出すようなことって出来ないんですかね
その手法が出来たら数学の発展が飛躍的に進と思うんだが

572 :132人目の素数さん:2020/06/05(金) 11:10:09 ID:MnfO91HS.net
そんなの研究者でなくてもみんなやってるだろ?

573 :132人目の素数さん:2020/06/05(金) 11:28:21 ID:1j2eCmzC.net
>>572
現在の数学の理論が構築されたのは、
・予めおおよそ結論が見えていてそこに向かっていたのではない(?)し
・各人が部分じゃ無くて個として研究している
ので、大規模コンピュータシステムの開発(…ファイナルファンタジー7みたいな大作ゲームでもいい)とは違う

574 :132人目の素数さん:2020/06/05(金) 12:01:42 ID:jT734fJW.net
皆が同じテーマ(の一部)を研究している研究室とかはあるかもしれないけど、
それは所謂ブラック研究室ってやつで、その研究室では自由な研究はできないだろうね

575 :132人目の素数さん:2020/06/05(金) 13:28:34 ID:tVW2geYu.net
数学だとどこに向かうかが既に提示されてるなら
個々が部品作れば全体としては自然と統合された大きい成果になると思うけど
それと比べてチームであることは何か利点ある?
工数が通常読めないなかで振り分けをきっちりやることが適切とも思えないけど

576 :132人目の素数さん:2020/06/05(金) 13:40:28 ID:OOzg85/a.net
有限単純群の分類とか3次元閉多様体の分類とか
分類関係だと多いね

577 :132人目の素数さん:2020/06/05(金) 13:43:47.42 ID:wTPNLp/M.net
数学は基本的にa train of thoughtだから
途中の車両を分担するというのは難しい

578 :132人目の素数さん:2020/06/05(金) 23:16:07 ID:GykoBOcR.net
0×∞は不定形って話を聞いて納得いかないです
lim(x→∞)f(x)=∞、lim(x→∞)g(x)=0のとき
lim(x→∞)f(x)×lim(x→∞)g(x)は速度の違いで不定形なのも納得できますが
0×lim(x→∞)f(x)
=lim(x→∞)0×f(x)
=lim(x→∞)0
=0
ってことで0と∞かけたらどうあがいても0になるように思うんですが

579 :132人目の素数さん:2020/06/05(金) 23:32:48.82 ID:PqYH7W6A.net
> 0×∞は不定形
というときの 0×∞ は 0×(有限) の極限という意味ではない
逆に、測度論では 0×(有限) の極限 としてしか現れないという
前提のもと 0×∞=0 と規約を設けるのが当たり前にある

要するに、納得できないとかじゃなく、おまえに文脈を踏まえる能力が足りないだけ

580 :132人目の素数さん:2020/06/06(土) 03:00:42.19 ID:Wq7+Ly5a.net
質問スレなんだから普通に答えりゃいいのに

581 :132人目の素数さん:2020/06/06(土) 03:36:35.65 ID:ero6ji90.net
回答者様の権利で人格の底の底まで冒涜しても良い、とでも思ってんだろ
質問者様の権利で人格の底の底まで冒涜されてでも回答に感謝崇め奉れって魂胆なんだろ
昭和だったらボコボコにぶちのめされ、それを警察に訴えても聞いて貰えず世から弾かれてただろ

582 :132人目の素数さん:2020/06/06(土) 03:52:52.21 ID:klZxi4yn.net
雑魚は失せな

583 :132人目の素数さん:2020/06/06(土) 05:03:28.35 ID:9TTPHkFL.net
>>581
バブル世代並みに薄っぺらそう

584 :132人目の素数さん:2020/06/06(土) 05:49:12 ID:ero6ji90.net
残念ながら人間社会も例に漏れず自然界、人間も例に漏れず動物なんで、
人間社会の綺麗事という名の人の皮を剥けば鬼畜生になるんだわ。
その良い例がネットを良い事に人の皮が剥けて悪態つける>>579だし、
人のレスを見て高を括って人の皮が剥けて舐めた口の聞き方に変わった>>582-583なんだわ。

585 :132人目の素数さん:2020/06/06(土) 11:50:20.88 ID:TR+Z6nNi.net
というID:ero6ji90の自己紹介でした

586 :132人目の素数さん:2020/06/06(土) 11:52:39.48 ID:0/4QKsok.net
こいついつも自己紹介してんな
しかも他のスレだとなぜかトリップ付けない迷惑な奴

587 :132人目の素数さん:2020/06/06(土) 12:22:58 ID:9TTPHkFL.net
eroさんとは概してコンドームやコンデンサーペーパー紙一重で隔てられてるだけマシというものだなあという印象

588 :132人目の素数さん:2020/06/06(土) 15:25:14.44 ID:ero6ji90.net
アンカー付いてんの無視して十把一絡げにして言い括るって、話術だよなぁ
普段からこういう話術を駆使してる所を見ると俺が指摘する迄もなくお前らは
都合に応じて人の皮を脱ぐ事をやってきたって事だな。将来安泰だな、困ったら人を幾らでも騙すんだろ

589 :132人目の素数さん:2020/06/06(土) 19:50:10 ID:jsznH9kO.net
正則である必要十分条件は固有値に0を持たない
(→)対偶を示す。固有多項式のλに0を代入するとdetA=0となりAは正則でない。よって→は成立。
(←)対偶を示す。正則でないのでdetA=0でdetA=det(A-0E)=g_A(0)だから0を固有値に持つ。よって←は成立。
これであってますか?

590 :132人目の素数さん:2020/06/06(土) 21:52:24 ID:QNbRyMAA.net
>>589
大丈夫かと

591 :132人目の素数さん:2020/06/07(日) 15:03:48.47 ID:uwiXF036.net
証明終わりの□や■が美しくなくて大嫌いなのですが
どうしたらいいでしょうか

592 :132人目の素数さん:2020/06/07(日) 15:06:06.80 ID:UVqHD4eS.net
>>591
うんちマーク(巻き糞)にするといいよ!

593 :132人目の素数さん:2020/06/07(日) 16:10:44.90 ID:ou9k9Tz1.net
多項式の正規直交化の簡単な方法ってないですか?シュミットの方法でやってるんですがめんどすぎるので

594 :132人目の素数さん:2020/06/07(日) 16:18:33.69 ID:LUPfFH8x.net
>>593
あるわけないだろ

595 :132人目の素数さん:2020/06/07(日) 18:46:05.32 ID:yxPmZpYB.net
正規直交基底が得られれば良く、それ以上の条件無いなら
既に知られてるのを調べるのが一番簡単と思う

596 :132人目の素数さん:2020/06/07(日) 20:48:50.57 ID:ou9k9Tz1.net
A=P^-1APが成り立つのはどんな時ですか?

597 :132人目の素数さん:2020/06/07(日) 21:06:34.15 ID:VQUHw7VB.net
AとPが可換のとき

598 :132人目の素数さん:2020/06/07(日) 21:37:14 ID:ou9k9Tz1.net
https://i.imgur.com/podjbHD.jpg
gA(t)=(t-λ1)gB(t)が成り立つのはなんでですか?
右辺ってgQ^-1AQ(t)のことで、これがgA(t)になるにはAとQは可換でないといけないですよね?でも可換かわからなくないですか?

599 :132人目の素数さん:2020/06/07(日) 21:57:11 ID:wrvFfNlQ.net
>>598
detは類函数だろ

600 :132人目の素数さん:2020/06/07(日) 21:58:02 ID:1GHLlal/.net
相似な正方行列の固有多項式は一致する

601 :132人目の素数さん:2020/06/07(日) 21:59:33 ID:thBe7J1E.net
>>598
両辺の行列の固有多項式を考えてるだけだよ
左辺の行列であるQ^-1AQはAと相似なのでその固有多項式はAの固有多項式に等しい
右辺の行列の固有多項式は定義通りに計算してるだけ

602 :132人目の素数さん:2020/06/07(日) 22:01:50 ID:ou9k9Tz1.net
理解できました、ありがとうございます。

603 :132人目の素数さん:2020/06/07(日) 23:31:12 ID:SYpW8IXo.net
>>598
何読んでるかわかった
どの程度の大学なんだろうな
しょーもない質問ばっかりしてるから大体わかるが

604 :132人目の素数さん:2020/06/08(月) 07:39:19.49 ID:sJyhGEg2.net
>>603
低学歴ですみませんねぇ

605 :132人目の素数さん:2020/06/08(月) 10:30:29.86 ID:pW8SSyKl.net
>>603
> 何読んでるかわかった

かまわん、続けたまえ

606 :132人目の素数さん:2020/06/08(月) 12:47:57.22 ID:+xKH1hoV.net
劣等感がdisるのなんか無視すりゃいいのさ

607 :132人目の素数さん:2020/06/08(月) 15:39:28 ID:X0SeXyoi.net
>>598への回答は「相似な正方行列の固有多項式は一致する」
というものでしかなく、相似だとなぜ固有多項式が一致するのかは
説明してないので、これでは質問者の疑問を「相似」という言葉で
置き換えただけであって、本当は回答になっていない。
相似だと固有多項式が一致する理由は

・ tI−Q^{-1}AQ = Q^{-1}(tI−A)Q

・ det(tI−Q^{-1}AQ) = det(Q^{-1}(tI−A)Q) = det(Q^{-1})det(tI−A)det(Q)
  = det(tI−A)det(Q^{-1})det(Q) = det(tI−A)det(Q^{-1}Q) = det(tI−A)

という計算による。
質問者は、回答がついてから2〜3分で「理解できました」と言っているが、
「相似」の一言でここまで辿り着けるなら、
質問せずとも自力で理解できていたのではないだろうか。
この人、本当に理解していたのだろうか。

608 :132人目の素数さん:2020/06/08(月) 15:50:36.13 ID:PSTZZXpa.net
無限次元ヒルベルト空間を作用域とするエルミート演算子Aがあるとします。
何らかの固有ベクトル x が存在する事を示してください。 (Ax = λx , 実数: λ)
具体的な構成方法あればベターですが、存在証明だけでも構いません。
(出典無しの思いつき命題なので、条件によっては存在しないかも)

どう手を付けたらいいのかお手上げです。 有限次元だと簡単なのは分かります。

609 :132人目の素数さん:2020/06/08(月) 16:06:32.80 ID:Oyl3lrND.net
>>608
Aの点スペクトルσ_p(A)が空でないとして、その元λに対してAψ=λψとなるψが存在する
無限次元の場合はむしろこれを固有ベクトルと定義する
ただしλは複素数を取りうるので、λを実数に制限するなら実数に制限すれば求める条件が得られる

610 :132人目の素数さん:2020/06/08(月) 16:26:30.59 ID:PSTZZXpa.net
>>609
すみません、疑問は 「σ_p(A)が空でない」ことの証明も含みます。
それと有限次元でエルミート行列の固有値は 実数なので、無限次元でもそうだと思ったのですが例外があるのでしょうか。
λ||x||^2 = (x, Ax) = (Ax, x) = λ^* ||x||^2

611 :132人目の素数さん:2020/06/08(月) 16:27:23.40 ID:jv/i8Uhl.net
Z[x]/(2x+1,x^2-5)≅Z[x]/(x+1,x^2-20)
ですか?
同型なら、どうやって示せばよいのでしょうか?
よろしくお願いします。

612 :132人目の素数さん:2020/06/08(月) 17:47:15 ID:+qlIDWgG.net
>>608
> 無限次元ヒルベルト空間を作用域とするエルミート演算子Aがあるとします。
> 何らかの固有ベクトル x が存在する事を示してください。 (Ax = λx , 実数: λ)
> 具体的な構成方法あればベターですが、存在証明だけでも構いません。
> (出典無しの思いつき命題なので、条件によっては存在しないかも)
>
> どう手を付けたらいいのかお手上げです。 有限次元だと簡単なのは分かります。
https://ja.wikipedia.org/wiki/%E3%82%B9%E3%83%9A%E3%82%AF%E3%83%88%E3%83%AB%E5%AE%9A%E7%90%86
の有界自己共役作用素の項に固有値を持たない例があるみたい。
正しいかは知らない。

613 :132人目の素数さん:2020/06/08(月) 18:01:08.36 ID:xLDJNKOk.net
ずらしていくだけみたいな奴で固有ベクトルないのってないかな

614 :132人目の素数さん:2020/06/08(月) 18:11:53.33 ID:Oyl3lrND.net
>>610
エルミート作用素である(固有ベクトルが定義できる条件より強い)ことを見落としてた、すまん

615 :132人目の素数さん:2020/06/08(月) 18:21:11 ID:4nsS10XA.net
整係数多項式を xx-c で割った余りは1次式: ax+b,
これを x+1 で割った余りは b-a.
一方、2x+1 で割った余りは mod(a,2)x + b - [a/2]
同型とは思えぬが

616 :132人目の素数さん:2020/06/08(月) 19:36:00 ID:D5XhObEq.net
>>611
x+10 = x(2x+1) - 2(x^2-5), 19 = 2(x+10) - (2x+1) より (2x+1, x^2-5) ⊃ (19, x+10)
2x+1 = 2(x+10) - 19, x^2-5 = (x-10)(x+10) + 5*19 より (2x+1, x^2-5) ⊂ (19, x+10)
よって (2x+1, x^2-5) = (19, x+10) = (19, x-9)
したがって Z[x]/(2x+1, x^2-5) ≅ Z/19Z (x &#x21A6; 9)
同様に (x+1, x^2-20) = (19, x+1), Z[x]/(x+1, x^2-20) ≅ Z/19Z (x &#x21A6; -1)

617 :132人目の素数さん:2020/06/08(月) 20:49:03.65 ID:DAWjkcK7.net
>>616
>Z[x]/(2x+1, x^2-5) ≅ Z/19Z (x &#x21A6; 9)

文字化けしているけど、写像 F: Z[x]/(2x+1, x^2-5) → Z/19Z を
F(f(x) + (2x+1, x^2-5)) = f(9) + 19Z
によって定めると、これはwell-definedで、しかも環の同型になるってこと?
マジか

618 :132人目の素数さん:2020/06/08(月) 21:39:53.75 ID:DAWjkcK7.net
>>617
しかも同型を逆に辿れば I = (2x+1, x^2-5) とおくと
Z[x]/I = {0 + I, 1 + I, … , 18 + I}
で、 Z/19Z は体だから Z[x]/I も体になるのか

619 :611:2020/06/09(火) 05:33:14.85 ID:NAChmZS+.net
>>616
ありがとうございました。

620 :132人目の素数さん:2020/06/09(火) 08:26:48.66 ID:ikqpxXBp.net
大学数学の勉強法を教えてくれませんか?

621 :132人目の素数さん:2020/06/09(火) 12:31:23.48 ID:eYq+xinT.net
>>616
こういうのってどうやって思いつくの?
何か知られているやり方があるの?

622 :132人目の素数さん:2020/06/09(火) 15:01:40.11 ID:poOS9jb4.net
>>621
つーか
Z[x]/(x+1)=Zガンダム
だろ

623 :132人目の素数さん:2020/06/09(火) 16:29:04.93 ID:eYq+xinT.net
(2x+1, x^2-5) = (19, x+10) = (19, x-9)
は試行錯誤で見つけるの?
偶然 x-9 がモニックな1次多項式だから、 x に 9 を代入する写像が単射になるだけで、
モニックな1次多項式じゃなければ単射になるとは限らないよね?

>>622
ガンダム?

624 :132人目の素数さん:2020/06/10(水) 06:18:21 ID:3jg++8t0.net
整数列であるにもかかわらず、その一般項を初等的に表そうとすると無理数が表記上現れてしまう事がある場合があります(例:フィボナッチ数列)が、
そうなる場合とそうならない場合って何が原因ですか?

625 :132人目の素数さん:2020/06/10(水) 07:54:38.32 ID:LfsSAZ7q.net
互除法

626 :132人目の素数さん:2020/06/10(水) 10:45:00.98 ID:DqbTfruJ.net
>>624
例えば、下のような3項間漸化式で定まる整数列があったとする。
A[n+2]=p*A[n+1]+q*A[n]
nをn+kに替えると
A[n+k+2]=p*A[n+k+1]+q*A[n+k]
となる。

ここで、二次方程式、x^2=px+q を考える。両辺にx^n あるいは、x^(n+k)を掛けると
x^(n+2)=px^(n+1)+qx^n 、 x^(n+k+2)=px^(n+k+1)+qx^(n+k)
両者には、強い親和性があることに気づくはず。

実際のところ、A[n+2]=p*A[n+1]+q*A[n] 形式で与えられる数列は、
二次方程式、x^2=px+q の解を使って表現できる。

x^2=px+q の解が、無理数を用いなければ表せない場合は、
当然A[n]の一般項も、無理数を用いなければ表せない。

627 :132人目の素数さん:2020/06/10(水) 11:03:14.73 ID:FRMzf4FR.net
>>624
線型回帰数列に限れば、特性多項式の根が全て有理数になるかどうかで決まるな
もちろん一般には表し方に依るから何とも言えない

【例】漸化式
a_1 = 1
a_{n+1} = (n*a_n) + 1
で定義される整数列の一般項

面白い問題おしえて〜な 32問目
https://rio2016.5ch.net/test/read.cgi/math/1586230333/314

628 :132人目の素数さん:2020/06/10(水) 12:31:18.84 ID:3jg++8t0.net
あっ、特性多項式の根が、解空間たるベクトル空間の基底になるからか

629 :132人目の素数さん:2020/06/10(水) 12:46:45.63 ID:Pr+hUW1Y.net
商群の問題なのですが、Q/Zという商群は具体的にはどうなりますか?
例えばZ/2Zだと具体的には{0,1}になりますよね🙋
簡単な問題かもしれなくてすみません💦

630 :132人目の素数さん:2020/06/10(水) 12:51:20.19 ID:cYefq+Ue.net
>>629
具体的に書こうとしたら長すぎるからってエラーなって書きこめないわ
ごめん

631 :132人目の素数さん:2020/06/10(水) 15:45:44.07 ID:QcYGbPiy.net
円周上の有理点だろ

632 :132人目の素数さん:2020/06/10(水) 16:14:13.35 ID:Krid0YN5.net
1/2log(1+x)/(1-x) (-1<x<1) のn次導関数の求め方教えてくれる人いませんか?

633 :132人目の素数さん:2020/06/10(水) 16:23:23.03 ID:1qamB+IX.net
>>629
Q∩[0,1)
ほんとに面白いのはR/Q

634 :132人目の素数さん:2020/06/10(水) 16:52:06.67 ID:FRMzf4FR.net
>>632
テイラー展開知らんの?

635 :132人目の素数さん:2020/06/10(水) 17:38:41.78 ID:fekDDIDe.net
>>630,633
ありがとうございます!😙
整数の違いが無視できるから0以上1未満の有理数で全て代表できるんですね
R/Qについても調べてみます󾟙

636 :132人目の素数さん:2020/06/10(水) 20:06:08 ID:85Y2ShY2.net
局所類体論とかでQ/Zは興味深い群だよねー

637 :132人目の素数さん:2020/06/10(水) 23:18:40.18 ID:zG7tRYEI.net
R/Q はどんなんでしょう。

638 :132人目の素数さん:2020/06/11(木) 01:15:19.33 ID:2VKGJNso.net
>>627
【例】
a_{n+1} = Σ[k=0,n] n!/k!    >318
   = e∫[1,∞] (t^n)e^(-t) dt  >319
   = [ n!e ]        >320-322

>>632
 (1/2){log(1+x) - log(1-x)}
をn回微分すれば
 (1/2)(n-1)!{(-1)^(n-1)/(1+x)^n + 1/(1-x)^n},

639 :132人目の素数さん:2020/06/11(Thu) 12:09:58 ID:q50aisbX.net
整数成分の二次正方行列Aについて
lim[n→∞]A^nが収束するとき、A^2=AまたはA^2=Oであることを示してください

640 :132人目の素数さん:2020/06/11(Thu) 15:24:09 ID:wu5ZyxZU.net
対角化不能であるもので収束するのは固有値が0の場合でこの時A^2=O。
二つの固有値の絶対値が1以下になる事が必要であるが、絶対値の積は整数だから積は-1,0,1のいずれかが必要。
±1のときは固有値は1の冪根であり、収束するから1しか取りえない。
この時冪等行列。
積が0のとき、固有値は少なくともひとつ0で、トレースも整数だからもう一方も整数、かつ絶対値が1以下だからもう片方の固有値は±1か0。
しかし収束するからやはり1か0。

641 :132人目の素数さん:2020/06/11(木) 16:38:08.47 ID:aF/rqx/4.net
>>639
整数列が収束⇔ある番号から先が一致
A^n=A^(n+1)

642 :132人目の素数さん:2020/06/12(金) 16:23:51.57 ID:pdxRRGtv.net
y=x/2+√(x+1) の逆関数の解き方と計算過程を教えてほしいです。

643 :132人目の素数さん:2020/06/12(金) 17:11:30 ID:dnFTYwvr.net
何がわからないのかわからない
±のどちらが逆関数になるかと定義域に気を付けるだけの問題に見えるが

644 :132人目の素数さん:2020/06/12(金) 17:41:29.80 ID:pdxRRGtv.net
>>643
>>642の式を x=… の形に式変形していく方法がわからないです
根号が混ざってて解き方に迷っています
初歩的な質問ですみません。

645 :132人目の素数さん:2020/06/12(金) 17:44:07.12 ID:dnFTYwvr.net
>>644
要は x について解けばいいわけでしょ?
y - (x/2) = √(x+1)
なんだから、この両辺を2乗すればいい

646 :132人目の素数さん:2020/06/12(金) 17:44:42.29 ID:hMAehrvS.net
ルート外してからの2次方程式の解の公式は試した?
最初からなんでもかんでも小綺麗にやろうとしたらだめだよ、まずは手を動かそうね

647 :132人目の素数さん:2020/06/12(金) 17:58:07.69 ID:pdxRRGtv.net
>>646
>>645
ありがとうございます!
解くことができました

648 :132人目の素数さん:2020/06/12(金) 18:45:49.93 ID:425wrA8u.net
(P,≦)は有限な順序集合
≦から(標準的に?)位相を定める。
φ≠Aを正則開集合とする。
Aの極小元をp1,...,pnとした時
A=op・cl{p1,...,pn}を示せ。
(opは開核作用素、clは閉包作用素)
⊇は明らか
⊆であるが、Aは正則開集合なので、A⊆cl{p1,...,pn}を示せばよい。

ですが,分かりません

649 :132人目の素数さん:2020/06/12(金) 19:13:47.75 ID:425wrA8u.net
イメージ的には、各p_iが"枝の末端"で、閉包作用素によって"先祖を辿りながらかき集めていく"から、主張の成立は分かるんだが。

650 :132人目の素数さん:2020/06/12(金) 20:51:53.16 ID:dnFTYwvr.net
>>648
よくわからないんだが、本当に成り立ってる?
P = {1, 2, 3} に通常の整数の大小関係で順序を定めるとき、
A = {1, 2} の極小元は 1 のみで、
op・cl{1} = {1} じゃないの?
これ間違ってる?

651 :132人目の素数さん:2020/06/12(金) 21:16:45.66 ID:425wrA8u.net
>>650
そのAは正則開集合じゃないね
clA=P

652 :132人目の素数さん:2020/06/12(金) 21:20:03 ID:dnFTYwvr.net
>>651
そうなの?
P - A = {3} = (2, ∞) が開集合だから A は閉集合で
cl A = A だと思ってた

653 :132人目の素数さん:2020/06/12(金) 21:45:08.28 ID:425wrA8u.net
>>652
Pの開集合系は{φ、{1},{1,2},{1,2,3}}だから。

654 :132人目の素数さん:2020/06/12(金) 21:53:24.78 ID:hMAehrvS.net
T0とかT1とか、みんなそれ覚えてるの?
それとも適宜調べて考えてるの?

655 :132人目の素数さん:2020/06/12(金) 21:54:26.55 ID:oLgfPUZm.net
≦は半順序っぽいけど、それにどう位相を入れるの?

656 :132人目の素数さん:2020/06/12(金) 21:56:41.51 ID:dnFTYwvr.net
>>653
所謂順序位相ではなくて?
{3} = (2, ∞) は開集合ではないの?

657 :132人目の素数さん:2020/06/13(土) 06:34:39.02 ID:RUCDf3th.net
>>656
≦は反射、対称、推移律を満たす(普通の)順序
順序集合Xとx∈Xに対して、[x]:={z∈X|z≦x}とする
{[x]|x∈X}によって生成されるXの開集合系をもってXを位相空間と考える

そうすると、P={1,2,3}に普通の順序を考えて位相空間を考えると、Pの開集合系は>>653のはず

658 :132人目の素数さん:2020/06/13(土) 11:06:40.67 ID:4tXYzXlR.net
>>657
なにその変な位相
[x] = (-∞, x] ってこと?
標準的な順序位相じゃないじゃん
なんか名前付いてるやつなんじゃないの?

659 :132人目の素数さん:2020/06/13(土) 11:54:31.07 ID:RUCDf3th.net
>>658
竹内外史の現代集合論入門の83ページ
でブール代数から順序、位相って定めてる

660 :132人目の素数さん:2020/06/13(土) 11:58:33.73 ID:4tXYzXlR.net
>>657
その位相なら>>648は証明できる

簡単のため、 B = {p1,...,pn} と置く。
A ⊆ cl B を示す。
a ∊ A の任意の近傍を N とする。 N ∩ B が空集合でないことを示せばよい。
>>657の位相において、任意の開集合は [x] の有限個の共通部分の和集合として書けるから、
N = ∪ O_λ として、 a が属する O_λ を1つ選び、それを O として、
O = ∩[k=1,m] [x_k] とする。このとき、全ての k に対して、 a ≦ x_k である。
すると、 b ≦ a となる b ∊ P は b ∊ [x_k] となるから、 b ∊ N である。
したがって、 p ≦ a となる p ∊ B をとれば、 p ∊ N である。

661 :132人目の素数さん:2020/06/13(土) 12:07:18.15 ID:pp0Pa+Vo.net
>>657
>順序集合Xとx∈Xに対して、[x]:={z∈X|z≦x}とする
x入れちゃうのか
なんかやだなそれ

662 :132人目の素数さん:2020/06/13(土) 13:02:28.61 ID:RUCDf3th.net
>>660
どうもです
∀a∈A∃p_i s.t. p_i≦a が要ですね
分かりました

663 :132人目の素数さん:2020/06/13(土) 13:14:05.28 ID:RUCDf3th.net
>>662
∀x∈G∃i p_i≦xである。
∵xが極小元でなければ、
y<xなるyをとれるがGは有限なので、
いつかはp_iに辿り着く。

664 :132人目の素数さん:2020/06/13(土) 18:35:50.08 ID:Xs2x9iFc.net
ルベーグの優収束定理の条件に、f_nもf_nの極限のfもリーマン可積分という条件をつけたものを考えれば(これを条件Aとします)
リーマン積分でも各点収束するf_nについて積分と極限を交換可能と言えると思うのですが
リーマン積分については、条件Aではなくf_nが一様収束するという条件で定理として書かれてる場合が多い気がして、
それが何故なのか気になっています

これは条件Aでリーマン積分と極限を交換可能と分かってもあまり嬉しくないからですか?
それとも、条件Aで積分と極限を交換可能とリーマン積分の範囲で示すのが面倒だからですか?

665 :132人目の素数さん:2020/06/13(土) 20:28:47.57 ID:4tXYzXlR.net
>>664
その主張は成り立たない

【反例】実数 R の閉区間 [0, 1] において、
f_n(x) = 4n^2 x (0 ≦ x ≦ (1/2n)),
4n^2 ((1/n) - x) ((1/2n) ≦ x ≦ (1/n)),
0 ((1/n) ≦ x ≦ 1)
とすると、 f_n(x) は n → ∞ で f(x) = 0 に各点収束する。
しかし、
∫[0,1] f_n(x) dx = 1

666 :132人目の素数さん:2020/06/13(土) 21:30:48.65 ID:Xs2x9iFc.net
>>665
f_nの係数は4n^2ではなく2nでしょうか?

それはともかく、考えてる条件Aはルベーグの優収束定理の条件にf_nもfもリーマン積分可能という他の条件を加えたもので、
ルベーグの優収束定理の条件はもちろん満たしてるものを考えているので、
f_nもfもリーマン積分可能なら、当然リーマン積分と極限を交換可能だと思います

その例だと、f_nを上から抑える、積分値が有限になる関数が存在しないので条件Aを満たしていないと思います

667 :132人目の素数さん:2020/06/13(土) 21:38:51.17 ID:4tXYzXlR.net
>>666
>f_nの係数は4n^2ではなく2nでしょうか?

f_n の係数は 4n^2 で合っているよ

条件については失礼、ちゃんと読んでいなかった
一様収束の代わりに一様有界としても成り立つことは「解析入門T」や「解析概論」には書かれているね
(アルゼラの定理)
解析概論によれば、「その証明はむずかしいから, ここでは述べない. 」ということらしいが
一様有界でも「むずかしい」なら、ルベーグの優収束定理の条件だと相当難しいんじゃない?

668 :132人目の素数さん:2020/06/13(土) 22:14:50 ID:A94tANLh.net
>>667
すいません、係数は勘違いしてました

リーマン積分では示すのが難しいからリーマン積分の文脈ではあまり触れない、っぽい感じですかね
ありがとうございます

669 :132人目の素数さん:2020/06/13(土) 23:51:30 ID:pQYFGc9G.net
Riemann積分は、そもそも有界区間で有界関数でないと定義されない。
なので、収束定理としては有界収束定理を考えれば十分といえる。

もちろん、Riemann広義積分に関しても優収束定理みたいなのを作ることは出来て、
それは、有界収束定理が出来ていれば、ひと手間かけるだけ。

結局のところ、有界収束定理の証明が面倒だからやらないというだけだと思う。

参考まで、
http://www.math.sci.hokudai.ac.jp/~ozawa/pdf/chokuhoutaijou.pdf
にアルツェラの定理の証明が紹介されている。

670 :132人目の素数さん:2020/06/13(土) 23:59:15.93 ID:Xs2x9iFc.net
>>669
ありがとうございます

671 :132人目の素数さん:2020/06/14(日) 01:57:06.86 ID:Mkz1zPCu.net
アルゼラの定理の証明は、

Arzela's Dominated Convergence Theorem for the Riemann Integral

でググるとpdfがそのまま読める。個人的にはこっちの方が読みやすい。
証明のやり方は>>669と本質的には同じだが、なぜか>>669は読みにくい感じがする。

672 :132人目の素数さん:2020/06/14(日) 20:08:43.40 ID:k1QKwFGm.net
n回のベルヌーイの成功の総数という確率変数の分散と各回の成功の数という確率変数の分散の関係の説明とその関係が成り立つ理由を説明しろという問題を出されたのですが、その関係性すらよくわかりません

各回の成功の数の確率変数の分散が、pq(成功の確率p 失敗をq)
総数の確率変数に分散が、平均の成功率pを一定にすると、np-Σ(k=1〜n)(pk^2) ※pkは成功の確率
であると配られたレジュメを見て考えたのですが、それすら当たっているかわかりません。

理解が浅くまともな質問が出来ず、ただ答えを聞くような感じになってしまいますが、教えていただけないでしょうか

673 :132人目の素数さん:2020/06/14(日) 21:42:23.84 ID:k1QKwFGm.net
それぞれの確率変数の分散を足したら、総数の場合の確率変数の分散になるという関係の説明をしないさいということなのかもしれませんね
もう少し自分で考えます。長文失礼いたしました。

674 :137番目の素数さん:2020/06/16(火) 21:19:27.67 ID:9e9m7S0m.net
行列について2つ質問があります。
@なぜベクトルの(x,y)などを行列では縦に書くのでしょうか。横に書くと1次変換の行列を後ろに掛けないといけなくなってめんどくさいとかなのでしょうか。
A固有値を求めるときに、定義式を整理すると
(A-λE)(固有ベクトル)=0になるのですが、この0はスカラーですか。それとも零ベクトルっていう解釈でいいのですか。

675 :132人目の素数さん:2020/06/16(火) 21:26:10.02 ID:hKoNkwWV.net
@転置とることで縦と横どちらでも相互に変換可能、だからどちらを採用するかは人の好みの問題
A整理する方法を見るか、もしくは左辺を計算してみたらどちらの意味かは明らかですよね

676 :137番目の素数さん:2020/06/16(火) 21:55:18.78 ID:9e9m7S0m.net
>>675
ありがとうございます。Aに関しては、固有ベクトルの一次変換だと考えて零ベクトルだという結論に至りました。

677 :132人目の素数さん:2020/06/17(水) 01:13:03.08 ID:ubQWTkww.net
質問自体が目的なのか

678 :132人目の素数さん:2020/06/17(水) 12:08:37.91 ID:Iebo131J.net
>>674
後に書く人も居るよ
てゆーか
作用素は後に書くのが本来は正当
関数もxfと書くべきなのに
欧米のSVOに合わせてfxと書いたのが元凶
置換なんかxστって書いていたら混乱もなかったに

679 :132人目の素数さん:2020/06/17(水) 12:22:44 ID:c2G3MCsy.net
本来の正当ってなんだ?
欧米で論文発表なんかの研究体制が整えられて欧米で主に研究進んできた歴史から行って正当なのは欧米だろう
個個にすごい人が少数いたってだけの未開地のルールなんてそれこそ傍流だろう

680 :132人目の素数さん:2020/06/17(水) 13:04:07.76 ID:ubQWTkww.net
逆ポーランド記法はHP電卓

681 :132人目の素数さん:2020/06/17(水) 13:49:10 ID:Iebo131J.net
文章が左から右に書くのに
作用素だけは右から左に抜けるからな
おかしいんだよ

682 :132人目の素数さん:2020/06/17(水) 13:51:06 ID:Iebo131J.net
ポーランド逆ポーランドとはまた別な話
そっちは中置演算記号を止めようという
至極尤もな話だ
逆ポーランドで統一が人類にとっては福音だろう

683 :132人目の素数さん:2020/06/17(水) 14:31:22.66 ID:SWOtibf0.net
日本語とか言う土人が使う言語で数学語ってるうちは五十歩百歩

684 :132人目の素数さん:2020/06/17(水) 14:36:59.30 ID:ql0odqd4.net
>>681
マイナー派閥は個性発揮する前に成果を出してね

685 :137番目の素数さん:2020/06/17(水) 16:48:06.24 ID:NarFfQgX.net
>>678
ありがとうございます。

686 :132人目の素数さん:2020/06/17(水) 21:31:02.22 ID:9Jnv/3PI.net
超限帰納法って必要ですか?物理学とか実世界への応用あるんですか?

687 :132人目の素数さん:2020/06/17(水) 22:24:23.28 ID:DbMxnCod.net
>>686
無限ゲーム理論ではたまに見ますね
何故そのような質問をされたのか分かりませんが、まあ、多くの専門ではそこまで必要ではないのではないでしょうか

688 :132人目の素数さん:2020/06/17(水) 22:37:40.38 ID:HCU7L5eH.net
こうやって掲示板で質問するということも実世界への応用と言えるのではないでしょうか

689 :132人目の素数さん:2020/06/18(木) 00:47:21.34 ID:/KxUQwGU.net
なアホな

690 :132人目の素数さん:2020/06/18(木) 02:55:10.14 ID:kb6U230C.net
超元気農法

691 :132人目の素数さん:2020/06/19(金) 11:40:20.49 ID:XbgJOmOQ.net
幾何的点について教えてください

「体k上のSpec(k)への射を持つスキームSを考える
Sにおける幾何的点ξはSpec(k^Sep)からSへの射のことを呼ぶ」
という定義ですが、k^Sepは体で、体の素イデアルは(0)だけですよね
ということは{(0)}からSへの射で、わざわざk^Sepやk^algを取る必要がない気がします
何故分離閉包や代数的閉包を使うのでしょうか?

692 :132人目の素数さん:2020/06/19(金) 11:40:52.20 ID:KOElBPqd.net
線形代数の質問です
n次正方行列Aを用いてR^n上の線形写像を
f_A(x)=Ax
と定める。このとき、Aが対称行列である事と
「f_A(W)⊂W⇒f_A(W^⊥)⊂W^⊥かつAの固有値が全て実数」
であることが同値であることを示して下さい
WはR^nの部分空間で^⊥は標準内積に関する直交補空間を表してます

693 :132人目の素数さん:2020/06/19(金) 12:30:45.35 ID:nxBpv/WV.net
>>691
例えば係数体kがQの場合で、X=Spec k[x]/(x^2-2)の場合、Qの分離閉包をKとでもすれば、幾何学的点p:Spec K→Spec k[x]/(x^2-2)はk代数の射 k[x]/(x^2-2)→Kで定まるけど、その場合、xを±√2のどちらに移すかで決まる。

694 :132人目の素数さん:2020/06/19(金) 12:54:52.10 ID:gzj6+i0k.net
>>692
「…」から対角化可能を示せば良いだろ

695 :132人目の素数さん:2020/06/19(金) 12:55:15.19 ID:JrB5K8+I.net
>>693
スキームの射が環の射から決まり、
環の射が分離閉包をターゲットにすることで決まってくる、ということですかね?
ありがとうございます

696 :132人目の素数さん:2020/06/19(金) 13:42:17.30 ID:voEP8x7P.net
>>692
→は容易。
複素数体に拡張されたエルミート内積も<,>で書くことにする。

Aがエルミート行列(i.e. 転置の複素共役で元に戻る)
⇔f_Aが自己共役(i.e. <u, f_A v> = <f_A u,v> ∀u,v)

は自明だから下の条件下でf:=f_Aが自己共役であることを示せば良い。
次元に対しての帰納法。
下の条件を仮定する。
固有値λをひとつ選べば、コレは実数だから固有実ベクトルvをひとつ選べる。
W=λRとおけばf(W)⊂Wであり、U=W^⊥とおけば仮定によりf(U)⊂Uでもある。
gとWの組みに帰納法の仮定が使えてgはW上の自己共役作用素となる。
よって任意のw1,w2∈Wとr1,r2∈Rに対して
<r1v+w1, f(r2+w2)>
= <r1v+w1, f(r2v)>+ <r1v+w1, f(w2)>
= <r1v, f(r2v)>+ <w1,f(w2)>
= <r1v, r2λv>+ <w1, f(w2)
= <r1λv, r2v>+ <f(w1), w2>
= <f(r1v), r2v>+ <f(w1), w2>
= <f(r1v), r2v+w2>+ <f(w1), r2v+w2>
= <f(r1v+w1), r2v+w2>
となり自己共役である。

697 :132人目の素数さん:2020/06/19(金) 13:50:01.66 ID:3Zdulx62.net
質問いいですか?

「証明されてない数学の何かの予想がある程度のところまでは問題なかったけど、物凄い桁数のところで否定される実例が挙げられてた。」という内容の厳密な証明って重要だよねっていう数学の読み物があって、この実例の部分が全く思い出せないんだけどわかる人居ます???

698 :132人目の素数さん:2020/06/19(金) 14:27:08.52 ID:IDrIk6aG.net
オイラー予想じゃないか?

699 :132人目の素数さん:2020/06/19(金) 14:29:41.07 ID:KOElBPqd.net
>>696
ありがとうございますスッキリしました

700 :132人目の素数さん:2020/06/19(金) 14:58:32.75 ID:3Zdulx62.net
>>698
ではなかったと思います
もっと恐怖を感じるレベルでした

701 :132人目の素数さん:2020/06/19(金) 15:10:27.49 ID:4cmZc3bL.net
その読み物ってもしかして「フェルマーの最終定理」?
だったら過大評価素数予想に対するスキューズ数では

702 :132人目の素数さん:2020/06/19(金) 15:20:43.60 ID:3Zdulx62.net
>>701
まさにそれです!ありがとうございます

703 :132人目の素数さん:2020/06/19(金) 16:59:18 ID:NS0YjiIe.net
Xを集合 U⊆(Xの巾集合)とすると、
Uを準基底とする開集合系の作り方は知られてますけど、
Uを基底とする開集合系って定まりますか?

704 :132人目の素数さん:2020/06/19(金) 17:02:29 ID:w+V5PFmt.net
>>696
w1,w2∈Uと間違えてないか?
そうすると<w1, f(w2)>=<f(w1), w2>が疑問だけど

705 :132人目の素数さん:2020/06/19(金) 17:04:00 ID:JfJoxKKB.net
>>704
そこで帰納法の仮定が使える

706 :132人目の素数さん:2020/06/19(金) 17:19:29.66 ID:w+V5PFmt.net
>>705
すまんどうやって帰納法使ってるのかもうちょい説明して欲しい
質問者じゃないのに申し訳ない

707 :132人目の素数さん:2020/06/19(金) 18:46:05.97 ID:x2LgfLh6.net
>>703
一般には明らかに不可能だろ

708 :132人目の素数さん:2020/06/19(金) 18:51:50.40 ID:+0+TEx0i.net
>>703
{∪V|V⊂U}が開集合族にならない例?
共通部分が入らなければいいんでしょ?

709 :132人目の素数さん:2020/06/19(金) 19:39:40.97 ID:JfJoxKKB.net
>>706
補題
Vがエルミート内積空間、f:V→Vが複素線形写像で以下が成立するとする。
A) f の固有値は全て実数
B) Aがf安定的(f(A)⊂A)ならA^⊥もf安定的
この時fはエルミート作用素。

方針は
1)fの固有空間Uはf安定的で、そこに制限すればfはエルミート
2)Uの直交補空間Wも仮定によりf安定的
3)fのWへの制限をgとすると(W,g)もA) B)を満たす。
(∵XがWのg安定的空間ならU+XのVにおける直交補空間がWにおけるXの直交補空間)

710 :132人目の素数さん:2020/06/20(土) 09:49:48.76 ID:pkUmXYcD.net
数学の証明や議論で常識外れの角度から切り込んでるキチガイじみたものってなんかある?
連続だが至る所微分不可能みたいなのよりもっとキチガイじみたもので。
将棋の羽生はプロをも超えるレジェンドだからプロが思いつくような手から逸れた常識外れの手を指してた。そんな感じで。

711 :132人目の素数さん:2020/06/20(土) 10:16:06.91 ID:crwGdaZH.net
>>710
岡潔とかグロタンディークとか

712 :132人目の素数さん:2020/06/20(土) 10:27:49 ID:QDhTfpJs.net
岡潔は世界中で同じような概念が出てるから違うのでは?

713 :132人目の素数さん:2020/06/20(土) 11:53:31.10 ID:0AD8TtTD.net
羽生レベルでいいんだからいいんじゃない

714 :132人目の素数さん:2020/06/20(土) 12:04:26.38 ID:pkUmXYcD.net
羽生をディスってる?

715 :132人目の素数さん:2020/06/20(土) 21:46:38.14 ID:crwGdaZH.net
もう将棋なんて計算機に負けてるオワコンだろ。
もう自動車を自分の足で追っかけて喜んでるフォレストガンプと大差ない。

716 :132人目の素数さん:2020/06/20(土) 21:59:01.52 ID:pkUmXYcD.net
>>715
一応プロ棋士名乗ってたYoutuberだったか誰かだったかがドヤ顔で反論してきた言葉
   「
   そーゆーこと言ってくる人居るけどー、そーゆー人にはこーゆー言葉返してるんだよー
   自動車は100年以上も前に人間より速いけど、だからといって100m走は無くなってないよね
   そーゆーこと
   」
だってさ

この言葉は余裕で論破できるけど、どういう風に論破できるか言ってみww

717 :132人目の素数さん:2020/06/20(土) 22:01:31.66 ID:pkUmXYcD.net
まぁだから、将棋がオワコンって事は正論なんだがな

今ってもう無料配布されてる将棋ソフトですら名人抜いてるんだろ?
Aperyだったか何だったか忘れたが

718 :132人目の素数さん:2020/06/20(土) 22:06:56.38 ID:pWavBVio.net
てか日本人にしか浸透してない競技てどうのこうの言われてもな
日本人って世界人口の何割よ
日本人が頭いいという宗教に入ってるならともかく、そうでなけりゃ日本の競技で強いからって世界的にどうのなんてへそが茶を沸かす議論だわ

719 :132人目の素数さん:2020/06/20(土) 22:11:23.80 ID:fY1zqfb2.net
黒人の100m走はほんとに世界でトップの話だよな
マイナー競技でキャッキャしてる将棋よりは才能がめちゃくちゃ必要
もちろん大半の数学者よりも必要だが

720 :132人目の素数さん:2020/06/20(土) 22:11:52.97 ID:hsq8T7LL.net
まあ羽生さんはチェスもめちゃくちゃ強いんですけどね

721 :132人目の素数さん:2020/06/20(土) 22:12:38.65 ID:fY1zqfb2.net
>>720
チェス(笑)強いからなんやねん

722 :132人目の素数さん:2020/06/20(土) 22:17:24.74 ID:dVO/fU9p.net
腕相撲強いから世界で一番つええんだよ羽生はよぉ!
と言いたいんだよぉ!

723 :132人目の素数さん:2020/06/20(土) 22:26:58.63 ID:wFhL4kfb.net
将棋はそもそも裾野が狭いからアホでも天才扱い
数学は学歴取るため世間的に広い競争が要求されるからそこそこ有能だが、雇用が広くアホでもなれる
短距離は全員が学校でやってて努力よりも才能で決まるものだから本当に上澄みが業界に入る

724 :132人目の素数さん:2020/06/20(土) 23:27:24.99 ID:crwGdaZH.net
>>710,716
短距離走や計算量ゴリ押しに奇策も糞もない
正攻法で現代数学勉強しろよカス

725 :132人目の素数さん:2020/06/20(土) 23:39:08.35 ID:H0q0bxsF.net
>>716
>  「
>   そーゆーこと言ってくる人居るけどー、そーゆー人にはこーゆー言葉返してるんだよー
>   自動車は100年以上も前に人間より速いけど、だからといって100m走は無くなってないよね
>   そーゆーこと
>   」
正しい

726 :132人目の素数さん:2020/06/20(土) 23:54:29.64 ID:pkUmXYcD.net
>>724
お前が自分の知らない分野については固定観念でディスるしか脳の無いアホなだけ
ということでお前の論破は失敗
何のヘイト溜まってんか知んねぇけど、俺が正攻法で現代数学勉強してないまで八つ当たりとか情緒不安定すぎやろww

727 :132人目の素数さん:2020/06/21(日) 01:35:07.80 ID:9Qpdto8a.net
短距離走で奇策に走る方がバカさ加減が際立つ。

728 :132人目の素数さん:2020/06/21(日) 01:37:24.29 ID:9Qpdto8a.net
まあ頑張って計算機使ったズルしてないことを証明してみせるんだな。
俺は普通に数学的に保証された認証技術を勉強するけど。

729 :132人目の素数さん:2020/06/21(日) 02:44:03.52 ID:iHniNJlg.net
>>710がどういうレベルの物を望んでいるのかイマイチよく分からない

730 :132人目の素数さん:2020/06/21(日) 11:40:57.51 ID:v7KbAOUS.net
最先端は全部が常識外れだろ

731 :132人目の素数さん:2020/06/21(日) 11:55:53.43 ID:Gp4DsLxp.net
割とそうでもない

732 :132人目の素数さん:2020/06/21(日) 14:02:32.48 ID:5HVpFOzx.net
https://i.imgur.com/L9uorO0.jpg
5番の問題がわかりません。x.y.zでそれぞれ微分したんですがその後がわからないです。

733 :132人目の素数さん:2020/06/21(日) 14:13:19.94 ID:i6WZBth6.net
0ですね

734 :132人目の素数さん:2020/06/21(日) 18:43:21.52 ID:U2E+Tv0g.net
このスレの人間、恐れを知らないな。
一回、潮が引いたばかりの浜辺に首から下を埋めてもらえばいいのに。

735 :132人目の素数さん:2020/06/21(日) 18:56:56.90 ID:509QOJ97.net
>>692
Wは任意のWだろ?
∀∃を意識して問題を写せ
任意のWと解釈すりゃ簡単

736 :132人目の素数さん:2020/06/21(日) 23:54:44.08 ID:7gY4JOhu.net
>>733
xyzはtによって変わるんだろうよ

737 :132人目の素数さん:2020/06/23(火) 23:47:47.00 ID:vViKCjGQ.net
1/(x^2+1)が実数の範囲で一様連続かどうかという問題で、微分係数の形にしないと上手く証明出来ないかなと思ったのですが、δをどのようにとったら上手く証明できますか?

738 :132人目の素数さん:2020/06/23(火) 23:51:13.56 ID:g2Oet10C.net
わからないんですね

739 :132人目の素数さん:2020/06/23(火) 23:57:21.46 ID:u1g7eSJn.net
>>737
微分してみたら?

740 :132人目の素数さん:2020/06/24(水) 00:00:18.43 ID:5s0rwNaq.net
>>739
やっぱり微分するしかないですかね

741 :132人目の素数さん:2020/06/24(水) 13:10:49.71 ID:854FP7B1.net
マルチ
回答済

742 :132人目の素数さん:2020/06/24(水) 13:44:20.65 ID:r8XE3H47.net
F=0のとき
dy/dx=-(∂F/∂x)/(∂F/∂y)
というのは
頭では分かるのだけど不思議
特に物理でボイルシャルルの法則で
(∂P/∂T)(∂T/∂V)(∂V/∂P)=-1
とかとても変な式にしか見えない

743 :132人目の素数さん:2020/06/24(水) 15:55:15.10 ID:9ySAKACB.net
"可付番順序数"についてどこか記述あればURL教えて下さい

744 :132人目の素数さん:2020/06/24(水) 16:02:17.68 ID:sQIniV6R.net
可付番より可算の方が恐らく圧倒的にメジャー
可算集合のwikipediaでもそれなりに分かりやすい

745 :132人目の素数さん:2020/06/24(水) 16:10:03.95 ID:9ySAKACB.net
>>744
俺が言ってるのは"可算集合"じゃ無くて"可付番順序数"な
全くの別概念

746 :132人目の素数さん:2020/06/24(水) 16:12:27.55 ID:9ySAKACB.net
可付番順序数の厳密な定義をちょっと聞きたくて。

747 :132人目の素数さん:2020/06/24(水) 17:00:32 ID:r8XE3H47.net
>>746
onajimono da yo

748 :132人目の素数さん:2020/06/24(水) 17:12:15.08 ID:9ySAKACB.net
>>747
じゃあそれぞれの定義と同値性が証明されてるところ教えて

749 :132人目の素数さん:2020/06/24(水) 17:17:02.24 ID:sQIniV6R.net
>>745
つまり可算順序数
順序数は集合なので、可算集合の順序数が可算順序数

750 :137番目の素数さん:2020/06/24(水) 17:32:23 ID:WvDwIBuG.net
アーベル群って結局
・分配法則が成り立つ
・零元がある
・逆元がある
の3つを満たす集合のことであってますか?
 あと線形空間の条件が8つあったのですが、要するに上の3つ+交換法則が成り立つっていうことであってますか?

751 :132人目の素数さん:2020/06/24(水) 17:44:45.88 ID:sQIniV6R.net
>>750
アーベル群に関しては2つ間違ってる
まず分配法則は演算2つに対して考えるので、演算が1つしかない群では出てこない
正解は結合法則
その上、結合法則、零元、逆元ではただの「群」
これに交換法則がくわわると「アーベル群」

これを踏まえてベクトル空間(線型空間)については、ベクトル空間の要素の全体がその和に関してアーベル群となる、つまり上の4つの条件を満たす+残りの4つとなる

752 :132人目の素数さん:2020/06/24(水) 18:43:56 ID:DVWoaAiL.net
A^3=[[0,a,b,c],[0,0,d,e],[0,0,0,f],[0,0,0,0]]
でるときA^4=0はどう示せば

753 :132人目の素数さん:2020/06/24(水) 19:07:47.34 ID:gs0zs19c.net
Aが冪零⇔A^4=O

754 :132人目の素数さん:2020/06/24(水) 19:52:53.59 ID:DVWoaAiL.net
>>753
冪零でA^12=0はわかりますけどA^4=0までどうしてわかるんですか?

755 :132人目の素数さん:2020/06/24(水) 19:59:32.08 ID:9ySAKACB.net
>>749
日評数学選書 現代集合論入門 竹内外史 8ページ
https://i.imgur.com/3Wc5rDR.jpg

ω^ωは非可算集合だろ
知らんのに出しゃばるなアホ

756 :132人目の素数さん:2020/06/24(水) 20:02:30.47 ID:9ySAKACB.net
お前ら>>749みたいなアホの言う事なんか何の信頼性も無いから無視でいいぞ

757 :132人目の素数さん:2020/06/24(水) 21:13:33.49 ID:r8XE3H47.net
>>748
dakara betumeidatte

758 :132人目の素数さん:2020/06/24(水) 21:15:09.05 ID:r8XE3H47.net
>>755
?
kasanshuugo no junjosuudakedo?

759 :132人目の素数さん:2020/06/24(水) 21:17:50.11 ID:r8XE3H47.net
>>746
>可付番順序数
mosikasite
ωto cofinal na yatu?
dokode mitandai?

760 :137番目の素数さん:2020/06/24(水) 21:29:39.35 ID:WvDwIBuG.net
>>751
ありがとうございます。

761 :137番目の素数さん:2020/06/24(水) 21:42:06.79 ID:WvDwIBuG.net
>>760
(x+y)+z=x+(y+z)
は結合法則でした。無知ですみません。

762 :132人目の素数さん:2020/06/24(水) 22:02:47.76 ID:JKVoD5V1.net
>>755
>ω^ωは非可算集合だろ

本当?
ググったらcountableだって出てきたんだが

http://math.bu.edu/people/tkohl/teaching/spring2020/532-Lecture-04-16-20-handout.pdf

http://dec41.user.srcf.net/h/II_L/logic_and_set_theory/2_3

http://cedric.milliet.free.fr/exo6c.pdf

763 :132人目の素数さん:2020/06/24(水) 22:03:30.59 ID:854FP7B1.net
>>742
どういう証明してるんだ?

764 :132人目の素数さん:2020/06/24(水) 22:04:13.77 ID:854FP7B1.net
>>748
見つからないのなら無いのさ
今時は「可算と可付番が同じ意味」がせいぜいだろ
>>755
ω^ωは集合の演算じゃなく順序数の演算で可算だ
>>749 は正しい

765 :132人目の素数さん:2020/06/24(水) 22:07:44.92 ID:854FP7B1.net
>>754
n 次正方行列 A が冪零なら A^n = 0
ジョルダン標準形にすれば即分かる

766 :132人目の素数さん:2020/06/24(水) 22:09:32.98 ID:gs0zs19c.net
この話なら整列可能じゃない?

767 :粋蕎 :2020/06/24(水) 23:51:50.91 ID:SonVvP1c.net
んんん?

>>762 >>764
門外漢に就き恥を承知で聞くんじゃが
aleph_0 と ω は違う言う事は立ち読みした本で見たが一方で
aleph_1=2^aleph_0
aleph_2=2^aleph_1
aleph_x=2^aleph_(x-1)
じゃろ?なら
ω も ω^2 も aleph_0 の要素範疇じゃろうけど
ω^ω どころか 2^ω からして aleph_1の範疇じゃろ?はて…

768 :132人目の素数さん:2020/06/24(水) 23:57:15.71 ID:y8wUYOca.net
>>767
> aleph_1=2^aleph_0

コレいわゆる連続体仮説ですな。


769 :132人目の素数さん:2020/06/25(Thu) 09:53:50 ID:epgEAfuS.net
加藤十吉「位相幾何学」問題 2.5 (p.47) より
R^n の中の図形 Y の中心 x の錐 X=x*Y がコンパクトであるとする.
中心 x のもう1つのコンパクトな錐 X’=x*Y’ に対し, X’ ⊂ X であり,
X’-Y’ が X の開集合のとき, Y と Y’ は同相になることを証明せよ.

※ この本での 錐 x*Y とは x*Y := { t.x + (1-t).y ; t∈[0,1], y∈Y } で定義される図形です.
ただし一意性: t.x + (1-t).y = t’.x + (1-t’).y’ ⇔ t=t’, y=y’ を前提条件とします.

もっと簡単な問題は巻末に略解が載っているのにこれはノーヒントでした.
具体例はいくつか挙げられても一般的な証明が思い浮かびません.

770 :132人目の素数さん:2020/06/25(木) 14:45:11.14 ID:gAVHynLj.net
>>767
aleph_x=2^aleph_(x-1) は一般連続体仮説で正しいとはされてない
ω^ω を集合や濃度の演算と思うのは誤解
これは順序数の演算で定義は ω^n の極限だから aleph_0

771 :132人目の素数さん:2020/06/25(木) 14:53:33.87 ID:TFMGu2uS.net
順序数の演算ならアレフ0=ωにはならんでしょ?

772 :132人目の素数さん:2020/06/25(木) 15:02:51.19 ID:gAVHynLj.net
>>769
y∈Y に対して
I(y) = { tx+(1-t)y | t∈[0,1] } とすれば X = ∪{ I(y) | y∈Y }
y'∈Y'⊂X'⊂X なら ∃y∈Y [ y'∈I(y) ] だから
これで y'∈Y' と y∈Y の対応が付く

773 :132人目の素数さん:2020/06/25(木) 15:49:40.54 ID:/1EHvcrg.net
>>769
仮定より x の近傍で XとX’は同相だから、>>772 のいうように対応をつければいい。

774 :769:2020/06/25(木) 17:36:01.91 ID:epgEAfuS.net
>>772, >>773 ありがとうございます
(1), (4) OK : 同相対応が取れる
(2) 不適格: X’-Y’ は X上の開集合ではない
(3) 不適格: X’ はコンパクトではない
一般のn次元について、前提条件を満たせば同相対応が取れること。
これを絵に頼らずロジックで示したいのです。
http://o.5ch.net/1ofkt.png

775 :132人目の素数さん:2020/06/25(木) 18:01:21.91 ID:DYnXYcHM.net
y∈Yに対しt(y)=min{ t | (y,t)∈X'}とおく。
t(y)=0とするとS={(y,t) | t>0}は開集合X'-Y'とdisjointだから{S={(y,t) | t≧0}もX'-Y'とdisjointとなるが、いずれもxを含むので矛盾する。
よってt(y)>0である。
よってf(y) = (y,t(y))は逆写像を持つ。

776 :132人目の素数さん:2020/06/25(木) 19:57:30.23 ID:gAVHynLj.net
>>771
誰も「アレフ0=ω」と言っとらん
順序数ωやω^ωの濃度がアレフ0なだけだ

777 :132人目の素数さん:2020/06/25(木) 20:19:17.45 ID:epgEAfuS.net
>>775
ありがとうございます。
(y, t) は「y∈Y とパラメータt で定まる点」なのですね。
Oが開集合で A∩O =φ なら 閉包(A) ∩O=φ なので etc. なんとか理解できました。
さらにコンパクト云々を考慮すると全単射連続写像 Y’ → Y は閉写像なので同相である。

778 :132人目の素数さん:2020/06/25(木) 21:14:44.81 ID:joqgUfXO.net
>>776
アレフ0=ωは定義。
アレフ0は無限基数の最小。
基数とは同じ濃度を持つ順序数の中で最小のもの。
したがってアレフ0はωになる。

779 :132人目の素数さん:2020/06/25(Thu) 21:29:34 ID:5Aqo2tkp.net
アレフ0=ωは定義。
したがってアレフ0はωになる。

でええやん。中の2行はなんやねん

780 :132人目の素数さん:2020/06/25(Thu) 21:32:28 ID:joqgUfXO.net
いや、基数の定義を知らない可能性もあると思って。

781 :132人目の素数さん:2020/06/25(木) 21:58:08.02 ID:Vl+MHKvd.net
|x|:=μα(∃可逆写像:α→x)
Card(κ):=∃x(κ=|x|)

782 :132人目の素数さん:2020/06/25(木) 22:03:37.51 ID:Vl+MHKvd.net
アレフ0:=ω
アレフ(α+1):=μκ(Card(κ)∧アレフ(α)<κ)
Lim(α)の時、アレフ(α):=∪{アレフ(β)|β<α}

が正式な定義
アレフ(α+1)は問題無く定義出来る(∵κ=|2^アレフ(α)|)

で、ここからアレフが狭義単調増加でもれなく無限基数を取り尽くしてる事が示される

783 :132人目の素数さん:2020/06/25(木) 22:42:16.82 ID:R/TM1uPM.net
>>767,768
それはベトbeth
それから
2^ω=ω
ω^ω=(((ω+ω+…(=ω^2))+(ω+ω+…(=ω^2))+…(=ω^3))+((ω+ω+…(=ω^2))+(ω+ω+…(=ω^2))+…(=ω^3))+…(=ω^4))+… … …
可算の可算個の合併に過ぎないので可算

784 :132人目の素数さん:2020/06/25(木) 22:46:13.17 ID:R/TM1uPM.net
>>755
>日評数学選書 現代集合論入門 竹内外史 8ページ
>https://i.imgur.com/3Wc5rDR.jpg
そこの可付番=可算だよ

785 :132人目の素数さん:2020/06/25(木) 22:46:38.55 ID:joqgUfXO.net
>>763
だからココで間違ってる奴はcardinal numberとirdinal numberの区別がついてないんだよ

786 :132人目の素数さん:2020/06/25(木) 22:48:55.55 ID:joqgUfXO.net
ordinal numverね。
ω^ωのcardinal はアレフ0だけどordinal numberとしてω^ωがωになるわけではない。
その区別がついてない。

787 :132人目の素数さん:2020/06/25(木) 23:53:13.70 ID:5Aqo2tkp.net
>>780
基数の定義なんか関係なく
「アレフ0=ωは定義。」ならアレフ0はωやん
何で基数の定義に言及する必要があるの?

788 :粋蕎 :2020/06/25(木) 23:57:42.78 ID:XozayM80.net
>>770
いや序数と基数は扱いが違うらしい事は解っとらんながら分かっとる。
今この場に限り基数関数の逆関数を序数関数と仮呼称し #^(-1)(x) と仮定義して言うと
#(ω)=aleph_0
#^(-1)(aleph_0)=ω
って事でええんじゃろ?
ω^ω=2^{ω*log{2}(ω)}=2^{ω*ln(ω)/ln(2)}=2^{ω*ln(ω)/ln(2)}=(2^ω)^{(ln(ω)/ln(2)}
={#^(-1)(2^aleph_0)}^{ln(ω)/ln(2)}
じゃけぇ aleph_0 じゃのうて aleph_1 じゃと思うが。

789 :132人目の素数さん:2020/06/26(金) 00:02:11.48 ID:MqOw0eEF.net
>>788
順序数に対するlogって何?

普通は、
ω^ω =(定義)∪{ω^n|n∈ω}
と計算する

で、任意の集合Xに対して、|X|≦アレフ0∧∀x∈X |x|≦アレフ0⇒|∪X|≦アレフ0が成り立つ

以上より、|ω^ω|≦アレフ0

790 :粋蕎 :2020/06/26(金) 00:03:21.58 ID:8ibqbiHa.net
>>783
んあ?ベート数beth_0?

791 :769:2020/06/26(金) 00:27:09.11 ID:R326Stdn.net
>>769 の件、あらためて考えてみました。
↓のような図形は問の条件を満たしているのに
自然な Y’→ Y 対応が連続写像になっていません。明らかに同相ではありません。
問の文章は一字一句そのままです。※以降の "錐" 定義にも欠落はないと思います。

なにか勘違いや見落としがあるでしょうか?
http://o.5ch.net/1ofsm.png

792 :132人目の素数さん:2020/06/26(金) 01:12:35.66 ID:/now+Nzk.net
>>787
ω^ωがωって言ってる奴はどこからか持ってきた順序数の冪の定義を引用した上で間違ってた。
こういうときはそもそも基数、順序数の定義を誤解してることが多い。
そこがあやふやなのでは何も議論ができないから確認のため書いたまで。

793 :132人目の素数さん:2020/06/26(金) 01:15:57.31 ID:/now+Nzk.net
>>791
それはYが入ってるR^nの中で錐を作ってる。
それではダメ。
YがR^nに入ってるなら、もう一つRをかけてその方向にxをとらないとダメ。

794 :132人目の素数さん:2020/06/26(金) 01:36:35 ID:R326Stdn.net
>>793 他での "錐" の定義はそうなのかもしれませんが、
この本では x, Y, X=x*Y 全て同じ R^n ユークリッド空間に置かれるのです。
たぶん病的な図形は扱わないのでこの先も問題は生じないと思いますが、これは雑な定義なのでしょうか。
https://i.imgur.com/VTzAmCU.png (p.44 より)

795 :132人目の素数さん:2020/06/26(金) 01:43:24 ID:/now+Nzk.net
>>794
ユークリッド幾何とかでなんかユークリッド空間の部分集合として考えるのに意味があるならそれでもいいけど、algebraic topologyとかでconeというとその空間を含んでるユークリッド空間の中でとる方がマイナー。
そう読めるなら著者もそこまで深く考えないで書いてるのかも。
数学者の書く本に間違いがないなどと思ってはいけない。

796 :132人目の素数さん:2020/06/26(金) 01:57:33 ID:gBZpsQR9.net
>>792
>ω^ωがωって言ってる奴
誰?

797 :132人目の素数さん:2020/06/26(金) 01:59:30.06 ID:lN2UznfQ.net
>>788
これだけ皆が説明してるのに
まだ間違うのが驚き!

798 :132人目の素数さん:2020/06/26(金) 03:29:52 ID:/now+Nzk.net
>>796
>>770

799 :132人目の素数さん:2020/06/26(金) 07:20:40 ID:gBZpsQR9.net
>>798
読解力無いな
>>770>>767への説明として書いているんだろ?
ω^ωの濃度はaleph_0という意図さね

800 :132人目の素数さん:2020/06/26(金) 07:47:30 ID:gBZpsQR9.net
>>781
>Card(κ):=∃x(κ=|x|)
κ=|κ|
でもいい?

801 :769:2020/06/26(金) 07:48:38 ID:R326Stdn.net
>>795
例えば >>794 の定義直後に書かれた錐拡大の説明とかを見ると
「X-{x0} と Y×(0,1] は同相である」と仮定しているように思われます。
著者はいちいち書かなくてもよいと思ったのかもしれません。大抵は気にしなくても満たされているので。
> 間違いがないなどと思ってはいけない。
確かにそうなので気をつけて読み進めようと思います。

802 :132人目の素数さん:2020/06/26(金) 08:05:21.79 ID:gBZpsQR9.net
>>791
>↓のような図形
とは?

803 :132人目の素数さん:2020/06/26(金) 08:09:47.26 ID:gBZpsQR9.net
>>801
同相じゃないの?

804 :132人目の素数さん:2020/06/26(金) 09:19:43.32 ID:AO9lGqa7.net
>>781
μって何?

805 :132人目の素数さん:2020/06/26(金) 10:50:24.00 ID:gBZpsQR9.net
>>804
最小
順序数全体も順序数的なクラスなので

806 :132人目の素数さん:2020/06/26(金) 11:20:03.51 ID:/now+Nzk.net
ばかばつか

807 :132人目の素数さん:2020/06/26(金) 12:46:08 ID:MqOw0eEF.net
>>800
∀x(||x||=|x|)は定理

808 :132人目の素数さん:2020/06/26(金) 15:22:58.54 ID:gBZpsQR9.net
>>807
そうじゃなくてCard(κ):= κ=|κ|でいい?ってこと

809 :132人目の素数さん:2020/06/26(金) 18:02:40.45 ID:MqOw0eEF.net
>>808
いや、自分で証明しろやw

810 :132人目の素数さん:2020/06/26(金) 18:25:14.75 ID:9b+ltZfm.net
>>805
書き方が特殊だけど理解した
サンクス

811 :132人目の素数さん:2020/06/26(金) 18:35:28.17 ID:gBZpsQR9.net
>>809
ダメってこと?
証明じゃなくてこれが定義でどうかってことだけど?

812 :132人目の素数さん:2020/06/26(金) 18:37:59.23 ID:gBZpsQR9.net
こっちの記述の方が不動点っぽくて素直だし

813 :132人目の素数さん:2020/06/27(土) 17:19:29 ID:NneRUAW/.net
>>811
|κ| が集合の場合しか使えない

814 :132人目の素数さん:2020/06/27(土) 17:25:15 ID:fLQwBj5R.net
複素関数で領域D内の点という言い方が出てきて
領域Dに含まれる点なのか領域Dの内点である点なのか明記はされてないのですが
通常どちらを指すものでしょうか?

815 :132人目の素数さん:2020/06/27(土) 17:54:40.55 ID:2s24CL/H.net
前者だろうけど、領域をCの連結開集合で定義してるだろうからどっちでもよくね

816 :132人目の素数さん:2020/06/27(土) 18:01:05.61 ID:fLQwBj5R.net
早速ありがとうございます

領域Dが開集合という記述は読んでいる教科書には見た限りなかったですが
複素関数領域Dで正則とか連続とかいうことは、通常、開集合Dに対して考えるものということでしょうか?
もしくは、領域Dを開集合と思って考えておいて応用上問題無いという感じでしょうか?

817 :132人目の素数さん:2020/06/27(土) 18:39:36 ID:hAOx7Ghg.net
>>816
おまえこれから領域という言葉を使うの禁止な
教科書の記述も全部連結開集合に書き換えて、教科書中の領域に関する主張は
全部、連結開集合として読んで矛盾しないか確認するまで使うの禁止

818 :132人目の素数さん:2020/06/27(土) 18:42:06.19 ID:eJ1jcmud.net
>>813
集合(順序数)じゃないのか?

819 :132人目の素数さん:2020/06/27(土) 20:04:27.72 ID:NneRUAW/.net
領域を連結開集合と書いてない教科書があるとは思わんかったな

820 :132人目の素数さん:2020/06/27(土) 21:17:54.41 ID:btd/y0AW.net
ごくたまに連結ではなく弧状連結で定義してるものもある
まあ開集合は変わらないけど

821 :132人目の素数さん:2020/06/27(土) 23:46:32.82 ID:H9rAvygY.net
>>819
> 領域を連結開集合と書いてない教科書があるとは思わんかったな
domainの訳語ではなくて、regionの訳語で領域を使っている微積の本はある。
高校数学周辺とかもそんな感じじゃない?

822 :132人目の素数さん:2020/06/27(土) 23:55:04.98 ID:XqRz5oW6.net
数学者は英語が下手なのではなく日本語が下手だから訳語がいい加減になるのか

823 :132人目の素数さん:2020/06/27(土) 23:57:17.47 ID:IOf8QY9v.net
高校数学の本を書いてるのは大体数学者ではないけどな

824 :132人目の素数さん:2020/06/28(日) 00:19:23.34 ID:zi6F4EcS.net
>>823

> 高校数学の本を書いてるのは大体数学者ではないけどな
学習指導要領に文句つけといてよ。

825 :132人目の素数さん:2020/06/28(日) 00:57:20.57 ID:9h9Pu3XV.net
>>823
数学を仕事の分野として選んだ人達のなかでどこからを数学者と呼ぶのか、その定義をちゃんと書いてみな。

826 :132人目の素数さん:2020/06/28(日) 01:22:58.13 ID:zI/dTtaS.net
カーッ(゚Д゚≡゚д゚)、ペッ

827 :132人目の素数さん:2020/06/28(日) 02:12:53.55 ID:NMwidY3t.net
>>819
なんで連結開集合に限定する??

828 :132人目の素数さん:2020/06/28(日) 02:22:17 ID:NMwidY3t.net
>>819
>領域
領域はむしろ
A region is a set consisting of a domain plus, perhaps, some or all of its boundary points. (The reader is warned that some authors use the term "region" for what we call a domain [following standard terminology], and others make no distinction between the two terms.)
こっちの訳語として使われることが多かろう

829 :132人目の素数さん:2020/06/28(日) 14:16:30.78 ID:H+4I9Y4I.net
>>827
微分可能の自然な定義は内点向けだし
連結でないと無意味な自由度がつく

830 :132人目の素数さん:2020/06/28(日) 15:52:17 ID:IFbL+hmS.net
形式から意味を読み取るのは簡単なのに、逆に意味から形式を導くのは更に困難な事について、
その原因をズバリと解説してるサイトってありますか?

831 :132人目の素数さん:2020/06/28(日) 20:14:47.14 ID:H+4I9Y4I.net
一般に成り立たない事の原因は個別事情に過ぎない

832 :132人目の素数さん:2020/06/28(日) 23:07:46.57 ID:QZe6TgM7.net
>>830
簡単の逆なのに「更に」困難って何と比較したのかわからなくなるな

833 :132人目の素数さん:2020/06/28(日) 23:24:16.24 ID:GNnWSYjY.net
都合のいい一方向関数やハッシュ関数が存在しないと現状のネット上の認証がすべて立ちいかなくなると
オラクルからのありがたい教えの毒電波、受信しましたあ!。

834 :132人目の素数さん:2020/06/30(火) 20:42:42.02 ID:LA56A/Qa.net
>>832
ストレートに答えを返せないからって、どうでも良いところの粗探ししかできないしょーもない人格要らんから
な?

835 :132人目の素数さん:2020/06/30(火) 21:01:55.59 ID:LA56A/Qa.net
>>832
俺が蔑んでるタイプの性格傾向やなお前

836 :132人目の素数さん:2020/06/30(火) 21:35:39.50 ID:tqLg6pWj.net
>>834
その気持ち分かるわ
なんかムカつくからビシッと言い返してやりたいけど良い返しが思い付かんときはそうやって人格否定にはしってしてまうよな

でもそんなレスしたところで結局負けた気分なって悔しくてたまらんくなるけど、どうすんのがええんやろな

837 :132人目の素数さん:2020/06/30(火) 22:28:55 ID:1Ml4Cz5+.net
>>836
人格否定を極めたら?達人にしか見えない世界がある

838 :132人目の素数さん:2020/06/30(火) 22:54:21 ID:ajKdRErt.net
重箱の隅など気にする方が分からん
過敏になる原因を調べたら?

839 :132人目の素数さん:2020/06/30(火) 23:02:30 ID:LA56A/Qa.net
>>836
ワロタww
質問に対して直で返すんが常識やろ

>>でもそんなレスしたところで結局負けた気分なって悔しくてたまらんくなるけど、どうすんのがええんやろな
レスを返したところで「ホラやっぱり負けた気分になってたんだろ〜」って準備した低次元なレスしてるけど
質問に直で返せばええだけやろ、出来ないんならだまっとけ
常識があれば、な。

840 :132人目の素数さん:2020/06/30(火) 23:51:53 ID:l6wZU4Bw.net
そういうのが嫌いだから数学に逃げてるのに
おまえらときたら・・・

841 :132人目の素数さん:2020/07/01(水) 01:10:25.45 ID:6R7JA3in.net
>>839
なるほどな
>>834みたいに煽りで返すやつは常識ないってことか
勉強なるわ

842 :132人目の素数さん:2020/07/01(水) 04:14:07 ID:soFMT7ws.net
>>835
かわいそう

843 :132人目の素数さん:2020/07/01(水) 07:18:56 ID:3MbPIell.net
まだネットどころかメディアもコンプライアンス徹底も普及せず、閉じた村社会が散在していた時代、
人格否定されるのを苦に自殺する人を量産していた人間が、ちょくちょく居た。

844 :132人目の素数さん:2020/07/01(水) 08:51:40.53 ID:CiSiomFB.net
ちょっと大学レベルかそれ以下かよくわからないのですが、
N進数、というときときNは普通2以上の「自然数」ですがこれを「実数」にすることは
可能でしょうか? (R進数と呼びましょうか)

まず素朴な疑問として、桁の数字、普通は0からN-1までの整数だけど、自然数の
場合は? 0以上の、底を超えない整数とか? (例えば底が3.5だったら、0, 1, 2, 3とか?)

そのような桁の数字で、級数 a_n * R^n + a_(n-1) * R^(n-1) + a_(n-2) * R^(n-2) + ... は
ちゃんと元の数に収束させることはできる?

これらがクリアーできればいいのかな?

845 :132人目の素数さん:2020/07/01(水) 14:00:26.07 ID:XbrWoU5H.net
e進数もある

846 :132人目の素数さん:2020/07/01(水) 15:56:35.63 ID:3MbPIell.net
ハウスドルフ、非整数有理数次元、非有理数実数次元

フラクタル図形

847 :132人目の素数さん:2020/07/04(土) 02:15:26.07 ID:7CHtHD9d.net
数学を専門に学ぶ人以外の数学は数学者が教えるべきじゃない
物理学者が教えるのがベストだと思う

848 :132人目の素数さん:2020/07/04(土) 02:16:21.77 ID:7CHtHD9d.net
数学者が教えるとどうしても原理原則だらけになって
全く実用的ではないから

849 :132人目の素数さん:2020/07/04(土) 04:19:25 ID:WLwo1mTc.net
>>847-848
語学じゃなくて言語学に相当するからな。純粋数学。
まあ文学に相当しちゃうのが一番ダメっぽいけど。

850 :132人目の素数さん:2020/07/04(土) 12:48:04.89 ID:nLP217oC.net
教育専門家の出番がないな

851 :132人目の素数さん:2020/07/04(土) 17:00:05.84 ID:FeJLVc4A.net
「部分集合の集合である」って言う述べ方する奴ウザいよな
AはXの部分集合の集合であるって言った場合、
単にA⊆Xとも読めるし、A⊆(Xの巾集合)とも読める
前者は「AはXの部分集合である集合である」という読み方だし、
後者は「AはXの部分集合からなる集合である」という読み方

自然言語でやるとこう言う厄介な事になるからマジで止めろ
死ね

一々こういう所で時間のロスをするから読解速度の邪魔をされるんだよ

852 :132人目の素数さん:2020/07/04(土) 17:37:36.53 ID:VlSg+iRT.net
>>851
前者と後者をそれぞれ英訳してみようか
(私案)
前者 "Let A be a set which is subset of X. "
または "Let A be a set such that A is a subset of X. "
後者 "Let A be a set of subsets of X. "

後者は普通にある表現だが、前者はあきらかに不自然で、
普通なら前者は "Let A be a subset of X. " と書かれるはず

後者を日本語訳すると「 A を X の部分集合の集合とする。」になる

もし「AはXの部分集合の集合である」を前者の意味で書いている文献があるなら、
そんな文献を読むのはやめるべき

853 :132人目の素数さん:2020/07/04(土) 18:57:45.98 ID:nLP217oC.net
コミュニケーション能力は必要だね

854 :132人目の素数さん:2020/07/04(土) 19:19:46.51 ID:Fvn4+d+y.net
>>851
そんな読み方するやついないだろう
お前の読解力が足りないだけや

855 :132人目の素数さん:2020/07/04(土) 19:59:15.41 ID:7CHtHD9d.net
>>851
Xの部分集合の集合Aとは
Aの元がXの部分集合だって意味以外無いが

>単にA⊆Xとも読める
読めませぬ

856 :132人目の素数さん:2020/07/04(土) 21:36:37.20 ID:FeJLVc4A.net
>>854
お前アホ過ぎやろ
書いてある文言が複数パターンの解釈が出来うる時は、前後の文脈との整合性を考えればどういう解釈が正しいかは自ずと見当が付く事ぐらいは当たり前だろ
だが、一々その見当に労力を使う事が無駄な集中力の浪費であって時間の無駄になるって言ってんだよ

 お 前 の 読 解 力 が 足 り な い だ け や

857 :132人目の素数さん:2020/07/04(土) 21:41:50.79 ID:FeJLVc4A.net
>>855
アホやな
つまり、「部分集合の集合である」と言った時の「の」が文法的にどういう役割を果たしてるかという事が原因で上記の解釈の分かれが起きてるって言ってんだよ
「の」が同格として働いてたらA⊆Xってなってしまうだろ。
そこで、前後の文脈を読む事で数学的理解としてA⊆Xなんだろうな」っていう感触を掴めるんだろうが

言ってる事分かるか?

自然言語(この場合は日本語)で書かれてる以上、それを読んだら、どんなに読解スピードがある人間でも、
  日本語としての理解 → 数学語としての理解
ってプロセスを経てる事を理解しろ

お前らは日本語としての理解をすっ飛ばして数学語としての理解だけでレスしようとしてるから的外れなレスになってんだよ

同じ事は「すべての○○にたいして××を成り立たせるような△△が存在する」って事でも言えるんだよ

858 :132人目の素数さん:2020/07/04(土) 21:56:14.12 ID:VlSg+iRT.net
>>857
>つまり、「部分集合の集合である」と言った時の「の」が文法的にどういう役割を果たしてるかという事が原因で上記の解釈の分かれが起きてるって言ってんだよ
>「の」が同格として働いてたらA⊆Xってなってしまうだろ。

部分集合は集合なんだから、その「の」が同格として働くと考えるのは数学的におかしい
したがって「部分集合の集合である」は「部分集合を要素に持つ集合」と解釈される
もしその文章が A ⊆ X を意味するなら、その文章を書いた人はセンスがないのでそこから先は読む必要がない

859 :132人目の素数さん:2020/07/04(土) 22:05:43.21 ID:7CHtHD9d.net
>>857
ハイハイその通り
おっしゃるとおりでございマシ

860 :132人目の素数さん:2020/07/04(土) 22:10:54.95 ID:Fvn4+d+y.net
>>856
だからそんな「前後の文脈から見当をつける」ことすらする必要がなく解釈の余地がないと言ってるんだが
ほんとばか

861 :132人目の素数さん:2020/07/04(土) 22:53:57.34 ID:FeJLVc4A.net
>>858
お前アホ過ぎ
会話になってない
低次元すぎ
相手してて疲れる
もうちょっと頭のレベル上げてから俺にレスしてこい

862 :132人目の素数さん:2020/07/04(土) 23:13:33.51 ID:Lw7BxtB4.net
論理で勝てないから相手をけなす
これは負けを認めている証拠

863 :132人目の素数さん:2020/07/05(日) 00:32:57.66 ID:kdMLUFmb.net
>>861
ハイハイその通りその通り

864 :132人目の素数さん:2020/07/05(日) 00:59:40 ID:YqxPp1BP.net
揉めてるような話で一生に使う労力なんてこの一連のレスに必要な労力より低いやろ

865 :132人目の素数さん:2020/07/05(日) 11:41:59.93 ID:b6WTgL0g.net
ただ勝ったと思いたいだけだろ

866 :132人目の素数さん:2020/07/05(日) 17:04:46.49 ID:M0qXA8vz.net
(d/dx) f(x) = g(x, f(x))という微分方程式がg(x_1, x_2)がある条件を満たすとき、初期条件を満たす解がある範囲で一意的に存在するという定理がありますが、
これは1階の微分方程式です。高階の微分方程式にも対応するような定理はあるのでしょうか?

867 :132人目の素数さん:2020/07/05(日) 21:32:04.89 ID:ovjWSxKQ.net
よろしくお願いします。
よくネットなどで微分商dx/dyのdx,dyが分離して取り扱える理由として、dxは微分形式として理解できると言うのを見かけるんですが、微分形式の除法を定義してdx/dyまで言及してるのを見たことがありません。
この場合はやっぱりウェッジ積に関するdyの逆元が存在してdx∧dy^-1だという主張なのでしょうか?
自分としてはdx/dyは、双対空間の元としてdyの逆元dy^-1が存在したときの、dxとのテンソル積のことだと思うのですが。

868 :132人目の素数さん:2020/07/05(日) 21:42:13 ID:al86bpUS.net
いえ違います
ポアンカレの補題です

869 :132人目の素数さん:2020/07/05(日) 23:05:00.34 ID:b6WTgL0g.net
>>866
f, g をベクトルにすれば高階にも対応する

870 :132人目の素数さん:2020/07/05(日) 23:10:31.97 ID:b6WTgL0g.net
>>867
微分形式を使いたい時は大抵多変数だからdx/dyなど意味ない

871 :132人目の素数さん:2020/07/05(日) 23:16:49.10 ID:kdMLUFmb.net
>>867
タダの変数だから自由自在に計算したらよろし
べつにウェッジしか考えちゃ行かんわけじゃナイ

872 :132人目の素数さん:2020/07/05(日) 23:39:31.12 ID:iZND6hdT.net
dxやdyを接束上の実数値関数と思えば普通の割り算

873 :132人目の素数さん:2020/07/06(月) 02:31:52 ID:q2noIBqO.net
一次近似までは許すが二次近似より高次の無限小は断じて許さん!!
て態度が結実結晶化すると

d・d=0、冪ゼロ性

になる

874 :132人目の素数さん:2020/07/06(月) 05:04:30 ID:d1BDPeOt.net
d^2=0である理由が二次のゼロだからではないでしょ?
もしそうならdxdyだって0じゃん?

875 :粋蕎 :2020/07/06(月) 06:13:31.71 ID:vT6vjXzA.net
んなもん式を整理せんと分からんじゃろ
st(ε)=0
無限小の代わりに零因子を用いる自動微分なんてのに頼る手も有るが
其れじゃと量子力学を習う衆が困るじゃろ

其れにしても遺憾、あんだけ皆が指摘・指導してくれたのに
基数と序数の違いをどうしても本能的に解釈しようとする orz
こら何ヵ月か経ったらリセットされるわ…下手に、慣れる前に感覚で覚える癖が抜けん orz

876 :132人目の素数さん:2020/07/06(月) 09:26:20.18 ID:q2noIBqO.net
>>874
http://www.ne.jp/asahi/search-center/internationalrelation/mathWeb/Limit/LimitOfFunctionOfTwoVariables/LandausSymbol.htm
ランダウの記号 無限小解析でググったら出て来た

877 :132人目の素数さん:2020/07/06(月) 10:17:17.32 ID:d1BDPeOt.net
>>876
違う。
意味わかってないからそんな間違いするんだよ。
そんなネットでググって勉強なんか無理。
ちゃんと教科書買って取り組まむしかないね。

878 :132人目の素数さん:2020/07/06(月) 10:33:13.08 ID:q2noIBqO.net
微分形式は微分形式で知っては居る。
相当いい加減な意味で物理屋さんの感覚的な無限小解析的当て推量だとこんなもんだろ。
>>873>>872を受けて流れで言ってるだけだし。

879 :132人目の素数さん:2020/07/06(月) 13:34:18.57 ID:FI2iVHF+.net
面積要素や体積要素で良いだろ

880 :132人目の素数さん:2020/07/06(月) 20:48:05.19 ID:371/0oCm.net
デデキンドの切断って循環論法じゃないの?
有理数体を二つに分けて境い目がどっちにも属さない場合は無理数って、どっちにも属さない数の存在はどこで定義されたの?

881 :132人目の素数さん:2020/07/06(月) 22:10:15.59 ID:oeqLTvIf.net
そういう分割を有理数でない数(実数)とよんだ。

882 :132人目の素数さん:2020/07/06(月) 23:27:35.01 ID:mZy63Gf9.net
>>880
> デデキンドの切断って循環論法じゃないの?
> 有理数体を二つに分けて境い目がどっちにも属さない場合は無理数って、どっちにも属さない数の存在はどこで定義されたの?
有理数が完備かどうかは、実数を定義しなくても議論できる。

883 :132人目の素数さん:2020/07/07(火) 00:28:23.73 ID:yBUx0unO.net
平方して2になる有理数がない事の証明は簡単だから
どっちにも属さない数の存在は定義じゃなく証明だな

884 :132人目の素数さん:2020/07/07(火) 04:37:27 ID:N+43tJM6.net
それは違う

885 :132人目の素数さん:2020/07/07(火) 06:47:18 ID:jKy5QVgS.net
何で違うの?

886 :132人目の素数さん:2020/07/07(火) 06:49:23 ID:jKy5QVgS.net
>>880
は?
どこが循環論法??
どっちにも属さない数が存在するかどうかは別の話だが

887 :132人目の素数さん:2020/07/09(Thu) 10:55:35 ID:U5n26Tbv.net
マクローリン展開を利用して小数第○位まで求めよって問題は式を途中で切って代入しますよね?このときに代入しなかった項の和が小数第○位に影響しないことを証明しないといけないですよね?

888 :132人目の素数さん:2020/07/09(Thu) 11:10:11 ID:5Qw7UOqY.net
はい

889 :132人目の素数さん:2020/07/09(Thu) 11:13:03 ID:U5n26Tbv.net
一般項を不等式で評価して無限等比級数の和の公式使うって流れでいいですよね?

890 :132人目の素数さん:2020/07/09(Thu) 11:23:09 ID:5Qw7UOqY.net
>>889
場合による
臨機応変に

891 :132人目の素数さん:2020/07/09(Thu) 14:34:29 ID:locj3z5H.net
内田伏一の本のツォルンの補題の証明に出てくるf-列のモチベーションは何ですか?

892 :132人目の素数さん:2020/07/09(Thu) 14:39:36 ID:IUMTTboY.net
>>891
Zornの補題に出てくる帰納的順序集合ってのは、いくらでも上界が取ってこれるってものだから、
この性質を使って、この上界を真の上界が取れなくなるまで超限的に真の上界を取ってきて集めていく作業がZornの補題のアイデア

で、これを一番ドストレートに証明してるのが超限帰納法を用いた証明だけど、素朴集合論の立場からは超限帰納法は使いにくい(?)から、
直接的には超限帰納法は使わず上記アイデアを実現していこうとするものが、素朴集合論の立場からのZornの補題の証明
だと俺は理解してる。

893 :132人目の素数さん:2020/07/09(木) 15:01:35.43 ID:Nd0aBl0H.net
>>891
松坂の本で構成する列と同じものか分からないけど、同じものならこれが分かりやすい
https://togetter.com/li/967760

894 :132人目の素数さん:2020/07/09(木) 15:25:36.79 ID:IUMTTboY.net
ちなみに、整列可能定理と超限帰納法を使えば数行で証明出来るw

895 :132人目の素数さん:2020/07/09(木) 15:29:02.02 ID:IUMTTboY.net
Zornの補題ってホント面白いんだよな
主張・証明に現れてくる用語はいずれもかなり初歩的で高校生でも理解できるし、素朴集合論の立場からの証明だって他の定理を一切使わず初等的に証明出来る
にもかかわらず、ガチ証明を分かった気になるだけでも6時間ぐらい掛かる
こういう定理好きだわ

896 :132人目の素数さん:2020/07/09(Thu) 16:28:23 ID:0Axmwauv.net
ZFCの言葉で全数学の授業を一貫して行うべきだと全世界で主張したい
選択公理すらまともに使えないまま大学を卒業していいのだろうかということ

897 :132人目の素数さん:2020/07/09(木) 17:13:51.44 ID:4YwVxgvf.net
覚えたてが良く陥る症状

898 :132人目の素数さん:2020/07/09(木) 17:41:36.98 ID:Nd0aBl0H.net
圏論をZFCでやる強者

899 :132人目の素数さん:2020/07/09(Thu) 18:24:43 ID:HTnVBh6a.net
>>892
>素朴集合論の立場からは超限帰納法は使いにくい
ソなの?

900 :132人目の素数さん:2020/07/09(Thu) 18:40:05 ID:IUMTTboY.net
>>899
ZFCでの超限帰納法は順序数を使って行うし、超限帰納法を使ったクラス関数の定義も順序数を使って行う。
松坂の素朴集合論の立場の順序数は定義がZFCでのそれとは違ってるが、わざわざオーソドックスじゃ無い流儀での議論なんて知らん

901 :132人目の素数さん:2020/07/09(木) 19:33:51.15 ID:locj3z5H.net
みなさんありがとうございました。ツォルンの補題の証明を読み終わりましたが、論理を追っただけという感じがします。
>>893を読んでみます。

902 :132人目の素数さん:2020/07/09(木) 19:44:31.80 ID:7/Hkj4s9.net
>>893の3行ぐらい、読んでみますって書いてる間に読めそうやけどな

903 :132人目の素数さん:2020/07/10(金) 00:29:27.01 ID:6bCWgN2c.net
完全理論と範疇的理論て1階の場合は同じですか?

904 :132人目の素数さん:2020/07/10(金) 00:53:13 ID:sXjd7Mgj.net
>>900
>松坂の素朴集合論
そういう独特のものののことだったのね

905 :132人目の素数さん:2020/07/10(金) 02:29:35 ID:uBVQvXu7.net
つい先日まで、竹内外史の現代集合論入門を流し読みしたけど、集合論に興味あるなら松坂の集合論のパート読むより、竹内の方がいいかも。
解説のくどいぐらいの丁寧さなら松坂の方が上だけど。

906 :132人目の素数さん:2020/07/10(金) 04:33:19 ID:uBVQvXu7.net
>>904
訂正
松坂の集合論に代表される順序数の取り扱いは、「順序型」なる取り扱いで、これは古いやり方。
現代は「順序数」を扱ってる。

(田中尚之、「公理的集合論」、68ページ)

907 :132人目の素数さん:2020/07/10(金) 07:45:15 ID:+yglHiCg.net
アホな質問とは思うんですが、このページのベストアンサーの「各列ベクトルの大きさは1になるから」のところ、イコールゼロなんですがなぜですか?
ユニタリー行列系の問題で前にも列ベクトルの大きさについてイコール1なら納得の行くところ、イコール0で書いてるものを見たことがあります
https://detail.chiebukuro.yahoo.co.jp/qa/question_detail/q12180381882

908 :132人目の素数さん:2020/07/10(金) 08:39:24 ID:vAAETInD.net
整列可能定理の証明で(A, α), (B, β)に対し、両者が整列集合として一致するか、または(A, α)が(B, β)の切片となっているとき
(A, α) ≦ (B, β)
と書くと書いてあります。

「(A, α)が(B, β)の切片となっている」というのは、A = B<b>となっていてかつA上でαとβが一致するという意味ですか?
それとも単にA = B<b>となっているという意味ですか?

909 :132人目の素数さん:2020/07/10(金) 10:52:38.44 ID:WTcwT03Q.net
>>108
前後の文章読まないと確定的にはわからないけどその記号の雰囲気なら

(A,α) ≦ (B,β)
:⇔∃f: A→B
. 1) f は単射で順序を保つ
. 2) fは全射である、または∃b∈B f(A)=[minB,b)

だろうな。
でもあくまで整列順序集合(クラス)の“同値類”に順序“≦”を導入したいんだろうから(A,α)/〜≦(B,β)/〜とかするか、先にクラスOrdを構成しておくかするのが普通のバズだけど初学者むけの教科書なら多分前者をやってるんだろな。

910 :132人目の素数さん:2020/07/10(金) 13:38:28 ID:uBVQvXu7.net
だからぁ〜〜〜〜、いっただろ
ZFCから順序数の議論に行かないからそんな議論になるんだよ
「順序型」で勉強してもZFCをやる時には再度順序数で議論やり直しになるんだぞ?

911 :132人目の素数さん:2020/07/10(金) 14:31:05 ID:vAAETInD.net
ところで整列可能定理はそれを使ってなにかを証明するのに使えるというような定理ですか?

912 :132人目の素数さん:2020/07/10(金) 14:44:21 ID:sFmsKvbI.net
選択公理やZornの補題が証明できる

913 :132人目の素数さん:2020/07/10(金) 14:47:06.34 ID:sFmsKvbI.net
一番意味があるのは任意濃度の順序数が存在する事の証明だな

914 :132人目の素数さん:2020/07/10(金) 17:11:03.14 ID:MQI8tKO+.net
>>908
αとかβって何

915 :132人目の素数さん:2020/07/10(金) 17:22:58 ID:sXjd7Mgj.net
>>907
何で疑問?疑問の余地無く明らかに間違い
だいたいabcdって実数?

916 :132人目の素数さん:2020/07/11(土) 08:45:59.77 ID:afQ2TK3g.net
数学科1年です
数学科向けの微積と線形の演習本があれば教えてください

917 :132人目の素数さん:2020/07/11(土) 13:00:11.83 ID:jttbO0Eg.net
大学入学時に買わされたけど、そっちじゃ売ってないの?

918 :132人目の素数さん:2020/07/11(土) 14:28:00 ID:Ry98196t.net
>>916
微分積分  聖文新社 全問精解 微積分演習
線形代数  聖文新社 基礎演習 線形代数
この2冊にたいていの問題は載っている 
辞典みたいな本

919 :132人目の素数さん:2020/07/11(土) 19:39:27.05 ID:dDydxdCs.net
田中尚之の「公理的集合論」で順序数の足し算、かけ算の定義が変な定義してるんだが、何で一々こんな流儀でやってんのかな?
普通は超限帰納法による定義やろ?
こっちの方がどう見ても議論がスッキリしてんのに。

920 :132人目の素数さん:2020/07/11(土) 19:44:36.95 ID:dDydxdCs.net
>>919
Kunenの集合論でも順序数の足し算が田中のと同じだな

921 :132人目の素数さん:2020/07/12(日) 00:42:30 ID:/QjIt0XQ.net
ネットワーク分析を理解するのに必要な大学の数学はなんですか。線形代数で十分ですか。

922 :132人目の素数さん:2020/07/12(日) 00:51:13.54 ID:uH7re/7e.net
全く知らんけど、情報数理の分野に進む人間にとって、グラフ理論・オートマトン・タブローとかの述語計算はほぼ必修やな

923 :132人目の素数さん:2020/07/12(日) 01:16:34 ID:KU+Qkyf2.net
ラマヌジャンの映画の宣伝で
現代では通信ネットワークでラマヌジャン・グラフが使われ出してるみたいにいってた
が、なにかは調べてない

924 :132人目の素数さん:2020/07/12(日) 09:59:58 ID:kzyqt/oe.net
有限次元ベクトル空間V→Vで任意の基底に対して同じ表現行列を持つような線形写像について教えて欲しいです

925 :132人目の素数さん:2020/07/12(日) 10:15:48.50 ID:yoaFs/0o.net
>>924
idの定数倍だっつーの

926 :132人目の素数さん:2020/07/12(日) 13:13:19 ID:V/YTPeNS.net
証明は簡単だった

927 :132人目の素数さん:2020/07/12(日) 20:53:58.46 ID:W1JXwaf7.net
単位付きの実数、例えば速さa(m/s)のような表記を単位ごと元として説明できるような代数系ってありますか?
MKS単位系だとR上の〜だと思うんですが…

928 :132人目の素数さん:2020/07/12(日) 21:03:16 ID:W1JXwaf7.net
m、kg、sの加減乗除とRの作用で生成されるような気もするんですが、それだけだと0(m)=0(s)とかなっちゃう気がしてます

929 :132人目の素数さん:2020/07/12(日) 21:10:57 ID:vkf1E1Cj.net
成分ごとに単位が設定されてるとしたときのRのいくつかの直和の元でいいんじゃね

930 :132人目の素数さん:2020/07/12(日) 21:15:28 ID:vkf1E1Cj.net
あ、直和そのままだと乗除が上手くいかないか
単位系の生成する乗法群を添字とするRの直和で、積に関しては単位Aの成分aと単位Bの成分bが単位ABの成分になるようにシフトさせればいい
次数づけみたいな感じで

931 :132人目の素数さん:2020/07/12(日) 21:32:05.03 ID:W1JXwaf7.net
>>930
ありがとうございます。もう少し考えてみます。物理っぽい話になってしまうのですがベクトルに単位を直接つけてる記述が気になってしまって。単位ってスカラーにつけるものだと思ってたんで。逆にもともと単位を含んだ数学的構造があれば、それで作った数ベクトルで物理をやろうと思ったんですがわけが分からなくなってしまいました。

932 :132人目の素数さん:2020/07/12(日) 21:42:34.05 ID:Vi21ufw4.net
>>931 そういうのが気になるなら、次が参考になる

量の計算を見直す
http://math.oshirase.com/%E9%87%8F%E3%81%AE%E8%A8%88%E7%AE%97%E3%82%92%E8%A6%8B%E7%9B%B4%E3%81%99/

933 :132人目の素数さん:2020/07/12(日) 21:50:26.14 ID:W1JXwaf7.net
>>932
すごい!ざっと見ただけですが、求めていたのはこれです!ありがとうございます!

934 :132人目の素数さん:2020/07/12(日) 22:20:47.74 ID:W1JXwaf7.net
量代数っていう言葉があるんですね
物理の学生はしってるんでしょうか?

935 :132人目の素数さん:2020/07/12(日) 23:12:46.41 ID:V/YTPeNS.net
生成元に除算も追加した多元環で充分だろ
4元単位系なら基本生成元は m, g, s, A で
これらの乗除 m^2, m/s, As = C を基底とする線形空間だ

936 :132人目の素数さん:2020/07/12(日) 23:46:20.58 ID:YW9E3TBl.net
>>932
これを知っていることに感心した

937 :132人目の素数さん:2020/07/13(月) 00:09:33.64 ID:+93kDUzK.net
>>932
アホじゃね?
加速度をxとしようがxcm/s^2としようが
どっちでもいいじゃん
この人の主張に沿うとそうなるんだが?

938 :132人目の素数さん:2020/07/13(月) 00:12:20.41 ID:+93kDUzK.net
mathematica最初に見た時
単位は文字(定数・変数)に過ぎない扱いで
よくわかってるなと思ったよ

939 :132人目の素数さん:2020/07/13(月) 00:33:38.11 ID:KdFhSu4T.net
>>937
ただの文字だとすると、0m≠0sってどうやって説明しますか?

940 :132人目の素数さん:2020/07/13(月) 00:34:44.16 ID:+93kDUzK.net
物理単位で面倒くさいのは
>>936
のように生成元がどれと言えないことがあるから
単純な例で言えば
物理的な不変量は光速だから
長さは速度・時間とすべきかもということ

941 :132人目の素数さん:2020/07/13(月) 00:36:33.68 ID:+93kDUzK.net
>>939
1m+1sとかな
何が出来る操作
成り立つ等式であるかはまた別ってことだろ

942 :132人目の素数さん:2020/07/13(月) 00:41:05.10 ID:+93kDUzK.net
>>939
0m=0sで悪いのはおそらく足し算を考えるから
1mに足せるのは0mで0sでないと考えたいんだろ

943 :132人目の素数さん:2020/07/13(月) 00:43:36.91 ID:+93kDUzK.net
けれど
1m+1sとは(1,1)(m,s)という2次元の物理量と考えれば
0m=0m+0s=0sとしていいかもね

944 :132人目の素数さん:2020/07/13(月) 00:46:09.01 ID:+93kDUzK.net
0mとは(0,y)(m,s)の総称で
0sとは(x,0)(m,s)の総称と見てもいいかも

945 :132人目の素数さん:2020/07/13(月) 02:47:16 ID:3VCpyfW1.net
ちょっと簡単なところで詰まってしまった。

自然数nに対し、I_n:={k/2^n|k=0,...,2^n}, I := ∪_{n∈N}I_n
と置くと、Iは閉区間[0,1]で稠密である事を示せ
∀r,s∈[0,1](r<s⇒∃i∈I r<i<s)を示せと言った方がいいか

946 :132人目の素数さん:2020/07/13(月) 03:04:21 ID:ajZXf4F6.net
n→∞で1/2^n→0なので
十分大きいNをとれば1/2^N<s-r
ここで開区間(r,s)にk/2^Nがいないとすれば
k/2^N≦rとなる最大のkをとるとs≦(k+1)/2^N
よってs-r≦1/2^Nとなり矛盾

947 :132人目の素数さん:2020/07/13(月) 04:25:16 ID:3VCpyfW1.net
>>946
サンクス
頭柔らかいね

948 :132人目の素数さん:2020/07/13(月) 08:49:49 ID:frUNQB2h.net
集合論において、整列集合はなぜ重要視されているのですか?
整列可能定理にしても、それを弱めた全順序集合化可能定理という定理があってもいいはずです。

949 :132人目の素数さん:2020/07/13(月) 09:30:55 ID:6OZiXRze.net
>>948
高々可算集合は性質がいい集合だよね
理由の一つに直感的な命題の証明なら数学的帰納法で殴れることがあるよね
一方任意の集合に対しては一見帰納法を適用できないように見えるけど、整列すれば超限帰納法が使えるよね
色々殴れて嬉しいね

950 :132人目の素数さん:2020/07/13(月) 10:02:07.69 ID:JBKnqi5p.net
加法群Z/4ZからZ/6Zへの準同型を全て求めよ
この問題を教えてください
大学一年で右も左もわかりません

951 :132人目の素数さん:2020/07/13(月) 10:56:34 ID:IUBSZ97G.net
自分もわからないが考えればできそうな気はするが
準同型はf(x+y)=f(x)+f(y)だろ
総当りでいけばいけるかと

952 :132人目の素数さん:2020/07/13(月) 10:58:49 ID:tCFI/pDQ.net
加法群Z/nZタイプの準同型は1の行き先で決まる

Z/4Z={0,1,2,3}からZ/6Z= {0,1,2,3,4,5}への準同型fを考える
まずf(0)=0は決まってる
次にf(1)=aとすると準同型性によって
f(2)=f(1+1)=f(1)+f(1)=a+a=2a
f(3)=f(1+1+1)=f(1)+f(1)+f(1)=a+a+a=3a
f(4)=f(1+1+1+1)=f(1)+f(1)+f(1)+f(1)=a+a+a+a=4a
と他は自動的に行き先が決まってしまう
ところがZ/4Zにおいては4=0だから最後の式はf(0)=0に一致しなくてはいけない
よって0=4a ∈Z/6Z
これからa=0もしくはa=3であることがわかる
(Z/6Zにおいて4×3=12=0であることに注意)
a=0のときは
f(0)=0、f(1)=0、f(2)=0、f(3)=0
a=3のときは
f(0)=0、f(1)=3、f(2)=6=0、f(3)=9=3
となる
この2パターンしかないことがわかった

953 :132人目の素数さん:2020/07/13(月) 11:12:04 ID:IUBSZ97G.net
適当に数値いれてみたところ
f(0)=f(0)+f(0)からf(0)=0は必須で
f(1)を0〜5としてf(4)=f(0)を求めると成立は0と3のときだけでは

954 :132人目の素数さん:2020/07/13(月) 11:15:49 ID:haDuED7+.net
>>950
Z/4Zの元xを1+4Z、Z/6Zの元yを1+6Zとし、Kを<2x>、Iを<3y>どする。
fを求める準同型の一つとする。
f(x) = ayとなる整数をとる。
f(x) = f(3x - 2x) = f(3x) = 3ay
によりa=2にとれる時はf=0。
aが奇数の時a=2b+1となる整数をとる時
f(x)=3(2b+1)y=3y。
以上によりf(x)=0,3yのいずれか。
また準同型g(t)=3t+6Zで定められる準同型g:Z→Z/6Zはg(4)=0によりZ→Z/4Zを通過するが、それをf0:Z/4Z→X/6Zとするとf0(x)=3y。
以上により0でない準同型Z/4Z→Z/6Zはf0のみ。



よりf(x)∈Iだからim

955 :132人目の素数さん:2020/07/13(月) 11:40:23 ID:qT5aGSv6.net
Hom(Z/nZ,Z/mZ)=Z/dZ,d=gcd(n,m)
Hom(Z/4Z,Z/6Z)=Z/2Z
2個

956 :132人目の素数さん:2020/07/13(月) 13:06:10 ID:XZzDguGr.net
>>948
定理として残るのは役に立つものだけ

957 :132人目の素数さん:2020/07/13(月) 13:31:53 ID:frUNQB2h.net
>>949
>>956
ありがとうございました。


「整列可能定理によって、ある整列集合(W,≦)と全単射f:W->R-{0}が存在する。Wの部分集合W_0を、超限帰納法により、次のように構成しよう。
f(a)が{f(x)|x∈W<a>∩W_0}に属する有限個の実数のQ上一次結合と成り得ないとき、そのときに限りa∈W_0とする。B={f(a)|a∈W_0}とおけば、
集合Bがハメル基である。」

とハメル基の存在を証明していますが、理解できません。まず最初の全単射が存在するという部分ですが、W=R-{0}として、fを恒等写像とすればよい
というのは間違っていませんか?

後半の超限帰納法のところは超限帰納法を使った議論を見たのが初めてのため全く分かりません。解説をお願いします。

958 :132人目の素数さん:2020/07/13(月) 13:44:59 ID:frUNQB2h.net
超限帰納法のベースケースが分かりません。

959 :132人目の素数さん:2020/07/13(月) 14:00:05 ID:tR49RdlH.net
>>957
全単射が存在するのは整列可能定理を仮定してるんだから当たり前じゃないの?

960 :132人目の素数さん:2020/07/13(月) 14:26:40 ID:frUNQB2h.net
>>959
ある整列集合として、R-{0}が取れるというのであっていますか?

961 :132人目の素数さん:2020/07/13(月) 14:52:31 ID:3VCpyfW1.net
こういう風な疑問で質問し合う状況が起きてしまうって事を著者は書いてて想像出来んかったんかな
想像出来てたら予め行間飛ばさずに丁寧に書けてたはずなんだがな

962 :132人目の素数さん:2020/07/13(月) 15:07:02.86 ID:tR49RdlH.net
>>960
ああ、そうか、その本の定義ならそうなるね。
あってます。

963 :132人目の素数さん:2020/07/13(月) 15:16:51.53 ID:tR49RdlH.net
>>960
その本の証明が周りくどくてわかりにくく感じてしまうのは、もう少し進んだ集合論の教科書なら

-整列可能定理-
∀S ∃Z ∃f:S→Z s.t. Z:順序数、f:全単射

の形にする事が多いから。
多分その本の著者は読者がそのレベルの話を意識してる。
もしその本の定義に従うなら定理を

∀S ∃≦ s.t. ≦はSに整列順序集合の構造を与える

の形にしてそのまま使う方がいいのだけどそうはしてないね。

964 :132人目の素数さん:2020/07/13(月) 15:19:03.16 ID:tR49RdlH.net
日本語へんになった。
著者はそのレベルの話しを意識してるね。
初学者向きの本で“簡単な定義”にしておくけど、普段自分たちが専門家同士で話するときはそんな“簡単な定義”なんてつかわないからなぁ。

965 :132人目の素数さん:2020/07/13(月) 15:59:30.74 ID:XZzDguGr.net
そもそも <a> の意味がわからんし
超限帰納法の使い方も分かりにくい

966 :132人目の素数さん:2020/07/13(月) 16:09:11 ID:frUNQB2h.net
W<a> = {x ∈ W | x < a}です。

967 :132人目の素数さん:2020/07/13(月) 16:21:41 ID:6OZiXRze.net
>>957
間違いではないけど本の記述の意図は汲み取れてなさそうに見える
>>963の二つの定義の同値性は分かる?

968 :132人目の素数さん:2020/07/13(月) 16:36:24 ID:3VCpyfW1.net
整列可能定理は、文字通り整列順序が存在するという主張であって、
整列集合に対して順序同型になる順序数が一意に存在するという定理とは別物。

969 :132人目の素数さん:2020/07/13(月) 16:47:07 ID:tR49RdlH.net
>>967
オレ質問者じゃないよ

970 :132人目の素数さん:2020/07/13(月) 16:57:54 ID:6OZiXRze.net
>>969
分かってる

971 :132人目の素数さん:2020/07/13(月) 16:59:37 ID:tR49RdlH.net
>>969
じゃなんでオレに聞くの?
オレの書いた事間違ってると思うなら間違い指摘して書けばいいじゃん?
なんでオレに聞くの?

972 :132人目の素数さん:2020/07/13(月) 17:04:36 ID:6OZiXRze.net
>>971
よく見ろ
お前には何も聞いてない

973 :132人目の素数さん:2020/07/13(月) 17:24:18.65 ID:tR49RdlH.net
>>972
ああ、すまん、質問者に聞いてるのね。
どうぞ教えてあげてください。

974 :132人目の素数さん:2020/07/13(月) 18:02:42.85 ID:mgUkZYtM.net
用語が多く混乱しているのですが、
階数1の局所自由層=可逆層=直線束=階数1のベクトル束
ですか?

975 :132人目の素数さん:2020/07/13(月) 22:46:19.56 ID:XZzDguGr.net
>>957 の記述って定義されてない物を超限帰納法で定義に使う事
についての説明はあったの?

976 :132人目の素数さん:2020/07/14(火) 00:16:48.70 ID:/OIO0Eke.net
小ネタ

実は「超限帰納法」は一般化できるんだよな
超限帰納法は順序数の持つ整列性を使ってるんだけど、この整列性を一般化させて「有基底的述語」って概念があって、
この有基底的述語に関する超限帰納法が成立する。

ググってみ

977 :132人目の素数さん:2020/07/14(火) 05:11:07 ID:ZW1OJ+xR.net
>>975
一切ないです。内田伏一著 集合と位相です。

978 :132人目の素数さん:2020/07/14(火) 13:04:03 ID:Qjs63CAj.net
>>976
ググったけど安心して読めるものはないな

979 :132人目の素数さん:2020/07/14(火) 13:50:16.53 ID:aWst/W6m.net
PAQ =(Er,O,O,O)となるP,Qって一意に定まりますか?
定まらないとするとPQの組はどのように求めればいいのでしょう
特にAが正則でないときに興味があります

980 :132人目の素数さん:2020/07/14(火) 14:25:41.66 ID:U7HKkSXI.net
>>974
これ分かる人っていますか?

981 :132人目の素数さん:2020/07/14(火) 15:22:51 ID:XiX/JMdJ.net
>>980
noether scheme上くらいなら正しいような。

982 :132人目の素数さん:2020/07/14(火) 17:27:31.73 ID:/yg7odXq.net
>>979
Pのs倍とQの1/s倍も条件を満たす
P,Qは
[A E]
[E 0]
を掃き出して
[PAQ P]
[Q 0]
にすれば求まる

983 :132人目の素数さん:2020/07/14(火) 17:52:09.88 ID:aWst/W6m.net
>>982
これが条件を満たすというのは分かるのですが、
答えの組が、この基本変形で求めたP,Qとそのスカラー倍以外にない
というのはどのように示せるのでしょうか
P,Qが正則なのかということも併せて教えてほしいです

984 :132人目の素数さん:2020/07/14(火) 17:55:36.48 ID:y7y+i5EA.net
>>981
豊富な直線束というwikipediaの記事を見ると、「Xをスキームまたは複素多様体とし、FをX上の層とする。(中略)Fが直線束であったとする。つまり局所自由なランク1であったとすると〜」と、任意のスキームX上で同じかのように書いてありますが、暗黙のうちにネーター性が使われてるんですかね
難しいのであまり理解できていませんが

985 :132人目の素数さん:2020/07/14(火) 18:04:33.48 ID:GQr/Yd2r.net
>>983
P,Qが正則なものだと、PAQの1,2行目を入れ換えて1,2列目を入れ換えてもPAQのままだからBP,QBの形の行列も条件を満たす
正則でないものだと、PをPAQPに置き換えても条件を満たす

986 :132人目の素数さん:2020/07/14(火) 18:23:20.04 ID:/dYVKwK3.net
>>984
いや、noetherスキームじゃないのなんかやった事ないからわからん。
いらんのかも。

987 :132人目の素数さん:2020/07/14(火) 19:17:23.12 ID:Qjs63CAj.net
>>983
掃き出しを構成する行列は全部正則
1と1, 0と0を交換しても変わらん

988 :132人目の素数さん:2020/07/14(火) 20:02:04.22 ID:aWst/W6m.net
>>985
書いてあることがよくわからなかったのですが、
もとの条件からはP,Qが正則であるとは言えないということですか
>>987
掃き出しを構成する行列も、その積も正則なのは分かっていて
基本変形でP,Qを求めればP,Qが正則というのは分かるのですが
基本変形で求められないP,Qの組はないのか
元の条件からP,Qが正則だと言えるのか、というのが分かりません

989 :132人目の素数さん:2020/07/14(火) 20:56:15 ID:GQr/Yd2r.net
>>988
書き方悪かった

簡単のためにAがn次正方行列(n>1)の場合のみ考えるが一般の行列でもだいたい同じ
掃き出しによって条件を満たす正則行列P,Qが得られたとする
Bを左からかけると1,2行目を入れ換えるような基本行列とすると、BP,QBも条件を満たす正則行列である
またPAQP,Qは条件を満たすがr<nならばPAQPが正則でない

990 :132人目の素数さん:2020/07/14(火) 22:23:27.36 ID:ZW1OJ+xR.net
Aをm×n行列、Bをn×m行列とし、C=A*Bとする。m≠nならばCは正則行列ではないことを示せ。

991 :132人目の素数さん:2020/07/14(火) 22:25:36.32 ID:xAU4yFd0.net
A=[[1,0,0],[0,1,0]]
B=[[1,0],[0,1],[0,0]]

992 :132人目の素数さん:2020/07/14(火) 22:32:36.67 ID:ZW1OJ+xR.net
>>991
ありがとうございました。

993 :132人目の素数さん:2020/07/14(火) 22:47:45.29 ID:/5FQKZKF.net
>>948
はぁ
弱めてどうする
整列可能なんだから当然全順序化されてるわ

994 :132人目の素数さん:2020/07/14(火) 22:58:21.91 ID:/5FQKZKF.net
>>957
>超限帰納法
そこでやってるのも超限帰納法と呼ぶんだ
超限帰納的な定義って呼ぶんだと思ってた
(超限)帰納法は証明手段のことだとばかり

995 :132人目の素数さん:2020/07/14(火) 23:01:56.05 ID:/5FQKZKF.net
>>992
ありがとうで良いのか? m>nじゃなくてか?

996 :132人目の素数さん:2020/07/15(水) 00:06:55.24 ID:UCnbD9aG.net
>>994
これ超限帰納的な定義っていうか?

997 :132人目の素数さん:2020/07/15(水) 00:38:28.28 ID:/Q3fMOpu.net
>>996
言わんか?aを組み込むか否かをaより下の条件で決めてるじゃん

998 :132人目の素数さん:2020/07/15(水) 05:28:28 ID:xmF7sJYz.net
次スレ建てました。
https://rio2016.5ch.net/test/read.cgi/math/1594758474/l50

999 :132人目の素数さん:2020/07/15(水) 10:59:39.04 ID:GC4Nr3Jf.net
離散位相、密着位相という名前はなぜそうつけられたんですか?直感的な説明をお願いします。

1000 :132人目の素数さん:2020/07/15(水) 11:17:50 ID:/Q3fMOpu.net
近さを測る分離する=開集合→全部バラララ=離散位相→全部一緒=密着位相

1001 :2ch.net投稿限界:Over 1000 Thread
2ch.netからのレス数が1000に到達しました。

総レス数 1001
280 KB
掲示板に戻る 全部 前100 次100 最新50
read.cgi ver.24052200